Anda di halaman 1dari 279

There is decreased secretion of which one of the following hormones in response to

major surgery:

A. Insulin

B. Cortisol

C. Renin

D. Anti diuretic hormone

E. Prolactin
Endocrine parameters reduced in
stress response:

 Insulin
 Testosterone
 Oestrogen

Insulin is often released in decreased quantities following surgery.

Stress response: Endocrine and metabolic changes

 Surgery precipitates hormonal and metabolic changes causing the stress


response
 Stress response is associated with: substrate mobilization, muscle protein loss,
sodium and water retention, suppression of anabolic hormone secretion,
activation of the sympathetic nervous system, immunological and
haematological changes.
 The hypothalamic-pituitary axis and the sympathetic nervous systems are
activated and there is a failure of the normal feedback mechanisms of control
of hormone secretion.

A summary of the hormonal changes associated with the stress response:

Increased Decreased No Change


Growth hormone Insulin Thyroid stimulating hormone
Cortisol Testosterone Luteinizing hormone
Renin Oestrogen Follicle stimulating hormone
Adrenocorticotrophic hormone (ACTH)
Aldosterone
Prolactin

Antidiuretic hormone
Glucagon

Sympathetic nervous system

 Stimulates catecholamine release


 Causes tachycardia and hypertension

Pituitary gland

 ACTH and growth hormone (GH) is stimulated by hypothalamic releasing


factors, corticotrophin releasing factor (CRF) and somatotrophin (or growth
hormone releasing factor)
 Perioperative increased prolactin secretion occurs by release of inhibitory
control
 Secretion of thyroid stimulating hormone (TSH), luteinizing hormone (LH)
and follicle stimulating hormone (FSH) does not change significantly
 ACTH stimulates cortisol production within a few minutes of the start of
surgery. More ACTH is produced than needed to produce a maximum
adrenocortical response.

Cortisol

 Significant increases within 4-6h of surgery (>1000 nmol litre-1)


 The usual negative feedback mechanism fails and concentrations of ACTH
and cortisol remain persistently increased
 The magnitude and duration of the increase correlate with the severity of stress
and the response is not abolished by the administration of corticosteroids.
 The metabolic effects of cortisol are enhanced:

Skeletal muscle protein breakdown to provide gluconeogenic precursors and amino


acids for protein synthesis in the liver
Stimulation of lipolysis
'Anti-insulin effect'
Mineralocorticoid effects
Anti-inflammatory effects

Growth hormone

 Increased secretion after surgery has a minor role


 Most important for preventing muscle protein breakdown and promote tissue
repair by insulin growth factors

Alpha Endorphin

 Increased

Antidiuretic hormone

 An important vasopressor and enhances haemostasis


 Renin is released causing the conversion of angiotensin I to angiotensin II,
which causes the secretion of aldosterone from the adrenal cortex. This
increases sodium reabsorption at the distal convoluted tubule

Insulin

 Release inhibited by stress


 Occurs via the inhibition of the alpha cells in the pancreas by the α2-
adrenergic inhibitory effects of catecholamines
 Insulin resistance by target cells occurs later
 The perioperative period is characterized by a state of functional insulin
deficiency

Thyroxine (T4) and tri-iodothyronine (T3)

 Circulating concentrations are inversely correlated with sympathetic activity


and after surgery there is a reduction in thyroid hormone production, which
normalises over a few days.

Metabolic effect of endocrine response

Carbohydrate metabolism

 Hyperglycaemia is a main feature of the metabolic response to surgery


 Due to increased increase in glucose production and a reduction in glucose
utilization
 Catecholamines and cortisol promote glycogenolysis and gluconeogenesis
 Initial failure of insulin secretion followed by insulin resistance affects the
normal responses
 The proportion of the hyperglycaemic response reflects the severity of surgery
 Hyperglycaemia impairs wound healing and increase infection rates

Protein metabolism
 Initially there is inhibition of protein anabolism, followed later, if the stress
response is severe, by enhanced catabolism
 The amount of protein degradation is influenced by the type of surgery and
also by the nutritional status of the patient
 Mainly skeletal muscle protein is affected
 The amino acids released form acute phase proteins (fibrinogen, C reactive
protein, complement proteins, a2-macroglobulin, amyloid A and
ceruloplasmin) and are used for gluconeogenesis
 Nutritional support has little effect on preventing catabolism

Lipid metabolism
Increased catecholamine, cortisol and glucagon secretion, and insulin deficiency,
promotes lipolysis and ketone body production.

Salt and water metabolism

 ADH causes water retention, concentrated urine, and potassium loss and may
continue for 3 to 5 days after surgery
 Renin causes sodium and water retention

Cytokines

 Glycoproteins
 Interleukins (IL) 1 to 17, interferons, and tumour necrosis factor
 Synthesized by activated macrophages, fibroblasts, endothelial and glial cells
in response to tissue injury from surgery or trauma
 IL-6 main cytokine associated with surgery. Peak 12 to 24 h after surgery and
increase by the degree of tissue damage Other effects of cytokines include
fever, granulocytosis, haemostasis, tissue damage limitation and promotion of
healing.

Modifying the response

 Opioids suppress hypothalamic and pituitary hormone secretion


 At high doses the hormonal response to pelvic and abdominal surgery is
abolished. However, such doses prolong recovery and increase the need for
postoperative ventilatory support
 Spinal anaesthesia can reduce the glucose, ACTH, cortisol, GH and
epinephrine changes, although cytokine responses are unaltered
 Cytokine release is reduced in less invasive surgery
 Nutrition prevents the adverse effects of the stress response. Enteral feeding
improves recovery
 Growth hormone and anabolic steroids may improve outcome
 Normothermia decreases the metabolic response
References
Deborah Burton, Grainne Nicholson, and George Hall
Endocrine and metabolic response to surgery .

Contin Educ Anaesth Crit Care Pain (2004) 4(5): 144-147


doi:10.1093/bjaceaccp/mkh040
Which of the following statements related to the coagulation cascade is true?

A. The intrinsic pathway is the main pathway in coagulation

B. Heparin inhibits the activation of Factor 8

C. The activation of factor 8 is the point when the intrinsic and the
extrinsic pathways meet

D. Tissue factor released by damaged tissue initiates the extrinsic


pathway

E. Thrombin converts plasminogen to plasmin

The extrinsic pathway is the main path of coagulation. Heparin inhibits the activation
of factors 2,9,10,11. The activation of factor 10 is when both pathways meet.
Thrombin converts fibrinogen to fibrin. During fibrinolysis plasminogen is converted
to plasmin to break down fibrin.

Coagulation cascade

Two pathways lead to fibrin formation

Intrinsic pathway (components already present in the blood)

 Minor role in clotting


 Subendothelial damage e.g. collagen
 Formation of the primary complex on collagen by high-molecular-weight
kininogen (HMWK), prekallikrein, and Factor 12
 Prekallikrein is converted to kallikrein and Factor 12 becomes activated
 Factor 12 activates Factor 11
 Factor 11 activates Factor 9, which with its co-factor Factor 8a form the tenase
complex which activates Factor 10

Extrinsic pathway (needs tissue factor released by damaged tissue)

 Tissue damage
 Factor 7 binds to Tissue factor
 This complex activates Factor 9
 Activated Factor 9 works with Factor 8 to activate Factor 10

Common pathway

 Activated Factor 10 causes the conversion of prothrombin to thrombin


 Thrombin hydrolyses fibrinogen peptide bonds to form fibrin and also
activates factor 8 to form links between fibrin molecules

Fibrinolysis
Plasminogen is converted to plasmin to facilitate clot resorption

Image sourced from Wikipedia

Intrinsic pathway Increased APTT Factors 8,9,11,12


Extrinsic pathway Increased PT Factor 7
Common pathway Increased APTT & PT Factors 2,5,10
Vitamin K dependent Factors 2,7,9,10
Which of the following is not secreted by the parietal cells?

A. Hydrochloric acid

B. Mucus

C. Magnesium

D. Intrinsic factor
E. Calcium
Chief of Pepsi cola = Chief cells
secrete PEPSInogen

Parietal cells: secrete HCl, Ca, Na, Mg and intrinsic factor


Chief cells: secrete pepsinogen
Surface mucosal cells: secrete mucus and bicarbonate

Gastric secretions

A working knowledge of gastric secretions is important for surgery because peptic


ulcers are common, surgeons frequently prescribe anti secretory drugs and because
there are still patients around who will have undergone acid lowering procedures
(Vagotomy) in the past.

Gastric acid

 Is produced by the parietal cells in the stomach


 pH of gastric acid is around 2 with acidity being maintained by the H+/K+ ATP
ase pump. As part of the process bicarbonate ions will be secreted into the
surrounding vessels.
 Sodium and chloride ions are actively secreted from the parietal cell into the
canaliculus. This sets up a negative potential across the membrane and as a
result sodium and potassium ions diffuse across into the canaliculus.
 Carbonic anhydrase forms carbonic acid which dissociates and the hydrogen
ions formed by dissociation leave the cell via the H+/K+ antiporter pump. At
the same time sodium ions are actively absorbed. This leaves hydrogen and
chloride ions in the canaliculus these mix and are secreted into the lumen of
the oxyntic gland.

This is illustrated diagrammatically below:


Image sourced from Wikipedia

Phases of gastric acid secretion


There are 3 phases of gastric secretion:

1. Cephalic phase (smell / taste of food)

 30% acid produced


 Vagal cholinergic stimulation causing secretion of HCL and gastrin release
from G cells

2. Gastric phase (distension of stomach )

 60% acid produced


 Stomach distension/low H+/peptides causes Gastrin release

3. Intestinal phase (food in duodenum)

 10% acid produced


 High acidity/distension/hypertonic solutions in the duodenum inhibits gastric
acid secretion via enterogastrones (CCK, secretin) and neural reflexes.
Regulation of gastric acid production
Factors increasing production include:

 Vagal nerve stimulation


 Gastrin release
 Histamine release (indirectly following gastrin release) from enterchromaffin
like cells

Factors decreasing production include:

 Somatostatin (inhibits histamine release)


 Cholecystokinin
 Secretin

The diagram below illustrates some of the factors involved in regulating gastric acid
secretion and the relevant associated pharmacology

Image sourced from Wikipedia

Below is a brief summary of the major hormones involved in food digestion:

Source Stimulus Actions


Gastrin G cells in Distension of Increase HCL, pepsinogen and IF
antrum of stomach, secretion, increases gastric motility,
the stomach extrinsic nerves trophic effect on gastric mucosa
Inhibited by: low
antral pH,
somatostatin
CCK I cells in Partially digested Increases secretion of enzyme-rich
upper small proteins and fluid from pancreas, contraction of
intestine triglycerides gallbladder and relaxation of sphincter
of Oddi, decreases gastric emptying,
trophic effect on pancreatic acinar
cells, induces satiety
Secretin S cells in Acidic chyme, Increases secretion of bicarbonate-rich
upper small fatty acids fluid from pancreas and hepatic duct
intestine cells, decreases gastric acid secretion,
trophic effect on pancreatic acinar cells
VIP Small Neural Stimulates secretion by pancreas and
intestine, intestines, inhibits acid and pepsinogen
pancreas secretion
Somatostatin D cells in Fat, bile salts and Decreases acid and pepsin secretion,
the pancreas glucose in the decreases gastrin secretion, decreases
and stomach intestinal lumen pancreatic enzyme secretion, decreases
insulin and glucagon secretion
inhibits trophic effects of gastrin,
stimulates gastric mucous production

A 45 year old male is diagnosed with carcinoma of the head of the pancreas. He
reports that his stool sticks to the commode and will not flush away. Loss of which of
the following enzymes is most likely to be responsible for this problem?

A. Lipase

B. Amylase

C. Trypsin

D. Elastase

E. None of the above

Theme from April 2012 Exam


Loss of lipase is one of the key features in the development of steatorrhoea which
typically consists of pale and offensive stools that are difficult to flush away.

Pancreatic cancer

 Adenocarcinoma
 Risk factors: Smoking, diabetes, Adenoma, Familial adenomatous polyposis
 Mainly occur in the head of the pancreas (70%)
 Spread locally and metastasizes to the liver
 Carcinoma of the pancreas should be differentiated from other periampullary
tumours with better prognosis
Clinical features

 Weight loss
 Painless jaundice
 Epigastric discomfort (pain usually due to invasion of the coeliac plexus is a
late feature)
 Pancreatitis
 Trousseau's sign: migratory superficial thrombophlebitis

Investigations

 USS: May miss small lesions


 CT Scanning (pancreatic protocol). If unresectable on CT then no further
staging needed.
 PET/CT for those with operable disease on CT alone
 ERCP/ MRI for bile duct assessment.
 Staging laparoscopy to exclude peritoneal disease.

Management

 Head of pancreas: Whipple's resection (SE dumping and ulcers). Newer


techniques include pylorus preservation and SMA/ SMV resection.
 Carcinoma body and tail: poor prognosis, distal pancreatectomy if operable.
 Usually adjuvent chemotherapy for resectable disease
 ERCP and stent for jaundice and palliation.
 Surgical bypass may be needed for duodenal obstruction.

Which of the following is not well absorbed following a gastrectomy?

A. Vitamin c

B. Zinc

C. Vitamin B12

D. Copper

E. Molybdenum

Vitamin B12. The others are unaffected

Post gastrectomy syndrome:

Rapid emptying food from stomach into the duodenum: diarrhoea, abdominal pain,
hypoglycaemia
Complications: Vitamin B12 and iron malabsorption, osteoporosis
Treatment: High protein, low carbohydrate diet. Replace B12/Fe/Ca

Gastric emptying

 The stomach serves both a mechanical and immunological function. Solid and
liquid are retained in the stomach during which time repeated peristaltic
activity against a closed pyloric sphincter will cause fragmentation of food
bolus material. Contact with gastric acid will help to neutralise any pathogens
present.

 The amount of time material spends in the stomach is related to its


composition and volume. For example a glass of water will empty more
quickly than a large meal. The presence of amino acids and fat will all serve to
delay gastric emptying.

Controlling factors
Neuronal stimulation of the stomach is mediated via the vagus and the
parasympathetic nervous system will tend to favor an increase in gastric motility. It is
for this reason that individuals who have undergone truncal vagotomy will tend to
routinely require either a pyloroplasty or gastro-enterostomy as they would otherwise
have delayed gastric emptying.

The following hormonal factors are all involved:

Delay emptying Increase emptying


Gastric inhibitory peptide Gastrin
Cholecystokinin
Enteroglucagon

Diseases affecting gastric emptying


All diseases that affect gastric emptying may result in bacterial overgrowth, retained
food and eventually the formation of bezoars that may occlude the pylorus and make
gastric emptying even worse. Fermentation of food may cause dyspepsia, reflux and
foul smelling belches of gas.

Iatrogenic
Gastric surgery can have profound effects on gastric emptying. As stated above any
procedure that disrupts the vagus can cause delayed emptying. Whilst this is
particularly true of Vagotomy this operation is now rarely performed. Surgeons are
divided on the importance of vagal disruption that occurs during an oesophagectomy
and some will routinely perform a pyloroplasty and other will not.
When a distal gastrectomy is performed the type of anastomosis performed will
impact on emptying. When a gastro-enterostomy is constructed, a posterior, retrocolic
gastroenterostomy will empty better than an anterior one.

Diabetic gastroparesis
This is predominantly due to neuropathy affecting the vagus nerve. The stomach
empties poorly and patients may have episodes of repeated and protracted vomiting.
Diagnosis is made by upper GI endoscopy and contrast studies, in some cases a radio
nucleotide scan is needed to demonstrate the abnormality more clearly. In treating
these conditions drugs such as metoclopramide will be less effective as they exert
their effect via the vagus nerve. One of the few prokinetic drugs that do not work in
this way is the antibiotic erythromycin.

Malignancies
Obviously a distal gastric cancer may obstruct the pylorus and delay emptying. In
addition malignancies of the pancreas may cause extrinsic compression of the
duodenum and delay emptying. Treatment in these cases is by gastric decompression
using a wide bore nasogastric tube and insertion of a stent or if that is not possible by
a surgical gastroenterostomy. As a general rule gastroenterostomies constructed for
bypass of malignancy are usually placed on the anterior wall of the stomach (in spite
of the fact that they empty less well). A Roux en Y bypass may also be undertaken but
the increased number of anastomoses for this in malignant disease that is being
palliated is probably not justified.

Congenital Hypertrophic Pyloric Stenosis


This is typically a disease of infancy. Most babies will present around 6 weeks of age
with projectile non bile stained vomiting. It has an incidence of 2.4 per 1000 live
births and is more common in males. Diagnosis is usually made by careful history and
examination and a mass may be palpable in the epigastrium (often cited seldom felt!).
The most important diagnostic test is an ultrasound that usually demonstrates the
hypertrophied pylorus. Blood tests may reveal a hypochloraemic metabolic alkalosis
if the vomiting is long standing. Once the diagnosis is made the infant is resuscitated
and a pyloromyotomy is performed (usually laparoscopically). Once treated there are
no long term sequelae.
Which vitamin is involved in the formation of collagen?

A. Vitamin A

B. Vitamin B

C. Vitamin C

D. Vitamin D

E. Vitamin E

Vitamin C is needed for the hydroxylation of proline during collagen synthesis.


Collagen

One of the major connective tissue proteins

 Composed of 3 polypeptide strands that are woven into a helix


 Numerous hydrogen bonds exist within molecule to provide additional
strength
 Many sub types but commonest sub type is I (90% of bodily collagen)
 Vitamin c is important in establishing cross links

Collagen Diseases

 Osteogenesis imperfecta
 Ehlers Danlos

Osteogenesis imperfecta:
-8 Subtypes
-Defect of type I collagen
-In type I the collagen is normal quality but insufficient quantity
-Type II- poor quantity and quality
-Type III- Collagen poorly formed, normal quantity
-Type IV- Sufficient quantity but poor quality
Patients have bones which fracture easily, loose joint and multiple other defects
depending upon which sub type they suffer from

Ehlers Danlos:
-Multiple sub types
-Abnormality of types 1 and 3 collagen
-Patients have features of hypermobility.
-Individuals are prone to joint dislocations and pelvic organ prolapse. In addition to
many other diseases related to connective tissue defects
A 56 year old man has long standing chronic pancreatitis and develops pancreatic
insufficiency. Which of the following will be absorbed normally?

A. Fat

B. Protein

C. Folic acid

D. Vitamin B12

E. None of the above


Pancreatic lipase is required for digestion of fat, Proteases facilitate protein and B12
absorption. Folate digestion is independent of the pancreas.

Pancreas exocrine physiology

Pancreatic juice

 Alkaline solution pH 8
 1500ml/day
 Composition: acinar secretion (ENZYMES: trypsinogen, procarboxylase,
amylase, lecithin) and ductile secretion (HCO, Na+, water)
 Pancreatic juice action: Trypsinogen is converted via enterokinase to active
trypsin in the duodenum. Trypsin then activates the other inactive enzymes.

A 56 year old male presents to the acute surgical take with severe abdominal pain. He
is normally fit and well. He has no malignancy. The biochemistry laboratory contacts
the ward urgently, his corrected calcium result is 3.6 mmol/l. What is the medication
of choice to treat this abnormality?

A. IV Pamidronate

B. Oral Alendronate

C. Dexamethasone

D. Calcitonin

E. IV Zoledronate

Theme from January 2012 exam

IV Pamidronate is the drug of choice as it most effective and has long lasting effects.
Calcitonin would need to be given with another agent, to ensure that the
hypercalcaemia is treated once its short term effects wear off. IV zoledronate is
preferred in scenarios associated with malignancy.

Management of hypercalcaemia

 Free Ca is affected by pH (increased in acidosis) and plasma albumin


concentration
 ECG changes include: Shortening of QTc interval
 Urgent management is indicated if:
Calcium > 3.5 mmol/l
Reduced consciousness
Severe abdominal pain
Pre renal failure

Management:

 Airway Breathing Circulation


 Intravenous fluid resuscitation with 3-6L of 0.9% Normal saline in 24h
 After hydration, give frusemide (to encourage excretion of Ca)
 Medical therapy (usually if Corrected calcium >3.0mmol/l)

Bisphosphonates

 Analogues of pryrophosphate
 Prevent osteoclast attachment to bone matrix and interfere with osteoclast
activity.
 Inhibit bone resorption.

Agents

Drug Side effects Notes


IV pyrexia, leucopaenia Most potent agent
Pamidronate
IV Zoledronate response lasts 30 Used for malignancy associated
days hypercalcaemia

Calcitonin

 Quickest onset of action however short duration (tachyphylaxis) therefore only


given with a second agent.

Prenisolone

 May be given in hypercalcaemia related to sarcoidosis, myeloma or vitamin D


intoxication.

n over enthusiastic medical student decides to ask you questions about ECGs. Rather
than admitting your dwindling knowledge on this topic, you bravely attempt to
answer her questions! One question is what segment of the ECG represents
ventricular repolarization?

A. QRS complex
B. Q-T interval

C. P wave

D. T wave

E. S-T segment

Theme from January 2012 exam

The T wave represents ventricular repolarization. The common sense approach to


remembering this, is to acknowledge that ventricular repolarization is the last phase of
cardiac contraction and should therefore correspond the the last part of the ECG.

The normal ECG

Image sourced from Wikipedia

P wave

 Represents the wave of depolarization that spreads from the SA node


throughout the atria
 Lasts 0.08 to 0.1 seconds (80-100 ms)
 The isoelectric period after the P wave represents the time in which the
impulse is traveling within the AV node
P-R interval

 Time from the onset of the P wave to the beginning of the QRS complex
 Ranges from 0.12 to 0.20 seconds in duration
 Represents the time between the onset of atrial depolarization and the onset of
ventricular depolarization

QRS complex

 Represents ventricular depolarization


 Duration of the QRS complex is normally 0.06 to 0.1 seconds

ST segment

 Isoelectric period following the QRS


 Represents period which the entire ventricle is depolarized and roughly
corresponds to the plateau phase of the ventricular action potential

T wave

 Represents ventricular repolarization and is longer in duration than


depolarization
 A small positive U wave may follow the T wave which represents the last
remnants of ventricular repolarization.

Q-T interval

 Represents the time for both ventricular depolarization and repolarization to


occur, and therefore roughly estimates the duration of an average ventricular
action potential.
 Interval ranges from 0.2 to 0.4 seconds depending upon heart rate.
 At high heart rates, ventricular action potentials shorten in duration, which
decreases the Q-T interval. Therefore the Q-T interval is expressed as a
"corrected Q-T (QTc)" by taking the Q-T interval and dividing it by the square
root of the R-R interval (interval between ventricular depolarizations). This
allows an assessment of the Q-T interval that is independent of heart rate.
 Normal corrected Q-Tc interval is less than 0.44 seconds.

The oxygen-haemoglobin dissociation curve is shifted to the right in which of the


following scenarios?

A. Hypothermia
B. Respiratory alkalosis

C. Low altitude

D. Decreased 2,3-DPG in transfused red cells

E. Chronic iron deficiency anaemia


Mnemonic to remember causes of right
shift of the oxygen dissociation curve:

CADET face RIGHT

C O2
A cidosis
2,3-DPG
E xercise
T emperature

The curve is shifted to the right when there is an increased oxygen requirement by the
tissue. This includes:

 Increased temperature
 Acidosis
 Increased DPG:

DPG is found in erythrocytes and is increased during glycolysis. It binds to the Hb


molecule, thereby releasing oxygen to tissues. DPG is increased in conditions
associated with poor oxygen delivery to tissues, such as anaemia and high altitude.

Oxygen Transport

Oxygen transport
Almost all oxygen is transported within erythrocytes. It has limited solubility and only
1% is carried as solution. Therefore the amount of oxygen transported will depend
upon haemoglobin concentration and its degree of saturation.

Haemoglobin
Globular protein composed of 4 subunits. Haem consists of a protoporphyrin ring
surrounding an iron atom in its ferrous state. The iron can form two additional bonds;
one with oxygen and the other with a polypeptide chain. There are two alpha and two
beta subunits to this polypeptide chain in an adult and together these form globin.
Globin cannot bind oxygen but is able to bind to carbon dioxide and hydrogen ions,
the beta chains are able to bind to 2,3 diphosphoglycerate. The oxygenation of
haemoglobin is a reversible reaction. The molecular shape of haemoglobin is such
that binding of one oxygen molecule facilitates the binding of subsequent molecules.
Oxygen dissociation curve

 The oxygen dissociation curve describes the relationship between the


percentage of saturated haemoglobin and partial pressure of oxygen in the
blood. It is not affected by haemoglobin concentration.
 Chronic anaemia causes 2, 3 DPG levels to increase, hence shifting the curve
to the right

Bohr effect

 Shifts to left = for given oxygen tension there is increased saturation of Hb


with oxygen i.e. Decreased oxygen delivery to tissues
 Shifts to right = for given oxygen tension there is reduced saturation of Hb
with oxygen i.e. Enhanced oxygen delivery to tissues

Image sourced from Wikipedia

Shifts to Left = Lower oxygen delivery Shifts to Right = Raised oxygen


delivery
 HbF, methaemoglobin,
carboxyhaemoglobin  raised [H+] (acidic)
 low [H+] (alkali)  raised pCO2
 low pCO2  raised 2,3-DPG*
 low 2,3-DPG  raised temperature
 low temperature

*2,3-diphosphoglycerate
A 73 year old lady is admitted for a laparoscopic cholecystectomy. During her pre-
operative assessment it is noted that she is receiving furosemide for the treatment of
hypertension. Where is the site of action of this diuretic?

A. Proximal convoluted tubule

B. Descending limb of the loop of Henle

C. Ascending limb of the loop of Henle

D. Distal convoluted tubule

E. Collecting ducts
Action of furosemide = ascending
limb of the loop of Henle

Furosemide and bumetanide are loop diuretics that act by inhibiting the Na-K-Cl
cotransporter in the thick ascending limb of the loop of Henle, reducing the absorption
of NaCl.

Diuretic agents

The diuretic drugs are divided into three major classes, which are distinguished
according to the site at which they impair sodium reabsorption: loop diuretics in the
thick ascending loop of Henle, thiazide type diuretics in the distal tubule and
connecting segment; and potassium sparing diuretics in the aldosterone - sensitive
principal cells in the cortical collecting tubule.
In the kidney, sodium is reabsorbed through Na+/ K+ ATPase pumps located on the
basolateral membrane. These pumps return reabsorbed sodium to the circulation and
maintain low intracellular sodium levels. This latter effect ensures a constant
concentration gradient.

Physiological effects of commonly used diuretics


Site of action Diuretic Carrier or Percentage of filtered
channel inhibited sodium excreted
Ascending limb of loop Frusemide Na+/K+ 2Cl - Upt to 25%
of Henle carrier
Distal tubule and Thiazides Na+Cl- carrier Between 3 and 5%
connecting segment
Cortical collecting Spironolactone Na+ channel Between 1 and 2%
tubule
A 45 year old man is referred to the breast clinic with gynaecomastia. He takes the
drugs listed below. Which is least likely to be the cause of his symptoms?

A. Spironolactone

B. Carbimazole

C. Chlorpromazine

D. Cimetidine

E. Methyldopa
Mnemonic for drugs causing
gynaecomastia: DISCO

D igitalis
I soniazid
S pironolactone
C imentidine
O estrogen

Mnemonic for causes of


gynaecomastia: METOCLOPRAMIDE

M etoclopramide
E ctopic oestrogen
T rauma skull/tumour breast, testes
O rchitis
C imetidine, Cushings
L iver cirrhosis
O besity
P araplegia
RA
A cromegaly
M ethyldopa
I soniazid
D igoxin
E thionamide

Carbimazole is not associated with gynaecomastia.

Gynaecomastia

Gynaecomastia describes an abnormal amount of breast tissue in males and is usually


caused by an increased oestrogen:androgen ratio. It is important to differentiate the
causes of galactorrhoea (due to the actions of prolactin on breast tissue) from those of
gynaecomastia
Causes of gynaecomastia

 physiological: normal in puberty


 syndromes with androgen deficiency: Kallman's, Klinefelter's
 testicular failure: e.g. Mumps
 liver disease
 testicular cancer e.g. Seminoma secreting hCG
 ectopic tumour secretion
 hyperthyroidism
 haemodialysis
 drugs: see below

Drug causes of gynaecomastia

 spironolactone (most common drug cause)


 cimetidine
 digoxin
 cannabis
 finasteride
 oestrogens, anabolic steroids

Very rare drug causes of gynaecomastia

 tricyclics
 isoniazid
 calcium channel blockers
 heroin
 busulfan
 methyldopa

43 year old lady is recovering on the intensive care unit following a Whipples
procedure. She has a central venous line in situ. Which of the following will lead to
the "y" descent on the waveform trace?

A. Ventricular contraction

B. Emptying of the right atrium

C. Emptying of the right ventricle

D. Opening of the pulmonary valve

E. Cardiac tamponade
JVP
3 Upward deflections and 2 downward
deflections

Upward deflections

 a wave = atrial contraction


 c wave = ventricular
contraction
 v wave = atrial venous filling

Downward deflections

 x wave = atrium relaxes and


tricuspid valve moves down
 y wave = ventricular filling

Theme from January 2012

The 'y' descent represents the emptying of the atrium and the filling of the right
ventricle.

Cardiac physiology

 The heart has four chambers ejecting blood into both low pressure and high
pressure systems.
 The pumps generate pressures of between 0-25mmHg on the right side and 0-
120 mmHg on the left.
 At rest diastole comprises 2/3 of the cardiac cycle.
 The product of the frequency of heart rate and stroke volume combine to give
the cardiac output which is typically 5-6L per minute.

Detailed descriptions of the various waveforms are often not a feature of MRCS A
(although they are on the syllabus). However, they are a very popular topic for
surgical physiology vivas in the oral examination.

Electrical properties

 Intrinsic myogenic rhythm within cardiac myocytes means that even the
denervated heart is capable of contraction.
 In the normal situation the cardiac impulse is generated in the sino atrial node
in the right atrium and conveyed to the ventricles via the atrioventricular node.
 The sino atrial node is also capable of spontaneous discharge and in the
absence of background vagal tone will typically discharge around 100x per
minute. Hence the higher resting heart rate found in cardiac transplant cases.
In the SA and AV nodes the resting membrane potential is lower than in
surrounding cardiac cells and will slowly depolarise from -70mV to around -
50mV at which point an action potential is generated.
 Differences in the depolarisation slopes between SA and AV nodes help to
explain why the SA node will depolarise first. The cells have a refractory
period during which they cannot be re-stimulated and this period allows for
adequate ventricular filling. In pathological tachycardic states this time period
is overridden and inadequate ventricular filling may then occur, cardiac output
falls and syncope may ensue.

Parasympathetic fibres project to the heart via the vagus and will release
acetylcholine. Sympathetic fibres release nor adrenaline and circulating adrenaline
comes from the adrenal medulla. Noradrenaline binds to β 1 receptors in the SA node
and increases the rate of pacemaker potential depolarisation.

Cardiac cycle

Image sourced from Wikipedia

 Mid diastole: AV valves open. Ventricles hold 80% of final volume. Outflow
valves shut. Aortic pressure is high.

 Late diastole: Atria contract. Ventricles receive 20% to complete filling.


Typical end diastolic volume 130-160ml.

 Early systole: AV valves shut. Ventricular pressure rises. Isovolumetric


ventricular contraction. AV Valves bulge into atria (c-wave). Aortic and
pulmonary pressure exceeded- blood is ejected. Shortening of ventricles pulls
atria downwards and drops intra atrial pressure (x-descent).
 Late systole: Ventricular muscles relax and ventricular pressures drop.
Although ventricular pressure drops the aortic pressure remains constant
owing to peripheral vascular resistance and elastic property of the aorta. Brief
period of retrograde flow that occurs in aortic recoil shuts the aortic valve.
Ventricles will contain 60ml end systolic volume. The average stroke volume
is 70ml (i.e. Volume ejected).

 Early diastole: All valves are closed. Isovolumetric ventricular relaxation


occurs. Pressure wave associated with closure of the aortic valve increases
aortic pressure. The pressure dip before this rise can be seen on arterial
waveforms and is called the incisura. During systole the atrial pressure
increases such that it is now above zero (v- wave). Eventually atrial pressure
exceed ventricular pressure and AV valves open - atria empty passively into
ventricles and atrial pressure falls (y -descent )

The negative atrial pressures are of clinical importance as they can allow air
embolization to occur if the neck veins are exposed to air. This patient positioning is
important in head and neck surgery to avoid this occurrence if veins are inadvertently
cut, or during CVP line insertion.

Mechanical properties

 Preload = end diastolic volume


 Afterload = aortic pressure

It is important to understand the principles of Laplace's law in surgery.

 It states that for hollow organs with a circular cross section, the total
circumferential wall tension depends upon the circumference of the wall,
multiplied by the thickness of the wall and on the wall tension.
 The total luminal pressure depends upon the cross sectional area of the lumen
and the transmural pressure. Transmural pressure is the internal pressure
minus external pressure and at equilibrium the total pressure must
counterbalance each other.
 In terms of cardiac physiology the law explains that the rise in ventricular
pressure that occurs during the ejection phase is due to physical change in
heart size. It also explains why a dilated diseased heart will have impaired
systolic function.

Starlings law

 Increase in end diastolic volume will produce larger stroke volume.


 This occurs up to a point beyond which cardiac fibres are excessively
stretched and stroke volume will fall once more. It is important for the
regulation of cardiac output in cardiac transplant patients who need to increase
their cardiac output.

Baroreceptor reflexes

 Baroreceptors located in aortic arch and carotid sinus.


 Aortic baroreceptor impulses travel via the vagus and from the carotid via the
glossopharyngeal nerve.
 They are stimulated by arterial stretch.
 Even at normal blood pressures they are tonically active.
 Increase in baroreceptor discharge causes:

*Increased parasympathetic discharge to the SA node.


*Decreased sympathetic discharge to ventricular muscle causing decreased
contractility and fall in stroke volume.
*Decreased sympathetic discharge to venous system causing increased compliance.
*Decreased peripheral arterial vascular resistance

Atrial stretch receptors

 Located in atria at junction between pulmonary veins and vena cava.


 Stimulated by atrial stretch and are thus low pressure sensors.
 Increased blood volume will cause increased parasympathetic activity.
 Very rapid infusion of blood will result in increase in heart rate mediated via
atrial receptors: the Bainbridge reflex.
 Decreases in receptor stimulation results in increased sympathetic activity this
will decrease renal blood flow-decreases GFR-decreases urinary sodium
excretion-renin secretion by juxtaglomerular apparatus-Increase in angiotensin
II.
 Increased atrial stretch will also result in increased release of atrial natriuretic
peptide.

Which of the following are not characteristic features of central chemoreceptors in the
control of ventilation?

A. They are located in the medulla oblongata

B. They are stimulated primarily by venous hypercapnia

C. They are relatively insensitive to hypoxia

D. They are less sensitive to changes in arterial pH than other


ventillatory receptors
E. During acute hypercapnia the carotid receptors will be stimulated
first

They are stimulated by arterial carbon dioxide. It takes longer to equilibrate than the
peripheral chemoreceptors located in the carotid. They are less sensitive to acidity due
to the blood brain barrier.

Control of ventilation

 Control of ventilation is coordinated by the respiratory centres,


chemoreceptors, lung receptors and muscles.
 Automatic, involuntary control of respiration occurs from the medulla.
 The respiratory centres control the respiratory rate and the depth of respiration.

Respiratory centres

 Medullary respiratory centre:

Inspiratory and expiratory neurones. Has ventral group which controls forced
voluntary expiration and the dorsal group controls inspiration. Depressed by opiates.

 Apneustic centre:

Lower pons
Stimulates inspiration - activates and prolongs inhalation
Overridden by pneumotaxic control to end inspiration

 Pneumotaxic centre:

Upper pons, inhibits inspiration at a certain point. Fine tunes the respiratory rate.

 Levels of PCO2 most important in ventilation control


 Levels of O2 are less important.
 Peripheral chemoreceptors: located in the bifurcation of carotid arteries and
arch of the aorta. They respond to changes in reduced pO2, increased H+ and
increased pCO2 in ARTERIAL BLOOD.
 Central chemoreceptors: located in the medulla. Respond to increased H+ in
BRAIN INTERSTITIAL FLUID to increase ventilation. NB the central
receptors are NOT influenced by O2 levels.

Lung receptors include:


 Stretch receptors: respond to lung stretching causing a reduced respiratory rate
 Irritant receptors: respond to smoke etc causing bronchospasm
 J (juxtacapillary) receptors

A 32 year old man has a glomerular filtration rate of 110ml / minute at a systolic
blood pressure of 120/80. If his blood pressure were to fall to 100/70 what would
glomerular filtration rate be?

A. 110ml / minute

B. 100ml/ minute

C. 55ml/ minute

D. 25ml/ minute

E. 75ml/ minute

The proposed drop in blood pressure falls within the range within which the kidney
autoregulates its blood supply. GFR will therefore remain unchanged.

Renal Physiology

Overview

 Each nephron is supplied with blood from an afferent arteriole that opens onto
the glomerular capillary bed.
 Blood then flows to an efferent arteriole, supplying the peritubular capillaries
and medullary vasa recta.
 The kidney receives up to 25% of resting cardiac output.

Control of blood flow

 The kidney is able to autoregulate its blood flow between systolic pressures of
80- 180mmHg so there is little variation in renal blood flow.
 This is achieved by myogenic control of arteriolar tone, both sympathetic
input and hormonal signals (e.g. renin) are responsible.

Glomerular structure and function

 Blood inside the glomerulus has considerable hydrostatic pressure.


 The basement membrane has pores that will allow free diffusion of smaller
solutes, larger negatively charged molecules such as albumin are unable to
cross.
 The glomerular filtration rate (GFR) is equal to the concentration of a solute in
the urine, times the volume of urine produced per minute, divided by the
plasma concentration (assuming that the solute is freely diffused e.g. inulin).
 In clinical practice creatinine is used because it is subjected to very little
proximal tubular secretion.
 Although subject to variability, the typical GFR is 125ml per minute.

 Glomerular filtration rate = Total volume of plasma per unit time leaving the
capillaries and entering the bowman's capsule

 Renal clearance = volume plasma from which a substance is removed per


minute by the kidneys

 Substances used to measure GFR have the following features:

1. Inert
2. Free filtration from the plasma at the glomerulus (not protein bound)
3. Not absorbed or secreted at the tubules
4. Plasma concentration constant during urine collection

Examples: inulin, creatinine

GFR = urine concentration (mmol/l) x urine volume (ml/min)


--------------------------------------------------------------------------
plasma concentration (mmol/l)

 The clearance of a substance is dependent not only on its diffusivity across the
basement membrane but also subsequent tubular secretion and / or
reabsorption.
 So glucose which is freely filtered across the basement membrane is usually
reabsorbed from tubules giving a clearance of zero.

Tubular function

 Reabsorption and secretion of substances occurs in the tubules.


 In the proximal tubule substrates such as glucose, amino acids and phosphate
are co-transported with sodium across the semi permeable membrane.
 Up to two thirds of filtered water is reabsorbed in the proximal tubules.
 This will lead to increase in urea concentration in the distal tubule allowing for
its increased diffusion.
 Substances to be secreted into the tubules are taken up from the peritubular
blood by tubular cells.
 Solutes such as paraaminohippuric acid are cleared with a single passage
through the kidneys and this is why it is used to measure renal plasma flow.
Ions such as calcium and phosphate will have a tubular reabsorption that is
influenced by plasma PTH levels.
 Potassium may be both secreted and re-absorbed and is co-exchanged with
sodium.

Loop of Henle

 Approximately 60 litres of water containing 9000mmol sodium enters the


descending limb of the loop of Henle in 24 hours.
 Loops from the juxtamedullary nephrons run deep into the medulla.
 The osmolarity of fluid changes and is greatest at the tip of the papilla.
 The thin ascending limb is impermeable to water, but highly permeable to
sodium and chloride ions.
 This loss means that at the beginning of the thick ascending limb the fluid is
hypo osmotic compared with adjacent interstitial fluid.
 In the thick ascending limb the reabsorption of sodium and chloride ions
occurs by both facilitated and passive diffusion pathways.
 The loops of Henle are co-located with vasa recta, these will have similar
solute compositions to the surrounding extracellular fluid so preventing the
diffusion and subsequent removal of this hypertonic fluid.
 The energy dependent reabsorption of sodium and chloride in the thick
ascending limb helps to maintain this osmotic gradient.

Which of the following does not stimulate insulin release?

A. Gastrin

B. Atenolol

C. Protein

D. Secretin

E. Vagal cholinergic action

Beta blockers inhibit the release of insulin.

Stimulation of insulin release:

 Glucose
 Amino acid
 Vagal cholinergic
 Secretin/Gastrin/CCK
 Fatty acids
 Beta adrenergic drugs

Insulin

 Anabolic hormone

Structure

 and chain linked by disulphide bridges

Synthesis

 Pro-insulin is formed by the rough endoplasmic reticulum in pancreatic beta


cells. Then pro-insulin is cleaved to form insulin and C-peptide. Insulin is
stored in secretory granules and released in response to Ca.

Function

 Secreted in response to hyperglycaemia


 Glucose utilisation and glycogen synthesis
 Inhibits lipolysis
 Reduces muscle protein loss

A 63 year old female is referred to the surgical clinic with an iron deficiency anaemia.
Her past medical history includes a left hemi colectomy but no other co-morbidities.
At what site is most dietery iron absorbed?

A. Stomach

B. Duodenum

C. Proximal ileum

D. Distal ileum

E. Colon

Iron is best absorbed from the proximal small bowel (duodenum and jejunum) in the
Fe 2+ state. Iron is transported across the small bowel mucosa by a divalent membrane
transporter protein (hence the improved absorption of F2 2+. The intestinal cells
typically store the bound iron as ferritin. Cells requiring iron will typically then
absorb the complex as needed.

Iron metabolism

Absorption  Duodenum and upper jejunum


 About 10% of dietary iron absorbed
 Fe2+ (ferrous iron) much better absorbed than Fe3+ (ferric iron)
 Ferrous iron is oxidized to form ferric iron, which is combined
with apoferritin to form ferritin
 Absorption is regulated according to body's need
 Increased by vitamin C, gastric acid
 Decreased by proton pump inhibitors, tetracycline, gastric
achlorhydria, tannin (found in tea)

Transport In plasma as Fe3+ bound to transferrin


Storage Ferritin (or haemosiderin) in bone marrow
Excretion Lost via intestinal tract following desquamation

Distribution in body
Total body iron 4g
Haemoglobin 70%
Ferritin and haemosiderin 25%
4%
Myoglobin
Plasma iron 0.1%
Which of the following drugs increases the rate of gastric emptying in the
vagotomised stomach?

A. Ondansetron

B. Metoclopramide

C. Cyclizine

D. Erythromycin

E. Chloramphenicol
Vagotomy seriously compromises gastric emptying which is why either a
pyloroplasty or gastro-enterostomy is routinely performed at the same time.

Chloramphenicol has no effect on gastric emptying. Ondansetron slows gastric


emptying slightly. Metoclopramide increases the rate of gastric emptying but its
effects are mediated via the vagus nerve.
Gastric emptying

 The stomach serves both a mechanical and immunological function. Solid and
liquid are retained in the stomach during which time repeated peristaltic
activity against a closed pyloric sphincter will cause fragmentation of food
bolus material. Contact with gastric acid will help to neutralise any pathogens
present.

 The amount of time material spends in the stomach is related to its


composition and volume. For example a glass of water will empty more
quickly than a large meal. The presence of amino acids and fat will all serve to
delay gastric emptying.

Controlling factors
Neuronal stimulation of the stomach is mediated via the vagus and the
parasympathetic nervous system will tend to favor an increase in gastric motility. It is
for this reason that individuals who have undergone truncal vagotomy will tend to
routinely require either a pyloroplasty or gastro-enterostomy as they would otherwise
have delayed gastric emptying.

The following hormonal factors are all involved:

Delay emptying Increase emptying


Gastric inhibitory peptide Gastrin
Cholecystokinin
Enteroglucagon

Diseases affecting gastric emptying


All diseases that affect gastric emptying may result in bacterial overgrowth, retained
food and eventually the formation of bezoars that may occlude the pylorus and make
gastric emptying even worse. Fermentation of food may cause dyspepsia, reflux and
foul smelling belches of gas.

Iatrogenic
Gastric surgery can have profound effects on gastric emptying. As stated above any
procedure that disrupts the vagus can cause delayed emptying. Whilst this is
particularly true of Vagotomy this operation is now rarely performed. Surgeons are
divided on the importance of vagal disruption that occurs during an oesophagectomy
and some will routinely perform a pyloroplasty and other will not.

When a distal gastrectomy is performed the type of anastomosis performed will


impact on emptying. When a gastro-enterostomy is constructed, a posterior, retrocolic
gastroenterostomy will empty better than an anterior one.
Diabetic gastroparesis
This is predominantly due to neuropathy affecting the vagus nerve. The stomach
empties poorly and patients may have episodes of repeated and protracted vomiting.
Diagnosis is made by upper GI endoscopy and contrast studies, in some cases a radio
nucleotide scan is needed to demonstrate the abnormality more clearly. In treating
these conditions drugs such as metoclopramide will be less effective as they exert
their effect via the vagus nerve. One of the few prokinetic drugs that do not work in
this way is the antibiotic erythromycin.

Malignancies
Obviously a distal gastric cancer may obstruct the pylorus and delay emptying. In
addition malignancies of the pancreas may cause extrinsic compression of the
duodenum and delay emptying. Treatment in these cases is by gastric decompression
using a wide bore nasogastric tube and insertion of a stent or if that is not possible by
a surgical gastroenterostomy. As a general rule gastroenterostomies constructed for
bypass of malignancy are usually placed on the anterior wall of the stomach (in spite
of the fact that they empty less well). A Roux en Y bypass may also be undertaken but
the increased number of anastomoses for this in malignant disease that is being
palliated is probably not justified.

Congenital Hypertrophic Pyloric Stenosis


This is typically a disease of infancy. Most babies will present around 6 weeks of age
with projectile non bile stained vomiting. It has an incidence of 2.4 per 1000 live
births and is more common in males. Diagnosis is usually made by careful history and
examination and a mass may be palpable in the epigastrium (often cited seldom felt!).
The most important diagnostic test is an ultrasound that usually demonstrates the
hypertrophied pylorus. Blood tests may reveal a hypochloraemic metabolic alkalosis
if the vomiting is long standing. Once the diagnosis is made the infant is resuscitated
and a pyloromyotomy is performed (usually laparoscopically). Once treated there are
no long term sequelae.
Which of the following haemodynamic changes is not seen in hypovolaemic shock?

A. Decreased cardiac output

B. Increased heart rate

C. Reduced left ventricle filling pressures

D. Reduced blood pressure

E. Reduced systemic vascular resistance


Cardiogenic Shock:
e.g. MI, valve abnormality

increased SVR (vasoconstriction in response to low BP)


increased HR (sympathetic response)
decreased cardiac output
decreased blood pressure

Hypovolaemic shock:
blood volume depletion
e.g. haemorrhage, vomiting, diarrhoea, dehydration, third-space losses during major
operations

increased SVR
increased HR
decreased cardiac output
decreased blood pressure

Septic shock:
occurs when the peripheral vascular dilatation causes a fall in SVR
similar response may occur in anaphylactic shock, neurogenic shock

reduced SVR
increased HR
normal/increased cardiac output
decreased blood pressure

SVR will typically increase

Shock

 Shock occurs when there is insufficient tissue perfusion.


 The pathophysiology of shock is an important surgical topic and may be
divided into the following aetiological groups:
 Septic
 Haemorrhagic
 Neurogenic
 Cardiogenic
 Anaphylactic

Septic shock
Septic shock is a major problem and those patients with severe sepsis have a mortality
rate in excess of 40%. In those who are admitted to intensive care mortality ranges
from 6% with no organ failure to 65% in those with 4 organ failure.

Sepsis is defined as an infection that triggers a particular Systemic Inflammatory


Response Syndrome (SIRS). This is characterised by body temperature outside 36
o
C - 38 o C, HR >90 beats/min, respiratory rate >20/min, WBC count >12,000/mm3 or
< 4,000/mm3.

Patients with infections and two or more elements of SIRS meet the diagnostic criteria
for sepsis. Those with organ failure have severe sepsis and those with refractory
hypotension -septic shock.
During the septic process there is marked activation of the immune system with
extensive cytokine release. This may be coupled with or triggered by systemic
circulation of bacterial toxins. These all cause endothelial cell damage and neutrophil
adhesion. The overall hallmarks are thus those of excessive inflammation,
coagulation and fibrinolytic suppression.

The surviving sepsis campaign highlights the following key areas for attention:

 Prompt administration of antibiotics to cover all likely pathogens coupled with


a rigorous search for the source of infection.
 Haemodynamic stabilisation. Many patients are hypovolaemic and require
aggressive fluid administration. Aim for CVP 8-12 cm H2O, MAP >65mmHg.
 Modulation of the septic response. This includes manoeuvres to counteract the
changes and includes measures such as tight glycaemic control, use of
activated protein C and sometimes intravenous steroids.

In surgical patients, the main groups with septic shock include those with anastomotic
leaks, abscesses and extensive superficial infections such as necrotising fasciitis.
When performing surgery the aim should be to undertake the minimum necessary to
restore physiology. These patients do not fare well with prolonged surgery. Definitive
surgery can be more safely undertaken when physiology is restored and clotting in
particular has been normalised.

Haemorrhagic shock
The average adult blood volume comprises 7% of body weight. Thus in the 70 Kg
adult this will equate to 5 litres. This changes in children (8-9% body weight) and is
slightly lower in the elderly.

The table below outlines the 4 major classes of haemorrhagic shock and their
associated physiological sequelae:

Parameter Class I Class II Class III Class IV


Blood loss ml <750ml 750-1500ml 1500-2000ml >2000ml
Blood loss % <15% 15-30% 30-40% >40%
Pulse rate <100 >100 >120 >140ml
Blood pressure Normal Decreased Decreased Decreased
Respiratory rate 14-20 20-30 30-40 >35
Urine output >30ml 20-30ml 5-15ml <5ml
Symptoms Normal Anxious Confused Lethargic

Decreasing blood pressure during haemorrhagic shock causes organ hypoperfusion


and relative myocardial ishaemia. The cardiac index gives a numerical value for tissue
oxygen delivery and is given by the equation: Cardiac index= 13.4 - [Hb] - SaO2 +
0.03 PaO2. Where Hb is haemoglobin concentration in blood and SaO2 the saturation
and PaO2 the partial pressure of oxygen. Detailed knowledge of this equation is
required for the MRCS Viva but not for part A, although you should understand the
principle.

In patients suffering from trauma the most likely cause of shock is haemorrhage.
However, the following may also be the cause or occur concomitantly:

 Tension pneumothorax
 Spinal cord injury
 Myocardial contusion
 Cardiac tamponade

When assessing trauma patients it is worth remembering that in order to generate a


palpable femoral pulse an arterial pressure of >65mmHg is required.

Once bleeding is controlled and circulating volume normalised the levels of


transfusion should be to maintain a Hb of 7-8 in those with no risk factors for tissue
hypoxia and Hb 10 for those who have such risk factors.

Neurogenic shock
This occurs most often following a spinal cord transection, usually at a high level.
There is resultant interruption of the autonomic nervous system. The result is either
decreased sympathetic tone or increased parasympathetic tone, the effect of
which is a decrease in peripheral vascular resistance mediated by marked
vasodilation.

This results in decreased preload and thus decreased cardiac output (Starlings law).
There is decreased peripheral tissue perfusion and shock is thus produced. In contrast
with many other types of shock peripheral vasoconstrictors are used to return vascular
tone to normal.

Cardiogenic shock
In medical patients the main cause is ischaemic heart disease. In the traumatic
setting direct myocardial trauma or contusion is more likely. Evidence of ECG
changes and overlying sternal fractures or contusions should raise the suspicion of
injury. Treatment is largely supportive and transthoracic echocardiography should be
used to determine evidence of pericardial fluid or direct myocardial injury. The
measurement of troponin levels in trauma patients may be undertaken but they are
less useful in delineating the extent of myocardial trauma than following MI.

When cardiac injury is of a blunt nature and is associated with cardiogenic shock the
right side of the heart is the most likely site of injury with chamber and or valve
rupture. These patients require surgery to repair these defects and will require
cardiopulmonary bypass to achieve this. Some may require intra aortic balloon pump
as a bridge to surgery.

Anaphylactic shock
Anaphylaxis may be defined as a severe, life-threatening, generalised or systemic
hypersensitivity reaction.

Anaphylaxis is one of the few times when you would not have time to look up the
dose of a medication. The Resuscitation Council guidelines on anaphylaxis have
recently been updated. Adrenaline is by far the most important drug in anaphylaxis
and should be given as soon as possible. The recommended doses for adrenaline,
hydrocortisone and chlorphenamine are as follows:

Adrenaline Hydrocortisone Chlorphenamine


< 6 months 150 mcg (0.15ml 1 in 25 mg 250 mcg/kg
1,000)
6 months - 6 years 150 mcg (0.15ml 1 in 50 mg 2.5 mg
1,000)
6-12 years 300 mcg (0.3ml 1 in 100 mg 5 mg
1,000)
Adult and child 12 500 mcg (0.5ml 1 in 200 mg 10 mg
years 1,000)

Adrenaline can be repeated every 5 minutes if necessary. The best site for IM
injection is the anterolateral aspect of the middle third of the thigh.

Common identified causes of anaphylaxis

 food (e.g. Nuts) - the most common cause in children


 drugs
 venom (e.g. Wasp sting)

A 25 year old man is undergoing respiratory spirometry. He takes a maximal


inspiration and maximally exhales. Which of the following measurements will best
illustrate this process?

A. Functional residual capacity

B. Vital capacity

C. Inspiratory capacity

D. Maximum voluntary ventilation

E. Tidal volume

Theme from April 2012 Exam


The maximum voluntary ventilation is the maximal ventilation over the course of 1
minute.

Lung volumes

The diagram demonstrates lung volumes and capacities


Image sourced from Wikipedia

Definitions

Tidal volume (TV)  Is the volume of air inspired and expired during each
ventilatory cycle at rest.
 It is normally 500mls in males and 340mls in
females.

Inspiratory reserve  Is the maximum volume of air that can be forcibly


volume (IRV) inhaled following a normal inspiration. 3000mls.

Expiratory reserve  Is the maximum volume of air that can be forcibly


volume (ERV) exhaled following a normal expiration. 1000mls.

Residual volume (RV)  Is that volume of air remaining in the lungs after a
maximal expiration.
 RV = FRC - ERV. 1500mls.

Functional residual  Is the volume of air remaining in the lungs at the end
capacity (FRC) of a normal expiration.
 FRC = RV + ERV. 2500mls.

Vital capacity (VC)  Is the maximal volume of air that can be forcibly
exhaled after a maximal inspiration.
 VC = TV + IRV + ERV. 4500mls in males, 3500mls
in females.

Total lung capacity  Is the volume of air in the lungs at the end of a
(TLC) maximal inspiration.
 TLC = FRC + TV + IRV = VC + RV. 5500-6000mls.

Forced vital capacity  The volume of air that can be maximally forcefully
(FVC)
exhaled.

Which of the following does not decrease the functional residual capacity?

A. Obesity

B. Pulmonary fibrosis

C. Muscle relaxants

D. Laparoscopic surgery

E. Upright position
Increased FRC:

 Erect position
 Emphysema
 Asthma

Decreased FRC:

 Pulmonary fibrosis
 Laparoscopic surgery
 Obesity
 Abdominal swelling
 Muscle relaxants

When the patient is upright the diaphragm and abdominal organs put less pressure on
the lung bases, allowing for an increase in the functional residual capacity (FRC).
Other causes of increased FRC include:

 Emphysema
 Asthma

In addition to those listed above, causes of reduced FRC include:

 Abdominal swelling
 Pulmonary oedema
 Reduced muscle tone of the diaphragm
 Age
Lung volumes

The diagram demonstrates lung volumes and capacities

Image sourced from Wikipedia

Definitions

Tidal volume (TV)  Is the volume of air inspired and expired during each
ventilatory cycle at rest.
 It is normally 500mls in males and 340mls in
females.

Inspiratory reserve  Is the maximum volume of air that can be forcibly


volume (IRV) inhaled following a normal inspiration. 3000mls.

Expiratory reserve  Is the maximum volume of air that can be forcibly


volume (ERV) exhaled following a normal expiration. 1000mls.

Residual volume (RV)  Is that volume of air remaining in the lungs after a
maximal expiration.
 RV = FRC - ERV. 1500mls.

Functional residual  Is the volume of air remaining in the lungs at the end
capacity (FRC) of a normal expiration.
 FRC = RV + ERV. 2500mls.

Vital capacity (VC)  Is the maximal volume of air that can be forcibly
exhaled after a maximal inspiration.
 VC = TV + IRV + ERV. 4500mls in males, 3500mls
in females.

Total lung capacity  Is the volume of air in the lungs at the end of a
(TLC) maximal inspiration.
 TLC = FRC + TV + IRV = VC + RV. 5500-6000mls.

Forced vital capacity  The volume of air that can be maximally forcefully
(FVC) exhaled.

Which of the following is the main site of dehydroepiandrosterone release?

A. Posterior pituitary

B. Zona reticularis of the adrenal gland

C. Zona glomerulosa of the adrenal gland

D. Juxtaglomerular apparatus of the kidney

E. Zona fasciculata of the adrenal gland


Adrenal cortex mnemonic:
GFR - ACD

DHEA possesses some androgenic activity and is almost exclusively released from
the adrenal gland.

Renin-angiotensin-aldosterone system

Adrenal cortex (mnemonic GFR - ACD)

 Zona glomerulosa (on outside): mineralocorticoids, mainly aldosterone


 Zona fasciculata (middle): glucocorticoids, mainly cortisol
 Zona reticularis (on inside): androgens, mainly dehydroepiandrosterone
(DHEA)

Renin

 Released by JGA cells in kidney in response to reduced renal perfusion, low


sodium
 Hydrolyses angiotensinogen to form angiotensin I

Factors stimulating renin secretion

 Low BP
 Hyponatraemia
 Sympathetic nerve stimulation
 Catecholamines
 Erect posture

Angiotensin

 ACE in lung converts angiotensin I --> angiotensin II


 Vasoconstriction leads to raised BP
 Stimulates thirst
 Stimulates aldosterone and ADH release

Aldosterone

 Released by the zona glomerulosa in response to raised angiotensin II,


potassium, and ACTH levels
 Causes retention of Na+ in exchange for K+/H+ in distal tubule
 Secretions from which of the following will contain the highest levels of
potassium?

A. Rectum

B. Small bowel

C. Gallbladder

D. Pancreas

E. Stomach

The rectum has the potential to generate secretions rich in potassium. This is
the rationale behind administration of resins for hyperkalaemia and the
development of hypokalaemia in patients with villous adenoma of the rectum.
 Potassium secretion -GI tract

Potassium secretions
Salivary glands Variable may be up to 60mmol/L
Stomach 10 mmol/L
Bile 5 mmol/L
Pancreas 4-5 mmol/L
Small bowel 10 mmol/L
Rectum 30 mmol/L

The above table provides average figures only and the exact composition
varies depending upon the existence of disease, serum aldosterone levels and
serum pH.
A key point to remember for the exam is that gastric potassium secretions are
low. Hypokalaemia may occur in vomiting, usually as a result of renal wasting
of potassium, not because of potassium loss in vomit.
What is the typical stroke volume in a resting 70 Kg man?

A. 10ml

B. 150ml

C. 125ml

D. 45ml

E. 70ml

Theme from April 2012 Exam


Stroke volumes range from 55-100ml.

Stroke volume-Cardiac physiology

The stroke volume equates to the volume of blood ejected from the ventricle during
each cycle of cardiac contraction. The volumes for both ventricles are typically equal
and equate roughly to 70ml for a 70Kg man. It is calculated by subtracting the end
systolic volume from the end diastolic volume.

Factors affecting stroke volume

 Cardiac size
 Contractility
 Preload
 Afterload

A patient loses 1.6L fresh blood from their abdominal drain. Which of the following
will not decrease?

A. Cardiac output

B. Renin secretion

C. Firing of carotid baroreceptors

D. Firing of aortic baroreceptors

E. Blood pressure
Renin secretion will increase as systemic hypotension will cause impairment of renal
blood flow. Although the kidney can autoregulate its own blood flow over a range of
systemic blood pressures a loss of 1.6 L will usually produce an increase in renin
secretion.

Shock

 Shock occurs when there is insufficient tissue perfusion.


 The pathophysiology of shock is an important surgical topic and may be
divided into the following aetiological groups:
 Septic
 Haemorrhagic
 Neurogenic
 Cardiogenic
 Anaphylactic

Septic shock
Septic shock is a major problem and those patients with severe sepsis have a mortality
rate in excess of 40%. In those who are admitted to intensive care mortality ranges
from 6% with no organ failure to 65% in those with 4 organ failure.

Sepsis is defined as an infection that triggers a particular Systemic Inflammatory


Response Syndrome (SIRS). This is characterised by body temperature outside 36
o
C - 38 o C, HR >90 beats/min, respiratory rate >20/min, WBC count >12,000/mm3 or
< 4,000/mm3.

Patients with infections and two or more elements of SIRS meet the diagnostic criteria
for sepsis. Those with organ failure have severe sepsis and those with refractory
hypotension -septic shock.

During the septic process there is marked activation of the immune system with
extensive cytokine release. This may be coupled with or triggered by systemic
circulation of bacterial toxins. These all cause endothelial cell damage and neutrophil
adhesion. The overall hallmarks are thus those of excessive inflammation,
coagulation and fibrinolytic suppression.

The surviving sepsis campaign highlights the following key areas for attention:

 Prompt administration of antibiotics to cover all likely pathogens coupled with


a rigorous search for the source of infection.
 Haemodynamic stabilisation. Many patients are hypovolaemic and require
aggressive fluid administration. Aim for CVP 8-12 cm H2O, MAP >65mmHg.
 Modulation of the septic response. This includes manoeuvres to counteract the
changes and includes measures such as tight glycaemic control, use of
activated protein C and sometimes intravenous steroids.
In surgical patients, the main groups with septic shock include those with anastomotic
leaks, abscesses and extensive superficial infections such as necrotising fasciitis.
When performing surgery the aim should be to undertake the minimum necessary to
restore physiology. These patients do not fare well with prolonged surgery. Definitive
surgery can be more safely undertaken when physiology is restored and clotting in
particular has been normalised.

Haemorrhagic shock
The average adult blood volume comprises 7% of body weight. Thus in the 70 Kg
adult this will equate to 5 litres. This changes in children (8-9% body weight) and is
slightly lower in the elderly.

The table below outlines the 4 major classes of haemorrhagic shock and their
associated physiological sequelae:

Parameter Class I Class II Class III Class IV


Blood loss ml <750ml 750-1500ml 1500-2000ml >2000ml
Blood loss % <15% 15-30% 30-40% >40%
Pulse rate <100 >100 >120 >140ml
Blood pressure Normal Decreased Decreased Decreased
Respiratory rate 14-20 20-30 30-40 >35
Urine output >30ml 20-30ml 5-15ml <5ml
Symptoms Normal Anxious Confused Lethargic

Decreasing blood pressure during haemorrhagic shock causes organ hypoperfusion


and relative myocardial ishaemia. The cardiac index gives a numerical value for tissue
oxygen delivery and is given by the equation: Cardiac index= 13.4 - [Hb] - SaO2 +
0.03 PaO2. Where Hb is haemoglobin concentration in blood and SaO2 the saturation
and PaO2 the partial pressure of oxygen. Detailed knowledge of this equation is
required for the MRCS Viva but not for part A, although you should understand the
principle.

In patients suffering from trauma the most likely cause of shock is haemorrhage.
However, the following may also be the cause or occur concomitantly:

 Tension pneumothorax
 Spinal cord injury
 Myocardial contusion
 Cardiac tamponade

When assessing trauma patients it is worth remembering that in order to generate a


palpable femoral pulse an arterial pressure of >65mmHg is required.

Once bleeding is controlled and circulating volume normalised the levels of


transfusion should be to maintain a Hb of 7-8 in those with no risk factors for tissue
hypoxia and Hb 10 for those who have such risk factors.

Neurogenic shock
This occurs most often following a spinal cord transection, usually at a high level.
There is resultant interruption of the autonomic nervous system. The result is either
decreased sympathetic tone or increased parasympathetic tone, the effect of
which is a decrease in peripheral vascular resistance mediated by marked
vasodilation.

This results in decreased preload and thus decreased cardiac output (Starlings law).
There is decreased peripheral tissue perfusion and shock is thus produced. In contrast
with many other types of shock peripheral vasoconstrictors are used to return vascular
tone to normal.

Cardiogenic shock
In medical patients the main cause is ischaemic heart disease. In the traumatic
setting direct myocardial trauma or contusion is more likely. Evidence of ECG
changes and overlying sternal fractures or contusions should raise the suspicion of
injury. Treatment is largely supportive and transthoracic echocardiography should be
used to determine evidence of pericardial fluid or direct myocardial injury. The
measurement of troponin levels in trauma patients may be undertaken but they are
less useful in delineating the extent of myocardial trauma than following MI.

When cardiac injury is of a blunt nature and is associated with cardiogenic shock the
right side of the heart is the most likely site of injury with chamber and or valve
rupture. These patients require surgery to repair these defects and will require
cardiopulmonary bypass to achieve this. Some may require intra aortic balloon pump
as a bridge to surgery.

Anaphylactic shock
Anaphylaxis may be defined as a severe, life-threatening, generalised or systemic
hypersensitivity reaction.

Anaphylaxis is one of the few times when you would not have time to look up the
dose of a medication. The Resuscitation Council guidelines on anaphylaxis have
recently been updated. Adrenaline is by far the most important drug in anaphylaxis
and should be given as soon as possible. The recommended doses for adrenaline,
hydrocortisone and chlorphenamine are as follows:

Adrenaline Hydrocortisone Chlorphenamine


< 6 months 150 mcg (0.15ml 1 in 25 mg 250 mcg/kg
1,000)
6 months - 6 years 150 mcg (0.15ml 1 in 50 mg 2.5 mg
1,000)
6-12 years 300 mcg (0.3ml 1 in 100 mg 5 mg
1,000)
Adult and child 12 500 mcg (0.5ml 1 in 200 mg 10 mg
years 1,000)
Adrenaline can be repeated every 5 minutes if necessary. The best site for IM
injection is the anterolateral aspect of the middle third of the thigh.

Common identified causes of anaphylaxis

 food (e.g. Nuts) - the most common cause in children


 drugs
 venom (e.g. Wasp sting)

Release of vasopressin from the pituitary will result in which of the following?

A. Vasoconstriction of the afferent glomerular arteriole

B. Increased permeability of the mesangial cells to glucose

C. Reduced permeability of the inner medullary portion of the collecting


duct to urea

D. Increased secretion of aldosterone from the macula densa

E. Increased water permeability of the distal tubule cells of the kidney

ADH (vasopressin) results in the insertion of aquaporin channels in apical membrane


of the distal tubule and collecting ducts.

Renal Physiology

Overview

 Each nephron is supplied with blood from an afferent arteriole that opens onto
the glomerular capillary bed.
 Blood then flows to an efferent arteriole, supplying the peritubular capillaries
and medullary vasa recta.
 The kidney receives up to 25% of resting cardiac output.

Control of blood flow

 The kidney is able to autoregulate its blood flow between systolic pressures of
80- 180mmHg so there is little variation in renal blood flow.
 This is achieved by myogenic control of arteriolar tone, both sympathetic
input and hormonal signals (e.g. renin) are responsible.

Glomerular structure and function


 Blood inside the glomerulus has considerable hydrostatic pressure.
 The basement membrane has pores that will allow free diffusion of smaller
solutes, larger negatively charged molecules such as albumin are unable to
cross.
 The glomerular filtration rate (GFR) is equal to the concentration of a solute in
the urine, times the volume of urine produced per minute, divided by the
plasma concentration (assuming that the solute is freely diffused e.g. inulin).
 In clinical practice creatinine is used because it is subjected to very little
proximal tubular secretion.
 Although subject to variability, the typical GFR is 125ml per minute.

 Glomerular filtration rate = Total volume of plasma per unit time leaving the
capillaries and entering the bowman's capsule

 Renal clearance = volume plasma from which a substance is removed per


minute by the kidneys

 Substances used to measure GFR have the following features:

1. Inert
2. Free filtration from the plasma at the glomerulus (not protein bound)
3. Not absorbed or secreted at the tubules
4. Plasma concentration constant during urine collection

Examples: inulin, creatinine

GFR = urine concentration (mmol/l) x urine volume (ml/min)


--------------------------------------------------------------------------
plasma concentration (mmol/l)

 The clearance of a substance is dependent not only on its diffusivity across the
basement membrane but also subsequent tubular secretion and / or
reabsorption.
 So glucose which is freely filtered across the basement membrane is usually
reabsorbed from tubules giving a clearance of zero.

Tubular function

 Reabsorption and secretion of substances occurs in the tubules.


 In the proximal tubule substrates such as glucose, amino acids and phosphate
are co-transported with sodium across the semi permeable membrane.
 Up to two thirds of filtered water is reabsorbed in the proximal tubules.
 This will lead to increase in urea concentration in the distal tubule allowing for
its increased diffusion.
 Substances to be secreted into the tubules are taken up from the peritubular
blood by tubular cells.
 Solutes such as paraaminohippuric acid are cleared with a single passage
through the kidneys and this is why it is used to measure renal plasma flow.
Ions such as calcium and phosphate will have a tubular reabsorption that is
influenced by plasma PTH levels.
 Potassium may be both secreted and re-absorbed and is co-exchanged with
sodium.

Loop of Henle

 Approximately 60 litres of water containing 9000mmol sodium enters the


descending limb of the loop of Henle in 24 hours.
 Loops from the juxtamedullary nephrons run deep into the medulla.
 The osmolarity of fluid changes and is greatest at the tip of the papilla.
 The thin ascending limb is impermeable to water, but highly permeable to
sodium and chloride ions.
 This loss means that at the beginning of the thick ascending limb the fluid is
hypo osmotic compared with adjacent interstitial fluid.
 In the thick ascending limb the reabsorption of sodium and chloride ions
occurs by both facilitated and passive diffusion pathways.
 The loops of Henle are co-located with vasa recta, these will have similar
solute compositions to the surrounding extracellular fluid so preventing the
diffusion and subsequent removal of this hypertonic fluid.
 The energy dependent reabsorption of sodium and chloride in the thick
ascending limb helps to maintain this osmotic gradient.

Which of the following hormones is mainly responsible for sodium - potassium


exchange in the salivary ducts?

A. Vasopressin

B. Angiotensin I

C. Aldosterone

D. Somatostatin

E. Cholecystokinin

Aldosterone is responsible for regulating ion exchange in salivary glands. It acts on a


sodium / potassium ion exchange pump.It is a mineralocorticoid hormone derived
from the zona glomerulosa of the adrenal gland.
Parotid gland

Anatomy of the parotid gland


Location Overlying the mandibular ramus; anterior and inferior to the ear.
Salivary duct Crosses the masseter, pierces the buccinator and drains adjacent
to the 2nd upper molar tooth (Stensen's duct).
Structures passing  Facial nerve (Mnemonic: The Zebra Buggered My Cat;
through the gland Temporal Zygomatic, Buccal, Mandibular, Cervical)
 External carotid artery
 Retromandibular vein
 Auriculotemporal nerve

Relations  Anterior: masseter, medial pterygoid, superficial


temporal and maxillary artery, facial nerve,
stylomandibular ligament
 Posterior: posterior belly digastric muscle,
sternocleidomastoid, stylohyoid, internal carotid artery,
mastoid process, styloid process

Arterial supply Branches of external carotid artery


Venous drainage Retromandibular vein
Lymphatic Deep cervical nodes
drainage
Nerve innervation  Parasympathetic-Secretomotor
 Sympathetic-Superior cervical ganglion
 Sensory- Greater auricular nerve

Parasympathetic stimulation produces a water rich, serous saliva. Sympathetic


stimulation leads to the production of a low volume, enzyme-rich saliva.
In a 70 Kg male, what proportion of total body fluid will be contributed by plasma?

A. 50%

B. 5%

C. 35%

D. 65%

E. 25%
70 Kg male = 42 L water (60%
of total body weight)
Fluid compartment physiology

Body fluid compartments comprise intracellular and extracellular compartments. The


latter includes interstitial fluid, plasma and transcellular fluid.
Typical figures are based on the 70 Kg male.

Body fluid volumes


Compartment Volume in litres Percentage of total volume
Intracellular 28 L 60-65%
Extracellular 14 L 35-40%
Plasma 3L 8%
Interstitial 10 L 24%
Transcellular 1 L 3%
Figures are approximate
A 23 year old man is undergoing an inguinal hernia repair under local anaesthesia.
The surgeon encounters a bleeding site which he manages with diathermy. About a
minute or so later the patient complains that he is able to feel the burning pain of the
heat at the operative site. Which of the following nerve fibres is responsible for the
transmission of this signal?

A. A α fibres

B. A β fibres

C. B fibres

D. C fibres

E. None of the above

Slow transmission of mechanothermal stimuli is transmitted via C fibres.


A α fibres transmit information relating to motor proprioception, A β fibres transmit
touch and pressure and B fibres are autonomic fibres.

Pain - neuronal transmission

Somatic pain

 Peripheral nociceptors are innervated by either small myelinated fibres (A-


gamma) fibres or by unmyelinated C fibres.
 The A gamma fibres register high intensity mechanical stimuli. The C fibres
usually register high intensity mechanothermal stimuli.

What is the approximate volume of pancreatic secretions in a 24 hour period?


A. 100ml

B. 200ml

C. 500ml

D. 1500ml

E. 3000ml

Typically the pancreas secretes up to 1.5L per day.

Pancreas exocrine physiology

Pancreatic juice

 Alkaline solution pH 8
 1500ml/day
 Composition: acinar secretion (ENZYMES: trypsinogen, procarboxylase,
amylase, lecithin) and ductile secretion (HCO, Na+, water)
 Pancreatic juice action: Trypsinogen is converted via enterokinase to active
trypsin in the duodenum. Trypsin then activates the other inactive enzymes.

A 34 year old lady has just undergone a parathyroidectomy for primary


hyperparathyroidism. The operation is difficult and all 4 glands were explored. The
wound was clean and dry at the conclusion of the procedure and a suction drain
inserted. On the ward she becomes irritable and develops respiratory stridor. On
examination her neck is soft and the drain empty. Which of the following treatments
should be tried initially?

A. Administration of intravenous calcium gluconate

B. Administration of intravenous lorazepam

C. Removal of the skin closure on the ward

D. Direct laryngoscopy

E. Administration of calcichew D3 orally

Exploration of the parathyroid glands may result in impairment of the blood supply.
Serum PTH levels can fall quickly and features of hypocalcaemia may ensue, these
include neuromuscular irritability and laryngospasm. Prompt administration of
intravenous calcium gluconate can be lifesaving. The absence of any neck swelling
and no blood in the drain would go against a contained haematoma in the neck (which
should be managed by removal of skin closure).

Calcium homeostasis

Calcium ions are linked to a wide range of physiological processes. The largest store
of bodily calcium is contained within the skeleton. Calcium levels are primarily
controlled by parathyroid hormone, vitamin D and calcitonin.

Hormonal regulation of calcium


Hormone Actions
Parathyroid hormone (PTH)  Increase calcium levels and decrease
phosphate levels
 Increases bone resorption
 Immediate action on osteoblasts to
increase ca2+ in extracellular fluid
 Osteoblasts produce a protein signaling
molecule that activate osteoclasts which
cause bone resorption
 Increases renal tubular reabsorption of
calcium
 Increases synthesis of 1,25(OH)2D
(active form of vitamin D) in the kidney
which increases bowel absorption of Ca2+
 Decreases renal phosphate reabsorption

1,25-dihydroxycholecalciferol  Increases plasma calcium and plasma


(the active form of vitamin D) phosphate
 Increases renal tubular reabsorption and
gut absorption of calcium
 Increases osteoclastic activity
 Increases renal phosphate reabsorption

Calcitonin  Secreted by C cells of thyroid


 Inhibits intestinal calcium absorption
 Inhibits osteoclast activity
 Inhibits renal tubular absorption of
calcium

Both growth hormone and thyroxine also play a small role in calcium metabolism.
Theme: Interpretation of aterial blood gas results

A. pH 7.19, pCO2 10.2, pO2 16 (FiO2 85%), Bicarbonate 23.8, Base


excess -2.2 mmol
B. pH 7.57, PaCO2 3.5, Pa O2 24.5 (FiO2 85%), Bicarbonate 23.5, Base
excess +1.8 mmol
C. pH 7.14, PaCO2 7.4, PaO2 8.9 (FiO2 40%), Bicarbonate 14 mmol,
Base excess -10.6
D. pH 7.36, PaCO2 7.3, PO2 8.9 (FiO2 40%), Bicarbonate 30.2, Base
excess 5.3
E. pH 7.32, PCO2 3.8, PaO2 22.2 (FiO2 40%), Bicarbonate 19.1, Base
excess -7.9

Which of the following arterial blood gases fit with the description below?

33. Acute respiratory acidosis

pH 7.19, pCO2 10.2, pO2 16 (FiO2 85%), Bicarbonate 23.8, Base excess -
2.2 mmol

34. Metabolic acidosis with a compensatory respiratory alkalosis

pH 7.32, PCO2 3.8, PaO2 22.2 (FiO2 40%), Bicarbonate 19.1, Base excess
-7.9

35. Chronic respiratory acidosis with a compensatory metabolic alkalosis

pH 7.36, PaCO2 7.3, PO2 8.9 (FiO2 40%), Bicarbonate 30.2, Base excess
5.3

Arterial blood gas interpretation

In advanced life support training, a 5 step approach to arterial blood gas interpretation
is advocated.

1. How is the patient?

2. Is the patient hypoxaemic?


The Pa02 on air should be 10.0-13.0 kPa

3. Is the patient acidaemic (pH <7.35) or alkalaemic (pH >7.45)

4. What has happened to the PaCO2?


If there is acidaemia, an elevated PaCO2 will account for this

5. What is the bicarbonate level or base excess?


A metabolic acidosis will have a low bicarbonate level and a low base excess (< -2
mmol)
A metabolic alkalosis will have a high bicarbonate and a high base excess (> +2
mmol)

Cortisol is predominantly produced by which of the following?

A. Zona fasciculata of the adrenal

B. Zona glomerulosa of the adrenal

C. Zona reticularis of the adrenal

D. Adrenal medulla

E. Posterior lobe of the pituitary


Relative Glucocorticoid
activity:

Hydrocortisone = 1
Prednisolone = 4
Dexamethasone = 25

Cortisol is produced by the zona fasciculata of the adrenal gland.

Cortisol

 Glucocorticoid
 Released by zona fasiculata of the adrenal gland
 90% protein bound; 10% active
 Circadian rhythm: High in the mornings
 Negative feedback via ACTH

Actions

 Glycogenolysis
 Glucaneogenesis
 Protein catabolism
 Lipolysis
 Stress response
 Anti-inflammatory
 Decrease protein in bones
 Increase gastric acid
 Increases neutrophils/platelets/red blood cells
 Inhibits fibroblastic activity
Which of the following is not an intravenous colloid?

A. Gelofusine

B. Dextran 40

C. Human albumin solution

D. Hydroxyethyl starch

E. Bicarbonate 8.4%

Bicarbonate is a crystalloid

Pre operative fluid management

Fluid management has been described in the British Consensus guidelines on IV


fluid therapy for Adult Surgical patients (GIFTASUP)

The Recommendations include:

 Use Ringer's lactate or Hartmann's when a crystalloid is needed for


resuscitation or replacement of fluids. Avoid 0.9% N. Saline (due to risk of
hyperchloraemic acidosis) unless patient vomiting or has gastric drainage.
 Use 0.4%/0.18% dextrose saline or 5% dextrose in maintenance fluids. It
should not be used in resuscitation or as replacement fluids.
 Adult maintenance fluid requirements are: Na 50-100 mmol/day and K 40-80
mmol/day in 1.5-2.5L fluid per day.
 Patients for elective surgery should NOT be nil by mouth for >2 hours (unless
has disorder of gastric emptying).
 Patients for elective surgery should be given carbohydrate rich drinks 2-3h
before. Ideally this should form part of a normal pre op plan to facilitate
recovery.
 Avoid mechanical bowel preparation.
 If bowel prep is used, simultaneous administration of Hartmann's or Ringer's
lactate should be considered.
 Excessive fluid losses from vomiting should be treated with a crystalloid with
potassium replacement. 0.9% N. Saline should be given if there is
hypochloraemia. Otherwise Hartmann's or Ringer lactate should be given for
diarrhoea/ileostomy/ileus/obstruction. Hartmann's should also be given in
sodium losses secondary to diuretics.
 High risk patients should receive fluids and inotropes.
 An attempt should be made to detect pre or operative hypovolaemia using
flow based measurements. If this is not available, then clinical evaluation is
needed i.e. JVP, pulse volume etc.
 In Blood loss or infection causing hypovolaemia should be treated with a
balanced crystalloid or colloid (or until blood available in blood loss). A
critically ill patient is unable to excrete Na or H20 leading to a 5% risk of
interstitial oedema. Therefore 5% dextrose as well as colloid should be given.
 Give 200mls of colloid in hypovolaemia, repeat until clinical parameters
improve.

Theme: Electrolyte disorders

A. Hypotonic hypovolaemic hyponatraemia


B. Hypotonic hypervolaemic hyponatraemia
C. Pseudohyponatraemia
D. Syndrome of inappropriate ADH secretion (SIADH)
E. Hypertonic hyponatraemia
F. Over administration 5% dextrose

Please select the most likely reason for hyponatraemia for each scenario given. Each
option may be used once, more than once or not at all.

38. A 73 year old man presents to pre operative clinic for an elective total hip
replacement. He is on frusemide for hypertension. He is found to have the
following blood results:
Na 120
Urine Na 10 (low)
Serum osmolality 280 (normal)

Hypotonic hypovolaemic hyponatraemia

The blood results reflect extra-renal sodium loss. The body is trying to
preserve the sodium by not allowing any sodium into the urine (hence the low
Na in the urine). Note with renal sodium loss the Urinary sodium is high.

39. A 67 year old man presents to pre operative clinic for an elective hernia
repair. He is on frusemide for heart failure. He is found to have the following
blood results:
Na 120
Urine Na 35 (high)
Urine osmolality 520 (high)
Serum osmolality 265 (low)

You answered Hypotonic hypervolaemic hyponatraemia

The correct answer is Syndrome of inappropriate ADH secretion


(SIADH)
This blood picture fits with SIADH. SIADH causes retention of fluid from
the urine (concentrated urine) into the blood vessels, therefore diluting the
fluid in the blood vessels (low osmolality). Management involves removing
the cause and fluid restriction.

40. A 77 year old man presents to pre operative clinic for a total knee
replacement. He is on frusemide for hypertension. He is known to have
multiple myeloma. He is found to have the following blood results:
Na 120
Serum osmolality 280 (normal)
Urine osmolallity normal
Urine Na normal

Pseudohyponatraemia

Hyperlipidaemia and multiple myeloma are known to cause a


pseudohyponatraemia.

SIADH:

 Low serum osmolality


 High/Normal urine osmolality

Hyponatraemia

Hyponatraemia

This is commonly tested in the MRCS (despite most surgeons automatically seeking
medical advice if this occurs!). The most common cause in surgery is the over
administration of 5% dextrose.

Hyponatraemia may be caused by water excess or sodium depletion. Causes of


pseudohyponatraemia include hyperlipidaemia (increase in serum volume) or a taking
blood from a drip arm. Urinary sodium and osmolarity levels aid making a diagnosis.

Classification
Urinary sodium > 20 Sodium depletion, renal loss Mnemonic: Syndrome
mmol/l of INAPPropriate Anti-
 Patient often Diuretic Hormone:
hypovolaemic In creased
 Diuretics (thiazides) Na (sodium)
 Addison's PP (urine)
 Diuretic stage of renal
failure
 SIADH (serum osmolality
low, urine osmolality
high, urine Na high)
 Patient often euvolaemic

Urinary sodium < 20 Sodium depletion, extra-renal loss


mmol/l
 Diarrhoea, vomiting,
sweating
 Burns, adenoma of rectum
(if villous lesion and
large)

Water excess (patient  Secondary


often hypervolaemic hyperaldosteronism: CCF,
and oedematous) cirrhosis
 Reduced GFR: renal
failure
 IV dextrose, psychogenic
polydipsia

Management

Symptomatic Hyponatremia :

Acute hyponatraemia with Na <120: immediate therapy. Central Pontine Myelinolisis,


may occur from overly rapid correction of serum sodium. Aim to correct until the Na
is > 125 at a rate of 1 mEq/h. Normal saline with frusemide is an alternative method.

The sodium requirement can be calculated as follows :

(125 - serum sodium) x 0.6 x body weight = required mEq of sodium


A 53 year old man is on the intensive care unit following an emergency abdominal
aortic aneurysm repair. He develops abdominal pain and diarrhoea and is profoundly
unwell. His abdomen has no features of peritonism. Which of the following arterial
blood gas pictures is most likely to be present?

A. pH 7.45, pO2 10.1, pCO2 3.2, Base excess 0, Lactate 0

B. pH 7.35, pO2 8.0, pCO2 5.2, Base excess 2, Lactate 1

C. pH 7.20, pO2 9.0, pCO2 3.5, Base excess -10, Lactate 8

D. pH 7.29, pO2 8.9, pCO2 5.9, Base excess -4, Lactate 3


E. pH 7.30, pO2 9.2 pCO2 4.8, Base excess -2, lactate 1

This man is likely to have a metabolic acidosis secondary to a mesenteric infarct.

Disorders of acid - base balance

Disorders of acid- base balance are often covered in the MRCS part A, both in the
SBA and EMQ sections.

The acid-base normogram below shows how the various disorders may be
categorised

Image sourced from Wikipedia

Metabolic acidosis

 This is the most common surgical acid - base disorder.


 Reduction in plasma bicarbonate levels.
 Two mechanisms:

1. Gain of strong acid (e.g. diabetic ketoacidosis)


2. Loss of base (e.g. from bowel in diarrhoea)
- Classified according to the anion gap, this can be calculated by:
(Na+ + K+) - (Cl- + HCO3-).
- If a question supplies the chloride level then this is often a clue that the anion gap
should be calculated. The normal range = 10-18 mmol/L
Normal anion gap ( = hyperchloraemic metabolic acidosis)

 Gastrointestinal bicarbonate loss: diarrhoea, ureterosigmoidostomy, fistula


 Renal tubular acidosis
 Drugs: e.g. acetazolamide
 Ammonium chloride injection
 Addison's disease

Raised anion gap

 Lactate: shock, hypoxia


 Ketones: diabetic ketoacidosis, alcohol
 Urate: renal failure
 Acid poisoning: salicylates, methanol

Metabolic acidosis secondary to high lactate levels may be subdivided into two types:

 Lactic acidosis type A: (Perfusion disorders e.g.shock, hypoxia, burns)


 Lactic acidosis type B: (Metabolic e.g. metformin toxicity)

Metabolic alkalosis

 Usually caused by a rise in plasma bicarbonate levels.


 Rise of bicarbonate above 24 mmol/L will typically result in renal excretion of
excess bicarbonate.
 Caused by a loss of hydrogen ions or a gain of bicarbonate. It is due mainly to
problems of the kidney or gastrointestinal tract

Causes

 Vomiting / aspiration (e.g. Peptic ulcer leading to pyloric stenosis, nasogastric


suction)
 Diuretics
 Liquorice, carbenoxolone
 Hypokalaemia
 Primary hyperaldosteronism
 Cushing's syndrome
 Bartter's syndrome
 Congenital adrenal hyperplasia

Mechanism of metabolic alkalosis


 Activation of renin-angiotensin II-aldosterone (RAA) system is a key factor
 Aldosterone causes reabsorption of Na+ in exchange for H+ in the distal
convoluted tubule
 ECF depletion (vomiting, diuretics) --> Na+ and Cl- loss --> activation of RAA
system --> raised aldosterone levels
 In hypokalaemia, K+ shift from cells --> ECF, alkalosis is caused by shift of
H+ into cells to maintain neutrality

Respiratory acidosis

 Rise in carbon dioxide levels usually as a result of alveolar hypoventilation


 Renal compensation may occur leading to Compensated respiratory acidosis

Causes

 COPD
 Decompensation in other respiratory conditions e.g. Life-threatening asthma /
pulmonary oedema
 Sedative drugs: benzodiazepines, opiate overdose

Respiratory alkalosis

 Hyperventilation resulting in excess loss of carbon dioxide


 This will result in increasing pH

Causes

 Psychogenic: anxiety leading to hyperventilation


 Hypoxia causing a subsequent hyperventilation: pulmonary embolism, high
altitude
 Early salicylate poisoning*
 CNS stimulation: stroke, subarachnoid haemorrhage, encephalitis
 Pregnancy

*Salicylate overdose leads to a mixed respiratory alkalosis and metabolic acidosis.


Early stimulation of the respiratory centre leads to a respiratory alkalosis whilst later
the direct acid effects of salicylates (combined with acute renal failure) may lead to an
acidosis
A 48 year old women suffers blunt trauma to the head and develops respiratory
compromise. As a result she develops hypercapnia. Which of the following effects is
most likely to ensue?

A. Cerebral vasoconstriction
B. Cerebral vasodilation

C. Cerebral blood flow will remain unchanged

D. Shunting of blood to peripheral tissues will occur in preference to


CNS perfusion

E. None of the above

Hypercapnia will tend to produce cerebral vasodilation. This is of considerable


importance in patients with cranial trauma as it may increase intracranial pressure.

Applied neurophysiology

 Pressure within the cranium is governed by the Monroe-Kelly doctrine. This


considers the skull as a closed box. Increases in mass can be accommodated
by loss of CSF. Once a critical point is reached (usually 100- 120ml of CSF
lost) there can be no further compensation and ICP rises sharply. The next step
is that pressure will begin to equate with MAP and neuronal death will occur.
Herniation will also accompany this process.
 The CNS can autoregulate its own blood supply. Vaso constriction and
dilatation of the cerebral blood vessels is the primary method by which this
occurs. Extremes of blood pressure can exceed this capacity resulting in risk
of stroke. Other metabolic factors such as hypercapnia will also cause
vasodilation, which is of importance in ventilating head injured patients.
 The brain can only metabolise glucose, when glucose levels fall,
consciousness will be impaired.

A patient is seen in clinic complaining of abdominal pain. Routine bloods show:

Na+ 142 mmol/l


K+ 4.0 mmol/l
Chloride 104 mmol/l
Bicarbonate 19 mmol/l
Urea 7.0 mmol/l
Creatinine 112 µmol/l

What is the anion gap?

A. 4 mmol/L

B. 14 mmol/L
C. 20 mmol/L

D. 21 mmol/L

E. 23 mmol/L

The anion gap may be calculated by using (sodium + potassium) - (bicarbonate +


chloride)

= (142 + 4.0) - (104 + 19) = 23 mmol/L

Anion gap

The anion gap is calculated by:

(sodium + potassium) - (bicarbonate + chloride)

A normal anion gap is 8-14 mmol/L

It is useful to consider in patients with a metabolic acidosis:

Causes of a normal anion gap or hyperchloraemic metabolic acidosis

 gastrointestinal bicarbonate loss: diarrhoea, ureterosigmoidostomy, fistula


 renal tubular acidosis
 drugs: e.g. acetazolamide
 ammonium chloride injection
 Addison's disease

Causes of a raised anion gap metabolic acidosis

 lactate: shock, hypoxia


 ketones: diabetic ketoacidosis, alcohol
 urate: renal failure
 acid poisoning: salicylates, methanol

A surgeon is considering using lignocaine as local anasthesia for a minor procedure.


Which of the following best accounts for its actions?

A. Blockade of neuronal acetylcholine receptors

B. Blockade of neuronal nicotinic receptors

C. Blockade of neuronal sodium channels


D. Blockade of neuronal potassium channels

E. Blockade of neuronal calcium channels

Lignocaine blocks sodium channels. They will typically be activated first, hence the
pain some patients experience on administration.

Local anaesthetic agents

Lidocaine

 An amide
 Local anaesthetic and a less commonly used antiarrhythmic (affects Na
channels in the axon)
 Hepatic metabolism, protein bound, renally excreted
 Toxicity: due to IV or excess administration. Increased risk if liver
dysfunction or low protein states. Note acidosis causes lidocaine to detach
from protein binding.
 Drug interactions: Beta blockers, ciprofloxacin, phenytoin
 Features of toxicity: Initial CNS over activity then depression as lidocaine
initially blocks inhibitory pathways then blocks both inhibitory and activating
pathways. Cardiac arrhythmias.
 Increased doses may be used when combined with adrenaline to limit systemic
absorption.

Cocaine

 Pure cocaine is a salt, usually cocaine hydrochloride. It is supplied for local


anaesthetic purposes as a paste.
 It is supplied for clinical use in concentrations of 4 and 10%. It may be applied
topically to the nasal mucosa. It has a rapid onset of action and has the
additional advantage of causing marked vasoconstriction.
 It is lipophillic and will readily cross the blood brain barrier. Its systemic
effects also include cardiac arrhythmias and tachcardia.
 Apart from its limited use in ENT surgery it is otherwise used rarely in
mainstream surgical practice.

Bupivicaine

 Bupivacaine binds to the intracellular portion of sodium channels and blocks


sodium influx into nerve cells, which prevents depolarization.
 It has a much longer duration of action than lignocaine and this is of use in
that it may be used for topical wound infiltration at the conclusion of surgical
procedures with long duration analgesic effect.
 It is cardiotoxic and is therefore contra indicated in regional blockage in case
the tourniquet fails.
 The co-administration of adrenaline concentrates it at the site of action and
allows the use of higher doses.

Prilocaine

 Similar mechanism of action to other local anaesthetic agents. However, it is


far less cardiotoxic and is therefore the agent of choice for intravenous
regional anaesthesia e.g. Biers Block.

All local anaesthetic agents dissociate in tissues and this contributes to their
therapeutic effect. The dissociation constant shifts in tissues that are acidic e.g. where
an abscess is present and this reduce the efficacy.

Doses of local anaesthetics


Agent Dose plain Dose with adrenaline
Lignocaine 3mg/Kg 7mg/Kg
Bupivicane 2mg/Kg 2mg/Kg
Prilocaine 6mg/Kg 9mg/Kg
These are a guide only as actual doses depend on site of administration, tissue
vascularity and co-morbidities.

References
An excellent review is provided by:
French J and Sharp L. Local Anaesthetics. Ann R Coll Surg Engl 2012; 94: 76-80.
A 22 year old man suffers a blunt head injury. He is drowsy and has a GCS of 7 on
admission. Which of the following is the major determinant of cerebral blood flow in
this situation?

A. Systemic blood pressure

B. Mean arterial pressure

C. Intra cranial pressure

D. Hypoxaemia

E. Acidosis

Theme from 2009 Exam

Hypoxaemia and acidosis may both affect cerebral blood flow. However, in the
traumatic situation increases in intracranial pressure are far more likely to occur
especially when GCS is low. This will adversely affect cerebral blood flow.
Cerebral blood flow

 CNS autoregulates its own blood supply


 Factors affecting the cerebral pressure include; systemic carbon dioxide levels,
CNS metabolism, CNS trauma, CNS pressure
 The PaCO2 is the most potent mediator
 Acidosis and hypoxaemia will increase cerebral blood flow but to a lesser
degree
 Cerebral perfusion pressure may increase in patients with head injuries and
this can result in impaired blood flow
 Intra cerebral pressure governed by Monroe-Kelly Doctrine which considers
brain as closed box, changes in pressure are offset by loss of CSF. When this
is no longer possible ICP rises
 Which main group of receptors does dobutamine bind to?

A. α-1

B. α-2

C. ß-1

D. ß-2

E. D-1

Dobutamine is a sympathomimetic with both alpha- and beta-agonist
properties; it displays a considerable selectivity for beta1-cardiac receptors.
 Inotropes and cardiovascular receptors

Inotrope Cardiovascular receptor action
Adrenaline α-1, α-2, β-1, β-2
Noradrenaline α-1,( α-2), (β-1), (β-2)
Dobutamine β-1, (β 2)
Dopamine (α-1), (α-2), (β-1), D-1,D-2
 Minor receptor effects in brackets

Effects of receptor binding


α-1, α-2 vasoconstriction
β-1 increased cardiac contractility and HR
β-2 vasodilatation
D-1 renal and spleen vasodilatation
D-2 inhibits release of noradrenaline
Which of the following is responsible for the release and synthesis of calcitonin?
A. Parathyroid glands

B. Anterior pituitary

C. Thyroid gland

D. Posterior pituitary

E. Adrenal glands

Calcitonin has the opposite effect of PTH and is release from the thyroid gland.

Calcium homeostasis

Calcium ions are linked to a wide range of physiological processes. The largest store
of bodily calcium is contained within the skeleton. Calcium levels are primarily
controlled by parathyroid hormone, vitamin D and calcitonin.

Hormonal regulation of calcium


Hormone Actions
Parathyroid hormone (PTH)  Increase calcium levels and decrease
phosphate levels
 Increases bone resorption
 Immediate action on osteoblasts to
increase ca2+ in extracellular fluid
 Osteoblasts produce a protein signaling
molecule that activate osteoclasts which
cause bone resorption
 Increases renal tubular reabsorption of
calcium
 Increases synthesis of 1,25(OH)2D
(active form of vitamin D) in the kidney
which increases bowel absorption of Ca2+
 Decreases renal phosphate reabsorption

1,25-dihydroxycholecalciferol  Increases plasma calcium and plasma


(the active form of vitamin D) phosphate
 Increases renal tubular reabsorption and
gut absorption of calcium
 Increases osteoclastic activity
 Increases renal phosphate reabsorption

Calcitonin  Secreted by C cells of thyroid


 Inhibits intestinal calcium absorption
 Inhibits osteoclast activity
 Inhibits renal tubular absorption of
calcium

Both growth hormone and thyroxine also play a small role in calcium metabolism.
What is the half life of insulin in the circulation of a normal healthy adult?

A. Less than 30 minutes

B. Between 1 and 2 hours

C. Between 2 and 3 hours

D. Between 4 and 5 hours

E. Over 6 hours

Insulin is degraded by enzymes in the circulation. It typically has a half life of less
than 30 minutes. Abnormalities of the clearance of insulin may occur in type 2
diabetes.

Insulin

 Anabolic hormone

Structure

 and chain linked by disulphide bridges

Synthesis

 Pro-insulin is formed by the rough endoplasmic reticulum in pancreatic beta


cells. Then pro-insulin is cleaved to form insulin and C-peptide. Insulin is
stored in secretory granules and released in response to Ca.

Function

 Secreted in response to hyperglycaemia


 Glucose utilisation and glycogen synthesis
 Inhibits lipolysis
 Reduces muscle protein loss

Which of the following statements about blood clotting is untrue?


A. Platelet adhesion to disrupted endothelium is dependent upon von
Willebrand factor

B. Protein C is a vitamin K dependent substance

C. The bleeding time provides an assessment of platelet function

D. The prothrombin time tests the extrinsic system

E. Administration of aprotinin during liver transplantation surgery prolongs


survival

Although aprotinin reduces fibrinolysis and thus bleeding, it is associated with increased risk
of death and was withdrawn in 2007. Protein C is dependent upon vitamin K and this may
paradoxically increase the risk of thrombosis during the early phases of warfarin treatment.

Coagulation cascade

Two pathways lead to fibrin formation

Intrinsic pathway (components already present in the blood)

 Minor role in clotting


 Subendothelial damage e.g. collagen
 Formation of the primary complex on collagen by high-molecular-weight kininogen
(HMWK), prekallikrein, and Factor 12
 Prekallikrein is converted to kallikrein and Factor 12 becomes activated
 Factor 12 activates Factor 11
 Factor 11 activates Factor 9, which with its co-factor Factor 8a form the tenase
complex which activates Factor 10

Extrinsic pathway (needs tissue factor released by damaged tissue)

 Tissue damage
 Factor 7 binds to Tissue factor
 This complex activates Factor 9
 Activated Factor 9 works with Factor 8 to activate Factor 10

Common pathway
 Activated Factor 10 causes the conversion of prothrombin to thrombin
 Thrombin hydrolyses fibrinogen peptide bonds to form fibrin and also activates
factor 8 to form links between fibrin molecules

Fibrinolysis
Plasminogen is converted to plasmin to facilitate clot resorption

Image sourced from Wikipedia

Intrinsic pathway Increased APTT Factors 8,9,11,12

Extrinsic pathway Increased PT Factor 7

Common pathway Increased APTT & PT Factors 2,5,10

Vitamin K dependent Factors 2,7,9,10

Theme: Management of vomiting

A. Ondansetron
B. Metoclopramide
C. Cyclizine
D. Erythromycin
E. Cisapride
F. Haloperidol

Please select the most appropriate drug for the given scenario. Each option may be
used once, more than once or not at all.
3. A 78 year old manwith diabetes develops autonomic gastropathy and persistant
a troublesome vomiting.

You answered Ondansetron

The correct answer is Erythromycin

Unlike metoclopramide the effects of erythromycin on gastric empyting are not


mediated via the vagus nerve.

4. A drug which blocks the chemoreceptor trigger zone in the area postrema.

You answered Haloperidol

The correct answer is Ondansetron

5 HT3 blockers are most effective for many types of nausea for this reason.

5. A 48 year old man with oesphageal varices has a profuse haemorrhage on the
ward.

Metoclopramide

Intravenous metoclopramide causes increased oesophageal pressure and this


may temporarily slow the rate of haemorrhage whilst more definitive measures
are instigated.

Vomiting

Reflex oral expulsion of gastric (and sometimes intestinal) contents - reverse


peristalsis and abdominal contraction

The vomiting centre is in part of the medulla oblongata and is triggered by receptors
in several locations:

 Labyrinthine receptors of ear (motion sickness)


 Overdistention receptors of duodenum and stomach
 Trigger zone of CNS - many drugs (e.g., opiates) act here
 Touch receptors in throat
 Sensory innervation rich, both extrinsic and intrinsic
Which of the following cell types is least likely to be found in a wound 1 week
following injury?

A. Macrophages

B. Fibroblasts

C. Myofibroblasts

D. Endothelial cells

E. Neutrophils

Theme from April 2012 Exam


Myofibroblasts are differentiated fibroblasts composed, in which the cytoskeleton
contains actin filaments. These cell types facilitate wound contracture and are the
hallmark of a mature wound.

Phases of wound healing

Phase Key features Cells Timeframe


Haemostasis  Vasospasm in adjacent Erythrocytes Seconds/
vessels Minutes
 Platelet plug formation and
generation of fibrin rich clot

Inflammation  Neutrophils migrate into Neutrophils, Days


wound (function impaired in fibroblasts and
diabetes). macrophages
 Growth factors released,
including basic fibroblast
growth factor and vascular
endothelial growth factor.
 Fibroblasts replicate within
the adjacent matrix and
migrate into wound.
 Macrophages and fibroblasts
couple matrix regeneration
and clot substitution.

Regeneration  Platelet derived growth factor Fibroblasts, Weeks


and transformation growth endothelial cells,
factors stimulate fibroblasts macrophages
and epithelial cells.
 Fibroblasts produce a
collagen network.
 Angiogenesis occurs and
wound resembles granulation
tissue.

Remodelling  Longest phase of the healing Myofibroblasts 6 weeks to 1


process and may last up to year
one year (or longer).
 During this phase fibroblasts
become differentiated
(myofibroblasts) and these
facilitate wound contraction.
 Collagen fibres are
remodelled.
 Microvessels regress leaving
a pale scar.

The blood - brain barrier is not highly permeable to which of the following?

A. Carbon dioxide

B. Barbituates

C. Glucose

D. Oxygen

E. Hydrogen ions

The blood brain barrier is relatively impermeable to highly dissociated compounds.

Cerebrospinal fluid

The CSF fills the space between the arachnoid mater and pia mater (covering surface
of the brain). The total volume of CSF in the brain is approximately 150ml.
Approximately 500 ml is produced by the ependymal cells in the choroid plexus
(70%), or blood vessels (30%). It is reabsorbed via the arachnoid granulations which
project into the venous sinuses.

Circulation
1. Lateral ventricles (via foramen Munro)
2. 3rd ventricle
3. Cerebral aqueduct (aqueduct Sylvius)
4. 4th ventricle (via foramina of Magendie and Luschka)
5. Subarachnoid space
6. Reabsorbed into venous system via arachnoid granulations in superior sagittal sinus

Composition
 Glucose: 50-80mg/dl
 Protein: 15-40 mg/dl
 Red blood cells: Nil
 White blood cells: 0-3 cells/ mm3

A 43 year old presents to the urology clinic complaining of impotence. Which of the
following will occur in response to increased penile parasympathetic stimulation?

A. Detumescence

B. Ejaculation

C. Erection

D. Vasospasm of the penile branches of the pudendal artery

E. Contraction of the smooth muscle in the epididymis and vas deferens

Parasympathetic stimulation causes erection. Sympathetic stimulation will produce


ejaculation, detumescence and vasospasm of the pudendal artery. It will also cause
contraction of the smooth muscle in the epididymis and vas to convey the ejaculate.

Penile erection

Physiology of erection
Autonomic  Sympathetic nerves originate from T11-L2 and parasympathetic
nerves from S2-4 join to form pelvic plexus.
 Parasympathetic discharge causes erection, sympathetic
discharge causes ejaculation and detumescence.

Somatic Supplied by dorsal penile and pudendal nerves. Efferent signals are
nerves relayed from Onufs nucleus (S2-4) to innervate ischiocavernosus and
bulbocavernosus muscles.

Autonomic discharge to the penis will trigger the veno-occlusive mechanism which
triggers the flow of arterial blood into the penile sinusoidal spaces. As the inflow
increases the increased volume in this space will secondarily lead to compression of
the subtunical venous plexus with reduced venous return. During the detumesence
phase the arteriolar constriction will reduce arterial inflow and thereby allow venous
return to normalise.

Priapism
Prolonged unwanted erection, in the absence of sexual desire, lasting more than 4
hours.

Classification of priaprism
Low flow Due to veno-occlusion (high intracavernosal pressures).
priaprism
 Most common type
 Often painful
 Often low cavernosal flow
 If present for >4 hours requires emergency treatment

High flow Due to unregulated arterial blood flow.


priaprism
 Usually presents as semi rigid painless erection

Recurrent Typically seen in sickle cell disease, most commonly of high


priaprism flow type.

Causes

 Intracavernosal drug therapies (e.g. for erectile dysfunction>


 Blood disorders such as leukaemia and sickle cell disease
 Neurogenic disorders such as spinal cord transection
 Trauma to penis resulting in arterio-venous malformations

Tests

 Exclude sickle cell/ leukaemia


 Consider blood sampling from cavernosa to determine whether high or low
flow (low flow is often hypoxic)

Management

 Ice packs/ cold showers


 If due to low flow then blood may be aspirated from copora or try
intracavernosal alpha adrenergic agonists.
 Delayed therapy of low flow priaprism may result in erectile dysfunction.

n class II haemorrhagic shock in a 70Kg male, one would not expect to find?

A. Blood loss greater than 750ml

B. Tachycardia

C. Decreased blood pressure

D. Urine output less than 20ml


E. Anxiety

Urine output in class II shock (assuming 70Kg adult) is typically between 20 and
30ml.

Shock

 Shock occurs when there is insufficient tissue perfusion.


 The pathophysiology of shock is an important surgical topic and may be
divided into the following aetiological groups:
 Septic
 Haemorrhagic
 Neurogenic
 Cardiogenic
 Anaphylactic

Septic shock
Septic shock is a major problem and those patients with severe sepsis have a mortality
rate in excess of 40%. In those who are admitted to intensive care mortality ranges
from 6% with no organ failure to 65% in those with 4 organ failure.

Sepsis is defined as an infection that triggers a particular Systemic Inflammatory


Response Syndrome (SIRS). This is characterised by body temperature outside 36
o
C - 38 o C, HR >90 beats/min, respiratory rate >20/min, WBC count >12,000/mm3 or
< 4,000/mm3.

Patients with infections and two or more elements of SIRS meet the diagnostic criteria
for sepsis. Those with organ failure have severe sepsis and those with refractory
hypotension -septic shock.

During the septic process there is marked activation of the immune system with
extensive cytokine release. This may be coupled with or triggered by systemic
circulation of bacterial toxins. These all cause endothelial cell damage and neutrophil
adhesion. The overall hallmarks are thus those of excessive inflammation,
coagulation and fibrinolytic suppression.

The surviving sepsis campaign highlights the following key areas for attention:

 Prompt administration of antibiotics to cover all likely pathogens coupled with


a rigorous search for the source of infection.
 Haemodynamic stabilisation. Many patients are hypovolaemic and require
aggressive fluid administration. Aim for CVP 8-12 cm H2O, MAP >65mmHg.
 Modulation of the septic response. This includes manoeuvres to counteract the
changes and includes measures such as tight glycaemic control, use of
activated protein C and sometimes intravenous steroids.
In surgical patients, the main groups with septic shock include those with anastomotic
leaks, abscesses and extensive superficial infections such as necrotising fasciitis.
When performing surgery the aim should be to undertake the minimum necessary to
restore physiology. These patients do not fare well with prolonged surgery. Definitive
surgery can be more safely undertaken when physiology is restored and clotting in
particular has been normalised.

Haemorrhagic shock
The average adult blood volume comprises 7% of body weight. Thus in the 70 Kg
adult this will equate to 5 litres. This changes in children (8-9% body weight) and is
slightly lower in the elderly.

The table below outlines the 4 major classes of haemorrhagic shock and their
associated physiological sequelae:

Parameter Class I Class II Class III Class IV


Blood loss ml <750ml 750-1500ml 1500-2000ml >2000ml
Blood loss % <15% 15-30% 30-40% >40%
Pulse rate <100 >100 >120 >140ml
Blood pressure Normal Decreased Decreased Decreased
Respiratory rate 14-20 20-30 30-40 >35
Urine output >30ml 20-30ml 5-15ml <5ml
Symptoms Normal Anxious Confused Lethargic

Decreasing blood pressure during haemorrhagic shock causes organ hypoperfusion


and relative myocardial ishaemia. The cardiac index gives a numerical value for tissue
oxygen delivery and is given by the equation: Cardiac index= 13.4 - [Hb] - SaO2 +
0.03 PaO2. Where Hb is haemoglobin concentration in blood and SaO2 the saturation
and PaO2 the partial pressure of oxygen. Detailed knowledge of this equation is
required for the MRCS Viva but not for part A, although you should understand the
principle.

In patients suffering from trauma the most likely cause of shock is haemorrhage.
However, the following may also be the cause or occur concomitantly:

 Tension pneumothorax
 Spinal cord injury
 Myocardial contusion
 Cardiac tamponade

When assessing trauma patients it is worth remembering that in order to generate a


palpable femoral pulse an arterial pressure of >65mmHg is required.

Once bleeding is controlled and circulating volume normalised the levels of


transfusion should be to maintain a Hb of 7-8 in those with no risk factors for tissue
hypoxia and Hb 10 for those who have such risk factors.

Neurogenic shock
This occurs most often following a spinal cord transection, usually at a high level.
There is resultant interruption of the autonomic nervous system. The result is either
decreased sympathetic tone or increased parasympathetic tone, the effect of
which is a decrease in peripheral vascular resistance mediated by marked
vasodilation.

This results in decreased preload and thus decreased cardiac output (Starlings law).
There is decreased peripheral tissue perfusion and shock is thus produced. In contrast
with many other types of shock peripheral vasoconstrictors are used to return vascular
tone to normal.

Cardiogenic shock
In medical patients the main cause is ischaemic heart disease. In the traumatic
setting direct myocardial trauma or contusion is more likely. Evidence of ECG
changes and overlying sternal fractures or contusions should raise the suspicion of
injury. Treatment is largely supportive and transthoracic echocardiography should be
used to determine evidence of pericardial fluid or direct myocardial injury. The
measurement of troponin levels in trauma patients may be undertaken but they are
less useful in delineating the extent of myocardial trauma than following MI.

When cardiac injury is of a blunt nature and is associated with cardiogenic shock the
right side of the heart is the most likely site of injury with chamber and or valve
rupture. These patients require surgery to repair these defects and will require
cardiopulmonary bypass to achieve this. Some may require intra aortic balloon pump
as a bridge to surgery.

Anaphylactic shock
Anaphylaxis may be defined as a severe, life-threatening, generalised or systemic
hypersensitivity reaction.

Anaphylaxis is one of the few times when you would not have time to look up the
dose of a medication. The Resuscitation Council guidelines on anaphylaxis have
recently been updated. Adrenaline is by far the most important drug in anaphylaxis
and should be given as soon as possible. The recommended doses for adrenaline,
hydrocortisone and chlorphenamine are as follows:

Adrenaline Hydrocortisone Chlorphenamine


< 6 months 150 mcg (0.15ml 1 in 25 mg 250 mcg/kg
1,000)
6 months - 6 years 150 mcg (0.15ml 1 in 50 mg 2.5 mg
1,000)
6-12 years 300 mcg (0.3ml 1 in 100 mg 5 mg
1,000)
Adult and child 12 500 mcg (0.5ml 1 in 200 mg 10 mg
years 1,000)
Adrenaline can be repeated every 5 minutes if necessary. The best site for IM
injection is the anterolateral aspect of the middle third of the thigh.

Common identified causes of anaphylaxis

 food (e.g. Nuts) - the most common cause in children


 drugs
 venom (e.g. Wasp sting)

Theme: Acid - base disorders

A. pH 7.64 pO2 10.0 kPa pCO2 2.8 kPa HCO3 20

B. pH 7.25 pO2 8.9 pCO2 3.2 HCO3 10

C. pH 7.20 pO2 6.2 pCO2 8.2 HCO3 27

D. pH 7.60 pO2 8.2 pCO2 5.8 HCO3 40

E. pH7.45 pO2 7.2 pCO2 2.5 HCO3 24

Please match the diagnosis with the arterial blood gas result. Each option may be used once,
more than once or not at all.

10. Pulmonary embolus

You answered pH 7.20 pO2 6.2 pCO2 8.2 HCO3 27

The correct answer is pH7.45 pO2 7.2 pCO2 2.5 HCO3 24

A combination of hypoxia and respiratory alkalosis should suggest a pulmonary


embolus. The respiratory alkalosis is due to hyperventilation associated with the
pulmonary embolism.

11. Ureterosigmoidostomy

pH 7.25 pO2 8.9 pCO2 3.2 HCO3 10

There is acidosis. To compensate the patient will attempt to reduce the pH level in
the blood by hyperventilating, hence the low CO2 level .

12. Peptic ulcer causing pyloric stenosis

pH 7.60 pO2 8.2 pCO2 5.8 HCO3 40

Disorders of acid - base balance

Disorders of acid- base balance are often covered in the MRCS part A, both in the SBA and
EMQ sections.

The acid-base normogram below shows how the various disorders may be categorised

Image sourced from Wikipedia

Metabolic acidosis

 This is the most common surgical acid - base disorder.


 Reduction in plasma bicarbonate levels.
 Two mechanisms:

1. Gain of strong acid (e.g. diabetic ketoacidosis)


2. Loss of base (e.g. from bowel in diarrhoea)
- Classified according to the anion gap, this can be calculated by:
(Na+ + K+) - (Cl- + HCO3-).
- If a question supplies the chloride level then this is often a clue that the anion gap should
be calculated. The normal range = 10-18 mmol/L

Normal anion gap ( = hyperchloraemic metabolic acidosis)

 Gastrointestinal bicarbonate loss: diarrhoea, ureterosigmoidostomy, fistula


 Renal tubular acidosis
 Drugs: e.g. acetazolamide
 Ammonium chloride injection
 Addison's disease

Raised anion gap

 Lactate: shock, hypoxia


 Ketones: diabetic ketoacidosis, alcohol
 Urate: renal failure
 Acid poisoning: salicylates, methanol

Metabolic acidosis secondary to high lactate levels may be subdivided into two types:

 Lactic acidosis type A: (Perfusion disorders e.g.shock, hypoxia, burns)


 Lactic acidosis type B: (Metabolic e.g. metformin toxicity)

Metabolic alkalosis

 Usually caused by a rise in plasma bicarbonate levels.


 Rise of bicarbonate above 24 mmol/L will typically result in renal excretion of excess
bicarbonate.
 Caused by a loss of hydrogen ions or a gain of bicarbonate. It is due mainly to
problems of the kidney or gastrointestinal tract

Causes

 Vomiting / aspiration (e.g. Peptic ulcer leading to pyloric stenosis, nasogastric


suction)
 Diuretics
 Liquorice, carbenoxolone
 Hypokalaemia
 Primary hyperaldosteronism
 Cushing's syndrome
 Bartter's syndrome
 Congenital adrenal hyperplasia

Mechanism of metabolic alkalosis

 Activation of renin-angiotensin II-aldosterone (RAA) system is a key factor


 Aldosterone causes reabsorption of Na+ in exchange for H+ in the distal convoluted
tubule
 ECF depletion (vomiting, diuretics) --> Na+ and Cl- loss --> activation of RAA system --
> raised aldosterone levels
 In hypokalaemia, K+ shift from cells --> ECF, alkalosis is caused by shift of H+ into cells
to maintain neutrality

Respiratory acidosis

 Rise in carbon dioxide levels usually as a result of alveolar hypoventilation


 Renal compensation may occur leading to Compensated respiratory acidosis

Causes

 COPD
 Decompensation in other respiratory conditions e.g. Life-threatening asthma /
pulmonary oedema
 Sedative drugs: benzodiazepines, opiate overdose

Respiratory alkalosis

 Hyperventilation resulting in excess loss of carbon dioxide


 This will result in increasing pH

Causes

 Psychogenic: anxiety leading to hyperventilation


 Hypoxia causing a subsequent hyperventilation: pulmonary embolism, high altitude
 Early salicylate poisoning*
 CNS stimulation: stroke, subarachnoid haemorrhage, encephalitis
 Pregnancy
*Salicylate overdose leads to a mixed respiratory alkalosis and metabolic acidosis. Early
stimulation of the respiratory centre leads to a respiratory alkalosis whilst later the direct
acid effects of salicylates (combined with acute renal failure) may lead to an acidosis
Which of the following best accounts for the action of PTH in increasing serum
calcium levels?

A. Activation of vitamin D to increase absorption of calcium from the


small intestine.

B. Direct stimulation of oestoclasts to absorb bone with release of


calcium.

C. Stimulation of phosphate absorption at the distal convoluted tubule of


the kidney.

D. Decreased porosity of the vessels at Bowmans capsule to calcium.

E. Vasospasm of the afferent renal arteriole thereby reducing GFR and


calcium urinary loss.

Theme from April 2012 Exam


PTH increases the activity of 1-α-hydroxylase enzyme, which converts 25-
hydroxycholecalciferol to 1,25-dihydroxycholecalciferol, the active form of vitamin
D.
Osteoclasts do not have a PTH receptor and effects are mediated via osteoblasts.

Parathyroid hormone

Parathyroid hormone is secreted by the chief cells of the parathyroid glands. It acts to
increase serum calcium concentration by stimulation of the PTH receptors in the
kidney and bone. PTH has a plasma half life of 4 minutes.

Effects of PTH
Bone Binds to osteoblasts which signal to osteoclasts to cause resorption of
bone and release calcium
Kidney Active reabsorption of calcium and magnesium from the distal
convoluted tubule. Decreases reabsorption of phosphate.
Intestine via Increases intestinal calcium absorption by increasing activated
kidney vitamin D. Activated vitamin D increases calcium absorption.
Which of the following drugs does not cause syndrome of inappropriate anti diuretic
hormone release?

A. Haloperidol

B. Carbamazepine
C. Amitriptylline

D. Cyclophosphamide

E. Methotrexate
Drugs causing SIADH: ABCD

A nalgesics: opioids, NSAIDs


B arbiturates
C yclophosphamide/
Chlorpromazine/ Carbamazepine
D iuretic (thiazides)

Hyponatraemia

Hyponatraemia

This is commonly tested in the MRCS (despite most surgeons automatically seeking
medical advice if this occurs!). The most common cause in surgery is the over
administration of 5% dextrose.

Hyponatraemia may be caused by water excess or sodium depletion. Causes of


pseudohyponatraemia include hyperlipidaemia (increase in serum volume) or a taking
blood from a drip arm. Urinary sodium and osmolarity levels aid making a diagnosis.

Classification
Urinary sodium > 20 Sodium depletion, renal loss Mnemonic: Syndrome
mmol/l of INAPPropriate Anti-
 Patient often Diuretic Hormone:
hypovolaemic In creased
 Diuretics (thiazides) Na (sodium)
 Addison's PP (urine)
 Diuretic stage of renal
failure
 SIADH (serum osmolality
low, urine osmolality
high, urine Na high)
 Patient often euvolaemic

Urinary sodium < 20 Sodium depletion, extra-renal loss


mmol/l
 Diarrhoea, vomiting,
sweating
 Burns, adenoma of rectum
(if villous lesion and
large)

Water excess (patient  Secondary


often hypervolaemic hyperaldosteronism: CCF,
and oedematous) cirrhosis
 Reduced GFR: renal
failure
 IV dextrose, psychogenic
polydipsia

Management

Symptomatic Hyponatremia :

Acute hyponatraemia with Na <120: immediate therapy. Central Pontine Myelinolisis,


may occur from overly rapid correction of serum sodium. Aim to correct until the Na
is > 125 at a rate of 1 mEq/h. Normal saline with frusemide is an alternative method.

The sodium requirement can be calculated as follows :

(125 - serum sodium) x 0.6 x body weight = required mEq of sodium


Which of the following changes are not typically seen in established dehydration?

A. Rising haematocrit

B. Urinary sodium <20mmol/ litre

C. Metabolic acidosis

D. Decreased serum urea to creatinine ratio

E. Hypernatraemia

Diagnosing dehydration can be complicated, laboratory features include:

 Hypernatraemia
 Rising haematocrit
 Metabolic acidosis
 Rising lactate
 Increased serum urea to creatinine ratio
 Urinary sodium <20 mmol/litre
 Urine osmolality approaching 1200mosmol/kg
Pre operative fluid management

Fluid management has been described in the British Consensus guidelines on IV


fluid therapy for Adult Surgical patients (GIFTASUP)

The Recommendations include:

 Use Ringer's lactate or Hartmann's when a crystalloid is needed for


resuscitation or replacement of fluids. Avoid 0.9% N. Saline (due to risk of
hyperchloraemic acidosis) unless patient vomiting or has gastric drainage.
 Use 0.4%/0.18% dextrose saline or 5% dextrose in maintenance fluids. It
should not be used in resuscitation or as replacement fluids.
 Adult maintenance fluid requirements are: Na 50-100 mmol/day and K 40-80
mmol/day in 1.5-2.5L fluid per day.
 Patients for elective surgery should NOT be nil by mouth for >2 hours (unless
has disorder of gastric emptying).
 Patients for elective surgery should be given carbohydrate rich drinks 2-3h
before. Ideally this should form part of a normal pre op plan to facilitate
recovery.
 Avoid mechanical bowel preparation.
 If bowel prep is used, simultaneous administration of Hartmann's or Ringer's
lactate should be considered.
 Excessive fluid losses from vomiting should be treated with a crystalloid with
potassium replacement. 0.9% N. Saline should be given if there is
hypochloraemia. Otherwise Hartmann's or Ringer lactate should be given for
diarrhoea/ileostomy/ileus/obstruction. Hartmann's should also be given in
sodium losses secondary to diuretics.
 High risk patients should receive fluids and inotropes.
 An attempt should be made to detect pre or operative hypovolaemia using
flow based measurements. If this is not available, then clinical evaluation is
needed i.e. JVP, pulse volume etc.
 In Blood loss or infection causing hypovolaemia should be treated with a
balanced crystalloid or colloid (or until blood available in blood loss). A
critically ill patient is unable to excrete Na or H20 leading to a 5% risk of
interstitial oedema. Therefore 5% dextrose as well as colloid should be given.
 Give 200mls of colloid in hypovolaemia, repeat until clinical parameters
improve.
 A 67 year old male is admitted to the surgical unit with acute abdominal pain.
He is found to have a right sided pneumonia. The nursing staff put him onto
15L O2 via a non rebreathe mask. After 30 minutes the patient is found
moribund, sweaty and agitated by the nursing staff. An arterial blood gas
reveals:
pH 7.15
pCO2 10.2
pO2 8
Bicarbonate 32
Base excess 5.2

What is the most likely cause for this patients deterioration?

A. Acute respiratory alkalosis secondary to hyperventilation

B. Over administration of oxygen in a COPD patient

C. Metabolic acidosis secondary to severe pancreatitis

D. Metabolic alkalosis secondary to hypokalaemia

E. Acute respiratory acidosis secondary to pneumonia



Theme from April 2012 exam
This patient has an acute respiratory acidosis, however this is on a background
of chronic respiratory acidosis (due to COPD) with a compensatory metabolic
alkalosis (the elevated bicarbonate is the main clue to the chronic nature of the
respiratory acidosis). This blood gas picture is typical in a COPD patient who
has received too much oxygen; these patients lose their hypoxic drive for
respiration, therefore retain CO2 and subsequently hypoventilate leading to
respiratory arrest. If the bicarbonate was normal, then the answer would be
acute respiratory acidosis secondary to pneumonia.
 Arterial blood gas interpretation

In advanced life support training, a 5 step approach to arterial blood gas
interpretation is advocated.

1. How is the patient?

2. Is the patient hypoxaemic?


The Pa02 on air should be 10.0-13.0 kPa

3. Is the patient acidaemic (pH <7.35) or alkalaemic (pH >7.45)

4. What has happened to the PaCO2?


If there is acidaemia, an elevated PaCO2 will account for this

5. What is the bicarbonate level or base excess?


A metabolic acidosis will have a low bicarbonate level and a low base excess
(< -2 mmol)
A metabolic alkalosis will have a high bicarbonate and a high base excess (>
+2 mmol)

Which of the following statements relating to the pharmacology of warfarin is untrue?


A. Interferes with clotting factors 2,7,9 and 10

B. It may not be pharmacologically active for up to 72 hours

C. The half life of warfarin is 40 hours

D. Warfarin has a large volume of distribution

E. It is metabolized in the liver


Factors 2,7,9,10
affected

Warfarin interferes with fibrin formation by affecting carboxylation of glutamic acid


residues in factors 2,7,9 and 10. Factor 2 has the longest half life of approximately 60
h, therefore it can take up to 3 days for warfarin to be fully effective. Warfarin has a
small volume of distribution as it is protein bound.

Warfarin

Warfarin is an oral anticoagulant which inhibits the reduction of vitamin K to its


active hydroquinone form, which in turn acts as a cofactor in the formation of clotting
factor II, VII, IX and X (mnemonic = 1972) and protein C

Factors that may potentiate warfarin

 Liver disease
 P450 enzyme inhibitors, e.g.: amiodarone, ciprofloxacin
 Cranberry juice
 Drugs which displace warfarin from plasma albumin, e.g. NSAIDs
 Inhibit platelet function: NSAIDs

Side-effects

 Haemorrhage
 Teratogenic
 Skin necrosis: when warfarin is first started biosynthesis of protein C is
reduced. This results in a temporary procoagulant state after initially starting
warfarin, normally avoided by concurrent heparin administration. Thrombosis
may occur in venules leading to skin necrosis.

Which of the following does not cause an increased anion gap acidosis?

A. Uraemia
B. Paraldehyde

C. Diabetic ketoacidosis

D. Ethylene glycol

E. Acetazolamide

Causes if increased anion acidosis:


MUDPILES

M - Methanol
U - Uraemia
D - DKA/AKA
P - Paraldehyde/phenformin
I - Iron/INH
L - Lactic acidosis
E - Ethylene glycol
S - Salicylates

Disorders of acid - base balance

Disorders of acid- base balance are often covered in the MRCS part A, both in the SBA and
EMQ sections.

The acid-base normogram below shows how the various disorders may be categorised
Image sourced from Wikipedia

Metabolic acidosis

 This is the most common surgical acid - base disorder.


 Reduction in plasma bicarbonate levels.
 Two mechanisms:

1. Gain of strong acid (e.g. diabetic ketoacidosis)


2. Loss of base (e.g. from bowel in diarrhoea)
- Classified according to the anion gap, this can be calculated by:
(Na+ + K+) - (Cl- + HCO3-).
- If a question supplies the chloride level then this is often a clue that the anion gap should
be calculated. The normal range = 10-18 mmol/L

Normal anion gap ( = hyperchloraemic metabolic acidosis)

 Gastrointestinal bicarbonate loss: diarrhoea, ureterosigmoidostomy, fistula


 Renal tubular acidosis
 Drugs: e.g. acetazolamide
 Ammonium chloride injection
 Addison's disease

Raised anion gap


 Lactate: shock, hypoxia
 Ketones: diabetic ketoacidosis, alcohol
 Urate: renal failure
 Acid poisoning: salicylates, methanol

Metabolic acidosis secondary to high lactate levels may be subdivided into two types:

 Lactic acidosis type A: (Perfusion disorders e.g.shock, hypoxia, burns)


 Lactic acidosis type B: (Metabolic e.g. metformin toxicity)

Metabolic alkalosis

 Usually caused by a rise in plasma bicarbonate levels.


 Rise of bicarbonate above 24 mmol/L will typically result in renal excretion of excess
bicarbonate.
 Caused by a loss of hydrogen ions or a gain of bicarbonate. It is due mainly to
problems of the kidney or gastrointestinal tract

Causes

 Vomiting / aspiration (e.g. Peptic ulcer leading to pyloric stenosis, nasogastric


suction)
 Diuretics
 Liquorice, carbenoxolone
 Hypokalaemia
 Primary hyperaldosteronism
 Cushing's syndrome
 Bartter's syndrome
 Congenital adrenal hyperplasia

Mechanism of metabolic alkalosis

 Activation of renin-angiotensin II-aldosterone (RAA) system is a key factor


 Aldosterone causes reabsorption of Na+ in exchange for H+ in the distal convoluted
tubule
 ECF depletion (vomiting, diuretics) --> Na+ and Cl- loss --> activation of RAA system --
> raised aldosterone levels
 In hypokalaemia, K+ shift from cells --> ECF, alkalosis is caused by shift of H+ into cells
to maintain neutrality

Respiratory acidosis
 Rise in carbon dioxide levels usually as a result of alveolar hypoventilation
 Renal compensation may occur leading to Compensated respiratory acidosis

Causes

 COPD
 Decompensation in other respiratory conditions e.g. Life-threatening asthma /
pulmonary oedema
 Sedative drugs: benzodiazepines, opiate overdose

Respiratory alkalosis

 Hyperventilation resulting in excess loss of carbon dioxide


 This will result in increasing pH

Causes

 Psychogenic: anxiety leading to hyperventilation


 Hypoxia causing a subsequent hyperventilation: pulmonary embolism, high altitude
 Early salicylate poisoning*
 CNS stimulation: stroke, subarachnoid haemorrhage, encephalitis
 Pregnancy

*Salicylate overdose leads to a mixed respiratory alkalosis and metabolic acidosis. Early
stimulation of the respiratory centre leads to a respiratory alkalosis whilst later the direct
acid effects of salicylates (combined with acute renal failure) may lead to an acidosis
Which one of the following is least associated with thrombocytopenia?

A. Heparin therapy

B. Rheumatoid arthritis

C. Infectious mononucleosis

D. Liver disease

E. Pregnancy

Rheumatoid arthritis, unlike systemic lupus erythematous, is generally associated with


a thrombocytosis. In some cases of Felty's syndrome thrombocytopaenia may be seen
secondary to hypersplenism. This however represents a small percentage of patients
with rheumatoid arthritis.

Thrombocytopenia

Causes of severe thrombocytopenia

 ITP
 DIC
 TTP
 haematological malignancy

Causes of moderate thrombocytopenia

 heparin induced thrombocytopenia (HIT)


 drug-induced (e.g. quinine, diuretics, sulphonamides, aspirin, thiazides)
 alcohol
 liver disease
 hypersplenism
 viral infection (EBV, HIV, hepatitis)
 pregnancy
 SLE/antiphospholipid syndrome
 vitamin B12 deficiency

hich of the following will increase the volume of pancreatic exocrine secretions?

A. Octreotide

B. Cholecystokinin

C. Aldosterone

D. Adrenaline

E. None of the above

Cholecystokinin will often increase the volume of pancreatic secretions.

Pancreas exocrine physiology

Pancreatic juice

 Alkaline solution pH 8
 1500ml/day
 Composition: acinar secretion (ENZYMES: trypsinogen, procarboxylase,
amylase, lecithin) and ductile secretion (HCO, Na+, water)
 Pancreatic juice action: Trypsinogen is converted via enterokinase to active
trypsin in the duodenum. Trypsin then activates the other inactive enzymes.

Where is the majority of iron found in the body?

A. Bone

B. Haemoglobin

C. Ferritin and haemosiderin

D. Myoglobin

E. Plasma iron

Approximately 70% of body iron is found bound to haemoglobin.

Iron metabolism

Absorption  Duodenum and upper jejunum


 About 10% of dietary iron absorbed
 Fe2+ (ferrous iron) much better absorbed than Fe3+ (ferric iron)
 Ferrous iron is oxidized to form ferric iron, which is combined
with apoferritin to form ferritin
 Absorption is regulated according to body's need
 Increased by vitamin C, gastric acid
 Decreased by proton pump inhibitors, tetracycline, gastric
achlorhydria, tannin (found in tea)

Transport In plasma as Fe3+ bound to transferrin


Storage Ferritin (or haemosiderin) in bone marrow
Excretion Lost via intestinal tract following desquamation

Distribution in body
Total body iron 4g
Haemoglobin 70%
Ferritin and haemosiderin 25%
4%
Myoglobin
Plasma iron 0.1%

A 44 year old man recieves a large volume transfusion of whole blood. The whole blood is
two weeks old. Which of the following best describes its handling of oxygen?
A. It will have a low affinity for oxygen

B. Its affinity for oxygen is unchanged

C. It will more readily release oxygen in metabolically active tissues than


fresh blood

D. The release of oxygen in metabolically acitve tissues will be the same as


fresh blood

E. It will have an increased affinity for oxygen

Stored blood has less 2,3 DPG and therefore has a higher affinity for oxygen, this reduces its
ability to release it at metabolising tissues.

Oxygen Transport

Oxygen transport
Almost all oxygen is transported within erythrocytes. It has limited solubility and only 1% is
carried as solution. Therefore the amount of oxygen transported will depend upon
haemoglobin concentration and its degree of saturation.

Haemoglobin
Globular protein composed of 4 subunits. Haem consists of a protoporphyrin ring
surrounding an iron atom in its ferrous state. The iron can form two additional bonds; one
with oxygen and the other with a polypeptide chain. There are two alpha and two beta
subunits to this polypeptide chain in an adult and together these form globin. Globin cannot
bind oxygen but is able to bind to carbon dioxide and hydrogen ions, the beta chains are
able to bind to 2,3 diphosphoglycerate. The oxygenation of haemoglobin is a reversible
reaction. The molecular shape of haemoglobin is such that binding of one oxygen molecule
facilitates the binding of subsequent molecules.

Oxygen dissociation curve

 The oxygen dissociation curve describes the relationship between the percentage of
saturated haemoglobin and partial pressure of oxygen in the blood. It is not affected
by haemoglobin concentration.
 Chronic anaemia causes 2, 3 DPG levels to increase, hence shifting the curve to the
right
Bohr effect

 Shifts to left = for given oxygen tension there is increased saturation of Hb with
oxygen i.e. Decreased oxygen delivery to tissues
 Shifts to right = for given oxygen tension there is reduced saturation of Hb with
oxygen i.e. Enhanced oxygen delivery to tissues

Image sourced from Wikipedia

Shifts to Left = Lower oxygen delivery Shifts to Right = Raised oxygen


delivery
 HbF, methaemoglobin, carboxyhaemoglobin
 low [H+] (alkali)  raised [H+] (acidic)
 low pCO2  raised pCO2
 low 2,3-DPG  raised 2,3-DPG*
 low temperature  raised temperature

*2,3-diphosphoglycerate

Which of the following does not occur during the physiological response to surgery?
A. Glycogenolysis

B. Increased acute phase proteins

C. Increased cortisol production

D. Bronchoconstriction

E. Release of nitric oxide by vessels

Response to surgery

Sympathetic nervous system

 Noradrenaline from sympathetic nerves and adrenaline from adrenal medulla


 Blood diverted from skin and visceral organs; bronchodilatation, reduced
intestinal motility, increased glucagon and glycogenolysis, insulin reduced
 Heart rate and myocardial contractility are increased

Acute phase response

 TNF-α, IL-1, IL-2, IL-6, interferon and prostaglandins are released


 Excess cytokines may cause SIRS
 Cytokines increase the release of acute phase proteins

Endocrine response

 Hypothalamus, pituitary, adrenal axis


 Increases ACTH and cortisol production:

increases protein breakdown


increases blood glucose levels

 Aldosterone increases sodium reabsorption


 Vasopressin increases water reabsorption and causes vasoconstriction

Vascular endothelium

 Nitric oxide produces vasodilatation


 Platelet activating factor enhances the cytokine response
 Prostaglandins produce vasodilatation and induce platelet aggregation
A 43 year old lady undergoes a day case laparoscopic cholecystectomy. The operation is
more difficult than anticipated and a drain is placed to the operative site. Whilst in recovery
the patient loses 1800ml of frank blood into the drain. Which of the following will not occur?

A. Release of aldosterone via the Bainbridge reflex

B. Reduced urinary sodium excretion

C. Increase in sympathetic discharge to ventricular muscle

D. Fall in parasympathetic discharge to the sino atrial node

E. Decreased stimulation from atrial pressure receptors

The Bainbridge reflex is the increase in heart rate mediated via atrial stretch receptors that
occurs following a rapid infusion of blood.

Cardiac physiology

 The heart has four chambers ejecting blood into both low pressure and high
pressure systems.
 The pumps generate pressures of between 0-25mmHg on the right side and 0-120
mmHg on the left.
 At rest diastole comprises 2/3 of the cardiac cycle.
 The product of the frequency of heart rate and stroke volume combine to give the
cardiac output which is typically 5-6L per minute.

Detailed descriptions of the various waveforms are often not a feature of MRCS A (although
they are on the syllabus). However, they are a very popular topic for surgical physiology
vivas in the oral examination.

Electrical properties

 Intrinsic myogenic rhythm within cardiac myocytes means that even the denervated
heart is capable of contraction.
 In the normal situation the cardiac impulse is generated in the sino atrial node in the
right atrium and conveyed to the ventricles via the atrioventricular node.
 The sino atrial node is also capable of spontaneous discharge and in the absence of
background vagal tone will typically discharge around 100x per minute. Hence the
higher resting heart rate found in cardiac transplant cases. In the SA and AV nodes
the resting membrane potential is lower than in surrounding cardiac cells and will
slowly depolarise from -70mV to around -50mV at which point an action potential is
generated.
 Differences in the depolarisation slopes between SA and AV nodes help to explain
why the SA node will depolarise first. The cells have a refractory period during which
they cannot be re-stimulated and this period allows for adequate ventricular filling.
In pathological tachycardic states this time period is overridden and inadequate
ventricular filling may then occur, cardiac output falls and syncope may ensue.

Parasympathetic fibres project to the heart via the vagus and will release acetylcholine.
Sympathetic fibres release nor adrenaline and circulating adrenaline comes from the adrenal
medulla. Noradrenaline binds to β 1 receptors in the SA node and increases the rate of
pacemaker potential depolarisation.

Cardiac cycle

Image sourced from Wikipedia

 Mid diastole: AV valves open. Ventricles hold 80% of final volume. Outflow valves
shut. Aortic pressure is high.

 Late diastole: Atria contract. Ventricles receive 20% to complete filling. Typical end
diastolic volume 130-160ml.
 Early systole: AV valves shut. Ventricular pressure rises. Isovolumetric ventricular
contraction. AV Valves bulge into atria (c-wave). Aortic and pulmonary pressure
exceeded- blood is ejected. Shortening of ventricles pulls atria downwards and
drops intra atrial pressure (x-descent).

 Late systole: Ventricular muscles relax and ventricular pressures drop. Although
ventricular pressure drops the aortic pressure remains constant owing to peripheral
vascular resistance and elastic property of the aorta. Brief period of retrograde flow
that occurs in aortic recoil shuts the aortic valve. Ventricles will contain 60ml end
systolic volume. The average stroke volume is 70ml (i.e. Volume ejected).

 Early diastole: All valves are closed. Isovolumetric ventricular relaxation occurs.
Pressure wave associated with closure of the aortic valve increases aortic pressure.
The pressure dip before this rise can be seen on arterial waveforms and is called the
incisura. During systole the atrial pressure increases such that it is now above zero
(v- wave). Eventually atrial pressure exceed ventricular pressure and AV valves open
- atria empty passively into ventricles and atrial pressure falls (y -descent )

The negative atrial pressures are of clinical importance as they can allow air embolization to
occur if the neck veins are exposed to air. This patient positioning is important in head and
neck surgery to avoid this occurrence if veins are inadvertently cut, or during CVP line
insertion.

Mechanical properties

 Preload = end diastolic volume


 Afterload = aortic pressure

It is important to understand the principles of Laplace's law in surgery.

 It states that for hollow organs with a circular cross section, the total circumferential
wall tension depends upon the circumference of the wall, multiplied by the
thickness of the wall and on the wall tension.
 The total luminal pressure depends upon the cross sectional area of the lumen and
the transmural pressure. Transmural pressure is the internal pressure minus
external pressure and at equilibrium the total pressure must counterbalance each
other.
 In terms of cardiac physiology the law explains that the rise in ventricular pressure
that occurs during the ejection phase is due to physical change in heart size. It also
explains why a dilated diseased heart will have impaired systolic function.

Starlings law
 Increase in end diastolic volume will produce larger stroke volume.
 This occurs up to a point beyond which cardiac fibres are excessively stretched and
stroke volume will fall once more. It is important for the regulation of cardiac output
in cardiac transplant patients who need to increase their cardiac output.

Baroreceptor reflexes

 Baroreceptors located in aortic arch and carotid sinus.


 Aortic baroreceptor impulses travel via the vagus and from the carotid via the
glossopharyngeal nerve.
 They are stimulated by arterial stretch.
 Even at normal blood pressures they are tonically active.
 Increase in baroreceptor discharge causes:

*Increased parasympathetic discharge to the SA node.


*Decreased sympathetic discharge to ventricular muscle causing decreased contractility and
fall in stroke volume.
*Decreased sympathetic discharge to venous system causing increased compliance.
*Decreased peripheral arterial vascular resistance

Atrial stretch receptors

 Located in atria at junction between pulmonary veins and vena cava.


 Stimulated by atrial stretch and are thus low pressure sensors.
 Increased blood volume will cause increased parasympathetic activity.
 Very rapid infusion of blood will result in increase in heart rate mediated via atrial
receptors: the Bainbridge reflex.
 Decreases in receptor stimulation results in increased sympathetic activity this will
decrease renal blood flow-decreases GFR-decreases urinary sodium excretion-renin
secretion by juxtaglomerular apparatus-Increase in angiotensin II.
 Increased atrial stretch will also result in increased release of atrial natriuretic
peptide.

Which of the following statements are not typically true in hypokalaemia?

A. It may occur as a result of mechanical bowel preparation

B. Chronic vomiting may increase renal potassium losses

C. It may be associated with aciduria

D. It may cause hyponatraemia

E. It often accompanies acidosis


Potassium depletion occurs either through the gastrointestinal tract or the kidney.
Chronic vomiting in itself is less prone to induce potassium loss than diarrhoea as
gastric secretions contain less potassium than those in the lower GI tract. If vomiting
produces a metabolic alkalosis then renal potassium wasting may occur as potassium
is excreted in preference to hydrogen ions. The converse may occur in potassium
depletion resulting in acid urine.

Hyperkalaemia

 Plasma potassium levels are regulated by a number of factors including


aldosterone, acid-base balance and insulin levels.
 Metabolic acidosis is associated with hyperkalaemia as hydrogen and
potassium ions compete with each other for exchange with sodium ions across
cell membranes and in the distal tubule.
 ECG changes seen in hyperkalaemia include tall-tented T waves, small P
waves, widened QRS leading to a sinusoidal pattern and asystole

Causes of hyperkalaemia

 Acute renal failure


 Drugs*: potassium sparing diuretics, ACE inhibitors, angiotensin 2 receptor
blockers, spironolactone, ciclosporin, heparin**
 Metabolic acidosis
 Addison's
 Tissue necrosis/rhabdomylosis: burns, trauma
 Massive blood transfusion

Foods that are high in potassium

 Salt substitutes (i.e. Contain potassium rather than sodium)


 Bananas, oranges, kiwi fruit, avocado, spinach, tomatoes

*beta-blockers interfere with potassium transport into cells and can potentially cause
hyperkalaemia in renal failure patients - remember beta-agonists, e.g. Salbutamol, are
sometimes used as emergency treatment

**both unfractionated and low-molecular weight heparin can cause hyperkalaemia.


This is thought to be caused by inhibition of aldosterone secretion
Approximately what proportion of salivary secretions is provided by the
submandibular glands?

A. 10%
B. 70%

C. 40%

D. 90%

E. 20%

Although they are small, the submandibular glands provide the bulk of salivary
secretions and contribute 70%, the sublingual glands provide 5% and the remainder
from the parotid.

Submandibular gland

Relations of the submandibular gland


Superficial Platysma, deep fascia and mandible
Submandibular lymph nodes
Facial vein (facial artery near mandible)
Marginal mandibular nerve
Cervical branch of the facial nerve
Deep Facial artery (inferior to the mandible)
Mylohoid muscle
Sub mandibular duct
Hyoglossus muscle
Lingual nerve
Submandibular ganglion
Hypoglossal nerve

Submandibular duct (Wharton's duct)

 Opens lateral to the lingual frenulum on the anterior floor of mouth.


 5 cm length
 Lingual nerve wraps around Wharton's duct. As the duct passes forwards it
crosses medial to the nerve to lie above it and then crosses back, lateral to it, to
reach a position below the nerve.

Innervation

 Sympathetic innervation- Superior Cervical ganglion via the Lingual nerve


 Parasympathetic innervation- Submandibular ganglion

Arterial supply
Branch of the Facial artery. The facial artery passes through the gland to groove its
deep surface. It then emerges onto the face by passing between the gland and the
mandible.
Venous drainage
Anterior Facial vein (lies deep to the Marginal Mandibular nerve)

Lymphatic drainage
Deep cervical and jugular chains of nodes
Which is not a cause of hyperuricaemia?

A. Severe psoriasis

B. Lesch-Nyhan syndrome

C. Hyperthyroidism

D. Diabetic ketoacidosis

E. Alcohol
Mnemonic of the drugs causing hyperuricaemia as a result of reduced excretion of
urate

'Can't leap'

C iclosporin
A lcohol
N icotinic acid
T hiazides

L oop diuretics
E thambutol
A spirin
P yrazinamide

Hyperuricaemia

 Increased levels of uric acid may be seen secondary to either increased cell
turnover or reduced renal excretion of uric acid. Hyperuricaemia may be found
in asymptomatic patients who have not experienced attacks of gout

 Hyperuricaemia may be associated with hyperlipidaemia and hypertension. It


may also be seen in conjunction with the metabolic syndrome
Increased synthesis

 Lesch-Nyhan disease
 Myeloproliferative disorders
 Diet rich in purines
 Exercise
 Psoriasis
 Cytotoxics

Decreased excretion

 Drugs: low-dose aspirin, diuretics, pyrazinamide


 Pre-eclampsia
 Alcohol
 Renal failure
 Lead

A 20 year old man is hit in the face and occludes his airway. Which of the following
stimuli and receptor groups would the most potently activated as a result?

A. Receptors in the apneustic centre

B. Hypoxia of centrally located chemoreceptors

C. Hypoxia of peripherally located chemoreceptors

D. Hypercapnia of the carotid bodies

E. Hypoxia in the aortic arch receptors

The carotid bodies are the most vascular site and hypercapnia the most potent
stimulus.

Control of ventilation

 Control of ventilation is coordinated by the respiratory centres,


chemoreceptors, lung receptors and muscles.
 Automatic, involuntary control of respiration occurs from the medulla.
 The respiratory centres control the respiratory rate and the depth of respiration.

Respiratory centres
 Medullary respiratory centre:

Inspiratory and expiratory neurones. Has ventral group which controls forced
voluntary expiration and the dorsal group controls inspiration. Depressed by opiates.

 Apneustic centre:

Lower pons
Stimulates inspiration - activates and prolongs inhalation
Overridden by pneumotaxic control to end inspiration

 Pneumotaxic centre:

Upper pons, inhibits inspiration at a certain point. Fine tunes the respiratory rate.

 Levels of PCO2 most important in ventilation control


 Levels of O2 are less important.
 Peripheral chemoreceptors: located in the bifurcation of carotid arteries and
arch of the aorta. They respond to changes in reduced pO2, increased H+ and
increased pCO2 in ARTERIAL BLOOD.
 Central chemoreceptors: located in the medulla. Respond to increased H+ in
BRAIN INTERSTITIAL FLUID to increase ventilation. NB the central
receptors are NOT influenced by O2 levels.

Lung receptors include:

 Stretch receptors: respond to lung stretching causing a reduced respiratory rate


 Irritant receptors: respond to smoke etc causing bronchospasm
 J (juxtacapillary) receptors

Which of the following statements relating to low molecular weight heparins is false?

A. They act via inhibition of Factor Xa

B. Large doses may be used prior to commencing cardiopulmonary


bypass

C. They have a highly predictable pharmacokinetic profile

D. They are derivatives of unfractionated heparin

E. They have a molecular mass in the range of 3000-10000Da


As they are not easily reversed they are unsuitable for this purpose.

Heparin

 Causes the formation of complexes between antithrombin and activated


thrombin/factors 7,9,10,11 & 12

Advantages of low molecular weight heparin

 Better bioavailability
 Lower risk of bleeding
 Longer half life
 Little effect on APTT at prophylactic dosages
 Less risk of HIT

Complications

 Bleeding
 Osteoporosis
 Heparin induced thrombocytopenia (HIT): occurs 5-14 days after 1st exposure
 Anaphylaxis

In surgical patients that may need a rapid return to theatre administration of


unfractionated heparin is preferred as low molecular weight heparins have a longer
duration of action and are harder to reverse.
A 43 year old lady presents with urinary incontinence. At which of the following
locations is Onufs nucleus likely to be found?

A. Medulla oblongata

B. Anterior horn of L5 nerve roots

C. Micturition centre in the Pons

D. Anterior horn of S2 nerve roots

E. None of the above

Onufs nucleus is located in the anterior horn of S2 and is the origin of neurones to the
external urethral sphincter.

Urinary incontinence
Urinary incontinence

Involuntary passage of urine. Most cases are female (80%). It has a prevalence of
11% in those aged greater than 65 years. The commonest variants include:

 Stress urinary incontinence (50%)


 Urge incontinence (15%)
 Mixed (35%)

Males
Males may also suffer from incontinence although it is a much rarer condition in men.
A number of anatomical factors contribute to this. Males have 2 powerful sphincters;
one at the bladder neck and the other in the urethra. Damage to the bladder neck
mechanism is a factor in causing retrograde ejaculation following prostatectomy. The
short segment of urethra passing through the urogenital diaphragm consists of striated
muscle fibres (the external urethral sphincter) and smooth muscle capable of more
sustained contraction. It is the latter mechanism that maintains continence following
prostatectomy.

Females
The sphincter complex at the level of bladder neck is poorly developed in females. As
a result the external sphincter complex is functionally more important, its composition
being similar to that of males. Innervation is via the pudendal nerve and the
neuropathy that may accompany obstetric events may compromise this and lead to
stress urinary incontinence.

Innervation
Somatic innervation to the bladder is via the pudendal, hypogastric and pelvic nerves.
Autonomic nerves travel in these nerve fibres too. Bladder filling leads to detrusor
relaxation (sympathetic) coupled with sphincter contraction. The parasympathetic
system causes detrusor contraction and sphincter relaxation. Overall control of
micturition is centrally mediated via centres in the Pons.

Stress urinary incontinence

 50% of cases, especially in females.


 Damage (often obstetric) to the supporting structures surrounding the bladder
may lead to urethral hypermobility.
 Other cases due to sphincter dysfunction, usually from neurological disorders
(e.g. Pudendal neuropathy, multiple sclerosis).

Urethral mobility:
Pressure not transmitted appropriately to the urethra resulting in involuntary passage
of urine during episodes of raised intra-abdominal pressure.

Sphincter dysfunction:
Sphincter fails to adapt to compress urethra resulting in involuntary passage of urine.
When the sphincter completely fails there is often to continuous passage of urine.

Urge incontinence
In these patients there is sense of urgency followed by incontinence. The detrusor
muscle in these patients is unstable and urodynamic investigation will demonstrate
overactivity of the detrusor muscle at inappropriate times (e.g. Bladder filling).
Urgency may be seen in patients with overt neurological disorders and those without.
The pathophysiology is not well understood but poor central and peripheral co-
ordination of the events surrounding bladder filling are the main processes.

Assessment
Careful history and examination including vaginal examination for cystocele.
Bladder diary for at least 3 days
Consider flow cystometry if unclear symptomatology or surgery considered and
diagnosis is unclear.
Exclusion of other organic disease (e.g. Stones, UTI, Cancer)

Management
Conservative measures should be tried first; Stress urinary incontinence or mixed
symptoms should undergo 3 months of pelvic floor exercise. Over active bladder
should have 6 weeks of bladder retraining.
Drug therapy for women with overactive bladder should be offered with oxybutynin if
conservative measures fail.
In women with detrusor instability who fail non operative therapy a trial of sacral
neuromodulation may be considered, with conversion to permanent implant if good
response. Augmentation cystoplasty is an alternative but will involve long term
intermittent self catheterisation.
In women with stress urinary incontinence a urethral sling type procedure may be
undertaken. Where cystocele is present in association with incontinence it should be
repaired particularly if it lies at the introitus.

NICE guidelines

 Initial assessment urinary incontinence should be classified as


stress/urge/mixed.
 At least 3/7 bladder diary if unable to classify easily.
 Start conservative treatment before urodynamic studies if a diagnosis is
obvious from the history
 Urodynamic studies if plans for surgery.
 Stress incontinence: Pelvic floor exercises 3/12, if fails consider surgery.
 Urge incontinence: Bladder training >6/52, if fails for oxybutynin
(antimuscarinic drugs) then sacral nerve stimulation.
 Pelvic floor exercises offered to all women in their 1st pregnancy.

A 43 year old lady presents with urinary incontinence. At which of the following
locations is Onufs nucleus likely to be found?

A. Medulla oblongata
B. Anterior horn of L5 nerve roots

C. Micturition centre in the Pons

D. Anterior horn of S2 nerve roots

E. None of the above

Onufs nucleus is located in the anterior horn of S2 and is the origin of neurones to the
external urethral sphincter.

Urinary incontinence

Urinary incontinence

Involuntary passage of urine. Most cases are female (80%). It has a prevalence of
11% in those aged greater than 65 years. The commonest variants include:

 Stress urinary incontinence (50%)


 Urge incontinence (15%)
 Mixed (35%)

Males
Males may also suffer from incontinence although it is a much rarer condition in men.
A number of anatomical factors contribute to this. Males have 2 powerful sphincters;
one at the bladder neck and the other in the urethra. Damage to the bladder neck
mechanism is a factor in causing retrograde ejaculation following prostatectomy. The
short segment of urethra passing through the urogenital diaphragm consists of striated
muscle fibres (the external urethral sphincter) and smooth muscle capable of more
sustained contraction. It is the latter mechanism that maintains continence following
prostatectomy.

Females
The sphincter complex at the level of bladder neck is poorly developed in females. As
a result the external sphincter complex is functionally more important, its composition
being similar to that of males. Innervation is via the pudendal nerve and the
neuropathy that may accompany obstetric events may compromise this and lead to
stress urinary incontinence.

Innervation
Somatic innervation to the bladder is via the pudendal, hypogastric and pelvic nerves.
Autonomic nerves travel in these nerve fibres too. Bladder filling leads to detrusor
relaxation (sympathetic) coupled with sphincter contraction. The parasympathetic
system causes detrusor contraction and sphincter relaxation. Overall control of
micturition is centrally mediated via centres in the Pons.

Stress urinary incontinence


 50% of cases, especially in females.
 Damage (often obstetric) to the supporting structures surrounding the bladder
may lead to urethral hypermobility.
 Other cases due to sphincter dysfunction, usually from neurological disorders
(e.g. Pudendal neuropathy, multiple sclerosis).

Urethral mobility:
Pressure not transmitted appropriately to the urethra resulting in involuntary passage
of urine during episodes of raised intra-abdominal pressure.

Sphincter dysfunction:
Sphincter fails to adapt to compress urethra resulting in involuntary passage of urine.
When the sphincter completely fails there is often to continuous passage of urine.

Urge incontinence
In these patients there is sense of urgency followed by incontinence. The detrusor
muscle in these patients is unstable and urodynamic investigation will demonstrate
overactivity of the detrusor muscle at inappropriate times (e.g. Bladder filling).
Urgency may be seen in patients with overt neurological disorders and those without.
The pathophysiology is not well understood but poor central and peripheral co-
ordination of the events surrounding bladder filling are the main processes.

Assessment
Careful history and examination including vaginal examination for cystocele.
Bladder diary for at least 3 days
Consider flow cystometry if unclear symptomatology or surgery considered and
diagnosis is unclear.
Exclusion of other organic disease (e.g. Stones, UTI, Cancer)

Management
Conservative measures should be tried first; Stress urinary incontinence or mixed
symptoms should undergo 3 months of pelvic floor exercise. Over active bladder
should have 6 weeks of bladder retraining.
Drug therapy for women with overactive bladder should be offered with oxybutynin if
conservative measures fail.
In women with detrusor instability who fail non operative therapy a trial of sacral
neuromodulation may be considered, with conversion to permanent implant if good
response. Augmentation cystoplasty is an alternative but will involve long term
intermittent self catheterisation.
In women with stress urinary incontinence a urethral sling type procedure may be
undertaken. Where cystocele is present in association with incontinence it should be
repaired particularly if it lies at the introitus.

NICE guidelines

 Initial assessment urinary incontinence should be classified as


stress/urge/mixed.
 At least 3/7 bladder diary if unable to classify easily.
 Start conservative treatment before urodynamic studies if a diagnosis is
obvious from the history
 Urodynamic studies if plans for surgery.
 Stress incontinence: Pelvic floor exercises 3/12, if fails consider surgery.
 Urge incontinence: Bladder training >6/52, if fails for oxybutynin
(antimuscarinic drugs) then sacral nerve stimulation.
 Pelvic floor exercises offered to all women in their 1st pregnancy.

Which of the following is associated with reduced lung compliance?

A. Older age

B. Emphysematous type COPD

C. Decline in pulmonary blood flow

D. Adopting a vertical posture

E. Adjusting a ventilator to maintain high lung volumes

Increased lung compliance =


Older age, COPD

Lung compliance is a measure of the ease of expansion of the lungs and thorax, determined
by pulmonary volume and elasticity. A high degree of compliance indicates a loss of elastic
recoil of the lungs, as in old age or emphysema. This increased lung compliance is due to loss
of supportive tissue around the airways. While a normal lung has a high passive elastic
recoil, the sick lung has a decreased elasticity (i.e. decreased transpulmonary pressure)
which leads to increased lung compliance.

Decreased compliance means that a greater change in pressure is needed for a given change
in volume, as in atelectasis, pulmonary fibrosis, pneumonia, or lack of surfactant.

Lung volumes

The diagram demonstrates lung volumes and capacities


Image sourced from Wikipedia

Definitions

Tidal volume (TV)  Is the volume of air inspired and expired during each
ventilatory cycle at rest.
 It is normally 500mls in males and 340mls in females.

Inspiratory reserve  Is the maximum volume of air that can be forcibly inhaled
volume (IRV) following a normal inspiration. 3000mls.

Expiratory reserve  Is the maximum volume of air that can be forcibly exhaled
volume (ERV) following a normal expiration. 1000mls.

Residual volume (RV)  Is that volume of air remaining in the lungs after a maximal
expiration.
 RV = FRC - ERV. 1500mls.

Functional residual  Is the volume of air remaining in the lungs at the end of a
capacity (FRC) normal expiration.
 FRC = RV + ERV. 2500mls.

Vital capacity (VC)  Is the maximal volume of air that can be forcibly exhaled
after a maximal inspiration.
 VC = TV + IRV + ERV. 4500mls in males, 3500mls in females.

Total lung capacity  Is the volume of air in the lungs at the end of a maximal
(TLC) inspiration.
 TLC = FRC + TV + IRV = VC + RV. 5500-6000mls.

Forced vital capacity  The volume of air that can be maximally forcefully exhaled.
(FVC)
Which of the following statements relating to alveolar ventilation is untrue?

A. Anatomical dead space is measured by helium dilution

B. Physiological dead space is increased in PE

C. Alveolar ventilation is defined as the volume of fresh air entering the


alveoli per minute

D. Anatomical dead space is increased by noradrenaline

E. Type 2 pneumocytes in the alveoli secrete surfactant

Anatomical dead space is measured by Fowlers method.

A patient inhales 100% oxygen to empty the conducting zone gases of nitrogen and
then exhales through a mouthpiece which analyses the nitrogen concentration at the
mouth. Initially the exhaled gases contain no nitrogen as this is dead space gas; the
nitrogen concentration will increase
as the alveolar gases are exhaled. Nitrogen which is measured following the breath of
100% oxygen must then have come only from gas exchanging areas of the lung and
not dead space.

Alveolar ventilation

 Minute ventilation is the total volume of gas ventilated per minute.

MV (ml/min)= tidal volume x Respiratory rate (resps/min).

 Dead space ventilation describes the volume of gas not involved in exchange
in the blood.

There are 2 types:

1. Anatomical dead space: 150mls

 Volume of gas in the respiratory tree not involved in gaseous exchange:


mouth, pharynx, trachea, bronchi up to terminal bronchioles
 Measured by Fowlers method
 Increased by:
 Standing, increased size of person, increased lung volume and drugs causing
bronchodilatation e.g. Adrenaline
2. Physiological dead space: normal 150 mls, increases in ventilation/perfusion
mismatch e.g. PE, COPD, hypotension

 Volume of gas in the alveoli and anatomical dead space not involved in
gaseous exchange.

 Alveolar ventilation is the volume of fresh air entering the alveoli per minute.

Alveolar ventilation = minute ventilation - Dead space volume


Which opioid receptor does morphine attach to?

A. mu

B. alpha

C. sigma

D. beta

E. kappa

Pethidine and other conventional opioids attach to this receptor.

Opioids

- Combine to specific opiate receptors in the CNS (periaqueductal grey matter, limbic
system, substantia gelatinosa)

- Morphine attaches to mu1 receptors


Which of the following inhibits the secretion of insulin?

A. Adrenaline

B. Glucagon

C. Gastrin

D. Arginine

E. Vagal cholinergic activity


Inhibition of insulin release:

 Alpha adrenergic drugs


 Beta blockers
 Sympathetic nerves

Insulin

 Anabolic hormone

Structure

 and chain linked by disulphide bridges

Synthesis

 Pro-insulin is formed by the rough endoplasmic reticulum in pancreatic beta


cells. Then pro-insulin is cleaved to form insulin and C-peptide. Insulin is
stored in secretory granules and released in response to Ca.

Function

 Secreted in response to hyperglycaemia


 Glucose utilisation and glycogen synthesis
 Inhibits lipolysis
 Reduces muscle protein loss

What is measured to obtain renal plasma flow?

A. Creatinine

B. Para-amino hippuric acid (PAH)

C. Inulin

D. Glucose

E. Protein

Renal plasma flow = (amount of PAH in urine per unit time) / (difference in PAH
concentration in the renal artery or vein)

Normal value = 660ml/min

Renal Physiology

Overview

 Each nephron is supplied with blood from an afferent arteriole that opens onto
the glomerular capillary bed.
 Blood then flows to an efferent arteriole, supplying the peritubular capillaries
and medullary vasa recta.
 The kidney receives up to 25% of resting cardiac output.

Control of blood flow

 The kidney is able to autoregulate its blood flow between systolic pressures of
80- 180mmHg so there is little variation in renal blood flow.
 This is achieved by myogenic control of arteriolar tone, both sympathetic
input and hormonal signals (e.g. renin) are responsible.

Glomerular structure and function

 Blood inside the glomerulus has considerable hydrostatic pressure.


 The basement membrane has pores that will allow free diffusion of smaller
solutes, larger negatively charged molecules such as albumin are unable to
cross.
 The glomerular filtration rate (GFR) is equal to the concentration of a solute in
the urine, times the volume of urine produced per minute, divided by the
plasma concentration (assuming that the solute is freely diffused e.g. inulin).
 In clinical practice creatinine is used because it is subjected to very little
proximal tubular secretion.
 Although subject to variability, the typical GFR is 125ml per minute.

 Glomerular filtration rate = Total volume of plasma per unit time leaving the
capillaries and entering the bowman's capsule

 Renal clearance = volume plasma from which a substance is removed per


minute by the kidneys

 Substances used to measure GFR have the following features:


1. Inert
2. Free filtration from the plasma at the glomerulus (not protein bound)
3. Not absorbed or secreted at the tubules
4. Plasma concentration constant during urine collection

Examples: inulin, creatinine

GFR = urine concentration (mmol/l) x urine volume (ml/min)


--------------------------------------------------------------------------
plasma concentration (mmol/l)

 The clearance of a substance is dependent not only on its diffusivity across the
basement membrane but also subsequent tubular secretion and / or
reabsorption.
 So glucose which is freely filtered across the basement membrane is usually
reabsorbed from tubules giving a clearance of zero.

Tubular function

 Reabsorption and secretion of substances occurs in the tubules.


 In the proximal tubule substrates such as glucose, amino acids and phosphate
are co-transported with sodium across the semi permeable membrane.
 Up to two thirds of filtered water is reabsorbed in the proximal tubules.
 This will lead to increase in urea concentration in the distal tubule allowing for
its increased diffusion.
 Substances to be secreted into the tubules are taken up from the peritubular
blood by tubular cells.
 Solutes such as paraaminohippuric acid are cleared with a single passage
through the kidneys and this is why it is used to measure renal plasma flow.
Ions such as calcium and phosphate will have a tubular reabsorption that is
influenced by plasma PTH levels.
 Potassium may be both secreted and re-absorbed and is co-exchanged with
sodium.

Loop of Henle

 Approximately 60 litres of water containing 9000mmol sodium enters the


descending limb of the loop of Henle in 24 hours.
 Loops from the juxtamedullary nephrons run deep into the medulla.
 The osmolarity of fluid changes and is greatest at the tip of the papilla.
 The thin ascending limb is impermeable to water, but highly permeable to
sodium and chloride ions.
 This loss means that at the beginning of the thick ascending limb the fluid is
hypo osmotic compared with adjacent interstitial fluid.
 In the thick ascending limb the reabsorption of sodium and chloride ions
occurs by both facilitated and passive diffusion pathways.
 The loops of Henle are co-located with vasa recta, these will have similar
solute compositions to the surrounding extracellular fluid so preventing the
diffusion and subsequent removal of this hypertonic fluid.
 The energy dependent reabsorption of sodium and chloride in the thick
ascending limb helps to maintain this osmotic gradient.

What is measured to obtain renal plasma flow?

A. Creatinine

B. Para-amino hippuric acid (PAH)

C. Inulin

D. Glucose

E. Protein

Renal plasma flow = (amount of PAH in urine per unit time) / (difference in PAH
concentration in the renal artery or vein)

Normal value = 660ml/min

Renal Physiology

Overview

 Each nephron is supplied with blood from an afferent arteriole that opens onto
the glomerular capillary bed.
 Blood then flows to an efferent arteriole, supplying the peritubular capillaries
and medullary vasa recta.
 The kidney receives up to 25% of resting cardiac output.

Control of blood flow

 The kidney is able to autoregulate its blood flow between systolic pressures of
80- 180mmHg so there is little variation in renal blood flow.
 This is achieved by myogenic control of arteriolar tone, both sympathetic
input and hormonal signals (e.g. renin) are responsible.

Glomerular structure and function

 Blood inside the glomerulus has considerable hydrostatic pressure.


 The basement membrane has pores that will allow free diffusion of smaller
solutes, larger negatively charged molecules such as albumin are unable to
cross.
 The glomerular filtration rate (GFR) is equal to the concentration of a solute in
the urine, times the volume of urine produced per minute, divided by the
plasma concentration (assuming that the solute is freely diffused e.g. inulin).
 In clinical practice creatinine is used because it is subjected to very little
proximal tubular secretion.
 Although subject to variability, the typical GFR is 125ml per minute.

 Glomerular filtration rate = Total volume of plasma per unit time leaving the
capillaries and entering the bowman's capsule

 Renal clearance = volume plasma from which a substance is removed per


minute by the kidneys

 Substances used to measure GFR have the following features:

1. Inert
2. Free filtration from the plasma at the glomerulus (not protein bound)
3. Not absorbed or secreted at the tubules
4. Plasma concentration constant during urine collection

Examples: inulin, creatinine

GFR = urine concentration (mmol/l) x urine volume (ml/min)


--------------------------------------------------------------------------
plasma concentration (mmol/l)

 The clearance of a substance is dependent not only on its diffusivity across the
basement membrane but also subsequent tubular secretion and / or
reabsorption.
 So glucose which is freely filtered across the basement membrane is usually
reabsorbed from tubules giving a clearance of zero.

Tubular function

 Reabsorption and secretion of substances occurs in the tubules.


 In the proximal tubule substrates such as glucose, amino acids and phosphate
are co-transported with sodium across the semi permeable membrane.
 Up to two thirds of filtered water is reabsorbed in the proximal tubules.
 This will lead to increase in urea concentration in the distal tubule allowing for
its increased diffusion.
 Substances to be secreted into the tubules are taken up from the peritubular
blood by tubular cells.
 Solutes such as paraaminohippuric acid are cleared with a single passage
through the kidneys and this is why it is used to measure renal plasma flow.
Ions such as calcium and phosphate will have a tubular reabsorption that is
influenced by plasma PTH levels.
 Potassium may be both secreted and re-absorbed and is co-exchanged with
sodium.

Loop of Henle

 Approximately 60 litres of water containing 9000mmol sodium enters the


descending limb of the loop of Henle in 24 hours.
 Loops from the juxtamedullary nephrons run deep into the medulla.
 The osmolarity of fluid changes and is greatest at the tip of the papilla.
 The thin ascending limb is impermeable to water, but highly permeable to
sodium and chloride ions.
 This loss means that at the beginning of the thick ascending limb the fluid is
hypo osmotic compared with adjacent interstitial fluid.
 In the thick ascending limb the reabsorption of sodium and chloride ions
occurs by both facilitated and passive diffusion pathways.
 The loops of Henle are co-located with vasa recta, these will have similar
solute compositions to the surrounding extracellular fluid so preventing the
diffusion and subsequent removal of this hypertonic fluid.
 The energy dependent reabsorption of sodium and chloride in the thick
ascending limb helps to maintain this osmotic gradient.

Which of the following is not a cause of hypercalcaemia?

A. Thiazides

B. Antacids

C. Coeliac disease

D. Sarcoidosis

E. Zolinger-Ellison syndrome
Mnemonic for the causes of
hypercalcaemia:

CHIMPANZEES

C alcium supplementation
H yperparathyroidism
I atrogentic (Drugs: Thiazides)
M ilk Alkali syndrome
P aget disease of the bone
A cromegaly and Addison's
Disease
N eoplasia
Z olinger-Ellison Syndrome
(MEN Type I)
E xcessive Vitamin D
E xcessive Vitamin A
S arcoidosis

Patients with coeliac disease tend to develop hypocalcaemia due to malabsorption of


calcium by the bowel.

Hypercalcaemia

Main causes

 Malignancy
 Primary hyperparathyroidism

Less common

 Sarcoidosis (extrarenal synthesis of calcitriol )


 Thiazides, lithium
 Immobilisation
 Pagets disease
 Vitamin A/D toxicity
 Thyrotoxicosis
 MEN
 Milk alkali syndrome

Clinical features

“Stones, bones, abdominal moans, and psychic groans”


Which of the following surgical procedures will have the greatest long term impact on
a patients calcium metabolism?

A. Distal gastrectomy

B. Cholecystectomy

C. Extensive small bowel resection


D. Sub total colectomy

E. Gastric banding for obesity

Calcium is mainly absorbed from the small bowel and this will have a direct long
term impact on calcium metabolism and increase the risk of osteoporosis. Gastric
banding and distal gastrectomy may affect a patients dietary choices but any potential
deleterious nutritional intake may be counteracted by administration of calcium
supplements orally. Only 10% of calcium is absorbed from the colon so that a sub
total colectomy will only have a negligible effect.

Calcium homeostasis

Calcium ions are linked to a wide range of physiological processes. The largest store
of bodily calcium is contained within the skeleton. Calcium levels are primarily
controlled by parathyroid hormone, vitamin D and calcitonin.

Hormonal regulation of calcium


Hormone Actions
Parathyroid hormone (PTH)  Increase calcium levels and decrease
phosphate levels
 Increases bone resorption
 Immediate action on osteoblasts to
increase ca2+ in extracellular fluid
 Osteoblasts produce a protein signaling
molecule that activate osteoclasts which
cause bone resorption
 Increases renal tubular reabsorption of
calcium
 Increases synthesis of 1,25(OH)2D
(active form of vitamin D) in the kidney
which increases bowel absorption of Ca2+
 Decreases renal phosphate reabsorption

1,25-dihydroxycholecalciferol  Increases plasma calcium and plasma


(the active form of vitamin D) phosphate
 Increases renal tubular reabsorption and
gut absorption of calcium
 Increases osteoclastic activity
 Increases renal phosphate reabsorption

Calcitonin  Secreted by C cells of thyroid


 Inhibits intestinal calcium absorption
 Inhibits osteoclast activity
 Inhibits renal tubular absorption of
calcium
Both growth hormone and thyroxine also play a small role in calcium metabolism.
A 52-year-old woman with a history of gastrectomy reports lethargy and a sore
tongue. Blood tests are reported as follows:

Hb 10.7 g/dl
MCV 121 fl
Plt 177 * 10^9/l
WBC 5.4 * 10^9/l

What is the most likely cause?

A. Vitamin B12 deficiency

B. Vitamin C deficiency

C. Iron deficiency anaemia

D. Anaemia of chronic disease

E. Vitamin E deficiency

A history of gastrectomy and a macrocytic anaemia should indicate a diagnosis of


B12 deficiency.

Vitamin B12 deficiency

Vitamin B12 is mainly used in the body for red blood cell development and also
maintenance of the nervous system. It is absorbed after binding to intrinsic factor
(secreted from parietal cells in the stomach) and is actively absorbed in the terminal
ileum. A small amount of vitamin B12 is passively absorbed without being bound to
intrinsic factor.

Causes of vitamin B12 deficiency

 pernicious anaemia
 post gastrectomy
 poor diet
 disorders of terminal ileum (site of absorption): Crohn's, blind-loop etc

Features of vitamin B12 deficiency

 macrocytic anaemia
 sore tongue and mouth
 neurological symptoms: e.g. Ataxia
 neuropsychiatric symptoms: e.g. Mood disturbances

Management

 if no neurological involvement 1 mg of IM hydroxocobalamin 3 times each


week for 2 weeks, then once every 3 months
 if a patient is also deficient in folic acid then it is important to treat the B12
deficiency first to avoid precipitating subacute combined degeneration of the
cord

A 43 year old lady is diagnosed with primary hyperparathyroidism. Her serum PTH
levels are elevated. An endocrine surgeon performs a parathyroidectomy. How long
will it take for the serum PTH levels to fall if the functioning adenoma has been
successfully removed?

A. 6 hours

B. 24 hours

C. 2 hours

D. 1 hour

E. 10 minutes

PTH has a very short half life usually less than 10 minutes. Therefore a demonstrable
drop in serum PTH should be identified within 10 minutes of removing the adenoma.
This is useful clinically since it is possible to check the serum PTH intraoperatively
prior to skin closure and explore the other glands if levels fail to fall.

Calcium homeostasis

Calcium ions are linked to a wide range of physiological processes. The largest store
of bodily calcium is contained within the skeleton. Calcium levels are primarily
controlled by parathyroid hormone, vitamin D and calcitonin.

Hormonal regulation of calcium


Hormone Actions
Parathyroid hormone (PTH)  Increase calcium levels and decrease
phosphate levels
 Increases bone resorption
 Immediate action on osteoblasts to
increase ca2+ in extracellular fluid
 Osteoblasts produce a protein signaling
molecule that activate osteoclasts which
cause bone resorption
 Increases renal tubular reabsorption of
calcium
 Increases synthesis of 1,25(OH)2D
(active form of vitamin D) in the kidney
which increases bowel absorption of Ca2+
 Decreases renal phosphate reabsorption

1,25-dihydroxycholecalciferol  Increases plasma calcium and plasma


(the active form of vitamin D) phosphate
 Increases renal tubular reabsorption and
gut absorption of calcium
 Increases osteoclastic activity
 Increases renal phosphate reabsorption

Calcitonin  Secreted by C cells of thyroid


 Inhibits intestinal calcium absorption
 Inhibits osteoclast activity
 Inhibits renal tubular absorption of
calcium

Both growth hormone and thyroxine also play a small role in calcium metabolism.
Which of the following statements relating to abnormal coagulation is false?

A. Warfarin affects the synthesis of factor 2,7,9,10

B. The prothrombin time is prolonged in Haemophilia A

C. Cholestatic jaundice can cause vitamin K deficiency

D. Disseminated intravascular coagulation is associated with


thrombocytopenia

E. Massive transfusion is associated with reduced levels of factor 5 and


8

In haemophilia A the APTT is prolonged and there is reduced levels of factor 8:C.
The bleeding time and PT are normal. Cholestatic jaundice prevents the absorption of
the fat soluble vitamin K. Massive transfusion (>10u blood or equivalent to the blood
volume of a person) puts the patient at risk of thrombocytopaenia, factor 5 and 8
deficiency.

Abnormal coagulation

Cause Factors affected


Heparin Prevents activation factors 2,9,10,11
Warfarin Affects synthesis of factors 2,7,9,10
DIC Factors 1,2,5,8,11
Liver disease Factors 1,2,5,7,9,10

Interpretation blood clotting test results


Disorder APTT PT Bleeding time
Haemophilia Increased Normal Normal
von Willebrand's disease Increased Normal Increased
Vitamin K deficiency Increased Increased Normal
Which of the following physiological changes do not occur following tracheostomy?

A. Alveolar ventilation is increased.

B. Anatomical dead space is reduced by 50%.

C. Work of breathing is increased.

D. Proportion of ciliated epithelial cells in the trachea may decrease.

E. Splinting of the larynx may lead to swallowing difficulties.

Work of breathing is decreased which is one reasons it is popular option for weaning
ventilated patients. Humdified air in this setting helps to reduce the viscosity of
mucous that forms.

Trachea

Trachea
Location C6 vertebra to the upper border of T5 vertebra
(bifurcation)
Arterial and venous Inferior thyroid arteries and the thyroid venous plexus.
supply
Nerve Branches of vagus, sympathetic and the recurrent nerves

Relations in the neck


Anterior(Superior to  Isthmus of the thyroid gland
inferior)  Inferior thyroid veins
 Arteria thyroidea ima (when that vessel exists)
 Sternothyroid
 Sternohyoid
 Cervical fascia
 Anastomosing branches between the anterior
jugular veins
Posterior Oesophagus.
Laterally  Common carotid arteries
 Right and left lobes of the thyroid gland
 Inferior thyroid arteries
 Recurrent laryngeal nerves

Relations in the thorax

Anterior

 Manubrium sterni, the remains of the thymus, the aortic arch, left common
carotid arteries, and the deep cardiac plexus

Lateral

 In the superior mediastinum, on the right side is the pleura and right vagus; on
its left side are the left recurrent nerve, the aortic arch, and the left common
carotid and subclavian arteries.

A 34 year old man presents with a peptic ulcer. Which of the following is responsible for the
release of gastric acid?

A. Chief cells

B. Parietal cells

C. Brunners Glands

D. G Cells

E. None of the above

Parietal cells are responsible for the release of gastric acid. Brunners glands are found in the
duodenum.

Gastric secretions

A working knowledge of gastric secretions is important for surgery because peptic ulcers are
common, surgeons frequently prescribe anti secretory drugs and because there are still
patients around who will have undergone acid lowering procedures (Vagotomy) in the past.
Gastric acid

 Is produced by the parietal cells in the stomach


 pH of gastric acid is around 2 with acidity being maintained by the H+/K+ ATP ase
pump. As part of the process bicarbonate ions will be secreted into the surrounding
vessels.
 Sodium and chloride ions are actively secreted from the parietal cell into the
canaliculus. This sets up a negative potential across the membrane and as a result
sodium and potassium ions diffuse across into the canaliculus.
 Carbonic anhydrase forms carbonic acid which dissociates and the hydrogen ions
formed by dissociation leave the cell via the H+/K+ antiporter pump. At the same
time sodium ions are actively absorbed. This leaves hydrogen and chloride ions in
the canaliculus these mix and are secreted into the lumen of the oxyntic gland.

This is illustrated diagrammatically below:

Image sourced from Wikipedia

Phases of gastric acid secretion


There are 3 phases of gastric secretion:

1. Cephalic phase (smell / taste of food)

 30% acid produced


 Vagal cholinergic stimulation causing secretion of HCL and gastrin release from G
cells

2. Gastric phase (distension of stomach )

 60% acid produced


 Stomach distension/low H+/peptides causes Gastrin release

3. Intestinal phase (food in duodenum)

 10% acid produced


 High acidity/distension/hypertonic solutions in the duodenum inhibits gastric acid
secretion via enterogastrones (CCK, secretin) and neural reflexes.

Regulation of gastric acid production


Factors increasing production include:

 Vagal nerve stimulation


 Gastrin release
 Histamine release (indirectly following gastrin release) from enterchromaffin like
cells

Factors decreasing production include:

 Somatostatin (inhibits histamine release)


 Cholecystokinin
 Secretin

The diagram below illustrates some of the factors involved in regulating gastric acid
secretion and the relevant associated pharmacology
Image sourced from Wikipedia

Below is a brief summary of the major hormones involved in food digestion:

Source Stimulus Actions

Gastrin G cells in Distension of Increase HCL, pepsinogen and IF secretion,


antrum of stomach, extrinsic increases gastric motility, trophic effect on
the stomach nerves gastric mucosa
Inhibited by: low
antral pH,
somatostatin

CCK I cells in Partially digested Increases secretion of enzyme-rich fluid


upper small proteins and from pancreas, contraction of gallbladder
intestine triglycerides and relaxation of sphincter of Oddi,
decreases gastric emptying, trophic effect
on pancreatic acinar cells, induces satiety

Secretin S cells in Acidic chyme, fatty Increases secretion of bicarbonate-rich fluid


upper small acids from pancreas and hepatic duct cells,
intestine decreases gastric acid secretion, trophic
effect on pancreatic acinar cells

VIP Small Neural Stimulates secretion by pancreas and


intestine, intestines, inhibits acid and pepsinogen
pancreas secretion
Somatostatin D cells in the Fat, bile salts and Decreases acid and pepsin secretion,
pancreas and glucose in the decreases gastrin secretion, decreases
stomach intestinal lumen pancreatic enzyme secretion, decreases
insulin and glucagon secretion
inhibits trophic effects of gastrin, stimulates
gastric mucous production

A 34 year old man presents with a peptic ulcer. Which of the following is responsible for the
release of gastric acid?

A. Chief cells

B. Parietal cells

C. Brunners Glands

D. G Cells

E. None of the above

Parietal cells are responsible for the release of gastric acid. Brunners glands are found in the
duodenum.

Gastric secretions

A working knowledge of gastric secretions is important for surgery because peptic ulcers are
common, surgeons frequently prescribe anti secretory drugs and because there are still
patients around who will have undergone acid lowering procedures (Vagotomy) in the past.

Gastric acid

 Is produced by the parietal cells in the stomach


 pH of gastric acid is around 2 with acidity being maintained by the H+/K+ ATP ase
pump. As part of the process bicarbonate ions will be secreted into the surrounding
vessels.
 Sodium and chloride ions are actively secreted from the parietal cell into the
canaliculus. This sets up a negative potential across the membrane and as a result
sodium and potassium ions diffuse across into the canaliculus.
 Carbonic anhydrase forms carbonic acid which dissociates and the hydrogen ions
formed by dissociation leave the cell via the H+/K+ antiporter pump. At the same
time sodium ions are actively absorbed. This leaves hydrogen and chloride ions in
the canaliculus these mix and are secreted into the lumen of the oxyntic gland.

This is illustrated diagrammatically below:

Image sourced from Wikipedia

Phases of gastric acid secretion


There are 3 phases of gastric secretion:

1. Cephalic phase (smell / taste of food)

 30% acid produced


 Vagal cholinergic stimulation causing secretion of HCL and gastrin release from G
cells

2. Gastric phase (distension of stomach )

 60% acid produced


 Stomach distension/low H+/peptides causes Gastrin release

3. Intestinal phase (food in duodenum)

 10% acid produced


 High acidity/distension/hypertonic solutions in the duodenum inhibits gastric acid
secretion via enterogastrones (CCK, secretin) and neural reflexes.

Regulation of gastric acid production


Factors increasing production include:

 Vagal nerve stimulation


 Gastrin release
 Histamine release (indirectly following gastrin release) from enterchromaffin like
cells

Factors decreasing production include:

 Somatostatin (inhibits histamine release)


 Cholecystokinin
 Secretin

The diagram below illustrates some of the factors involved in regulating gastric acid
secretion and the relevant associated pharmacology

Image sourced from Wikipedia


Below is a brief summary of the major hormones involved in food digestion:

Source Stimulus Actions

Gastrin G cells in Distension of Increase HCL, pepsinogen and IF secretion,


antrum of stomach, extrinsic increases gastric motility, trophic effect on
the stomach nerves gastric mucosa
Inhibited by: low
antral pH,
somatostatin

CCK I cells in Partially digested Increases secretion of enzyme-rich fluid


upper small proteins and from pancreas, contraction of gallbladder
intestine triglycerides and relaxation of sphincter of Oddi,
decreases gastric emptying, trophic effect
on pancreatic acinar cells, induces satiety

Secretin S cells in Acidic chyme, fatty Increases secretion of bicarbonate-rich fluid


upper small acids from pancreas and hepatic duct cells,
intestine decreases gastric acid secretion, trophic
effect on pancreatic acinar cells

VIP Small Neural Stimulates secretion by pancreas and


intestine, intestines, inhibits acid and pepsinogen
pancreas secretion

Somatostatin D cells in the Fat, bile salts and Decreases acid and pepsin secretion,
pancreas and glucose in the decreases gastrin secretion, decreases
stomach intestinal lumen pancreatic enzyme secretion, decreases
insulin and glucagon secretion
inhibits trophic effects of gastrin, stimulates
gastric mucous production

Which of the following does not lead to relaxation of the lower oesophageal
sphincter?

A. Metoclopramide

B. Botulinum toxin type A

C. Nicotine

D. Alcohol
E. Theophylline

Metoclopramide acts directly on the smooth muscle of the LOS to cause it to contract.
Theophylline is a phosphodiesterase inhibitor (mimics action of prostaglandin E1)
which causes relaxation of the LOS.

Peristalsis

 Circular smooth muscle contracts behind the food bolus and longitudinal
smooth muscle propels the food through the oesophagus
 Primary peristalsis spontaneously moves the food from the oesophagus into
the stomach (9 seconds)
 Secondary peristalsis occurs when food, which doesn't enter the stomach,
stimulates stretch receptors to cause peristalsis
 In the small intestine each peristalsis waves slows to a few seconds and causes
mixture of chyme
 In the colon three main types of peristaltic activity are recognised (see below)

Colonic peristalsis
Segmentation Localised contractions in which the bolus is subjected to
contractions local forces to maximise mucosal absorption
Antiperistaltic Localised reverse peristaltic waves to slow entry into
contractions towards colon and maximise absorption
ileum
Mass movements Waves migratory peristaltic waves along the entire colon
to empty the organ prior to the next ingestion of food
bolus
Which of the following is not released from the islets of Langerhans?

A. Pancreatic polypeptide

B. Glucagon

C. Secretin

D. Somatostatin

E. Insulin

Secretin is released from mucosal cells in the duodenum and jejunum.

Pancreas endocrine physiology


Hormones released from the islets of Langerhans
Beta cells Insulin (70% of total secretions)
Alpha cells Glucagon
Delta cells Somatostatin
F cells Pancreatic polypeptide

Which of the following is not classically seen in coning resulting from raised intra
cranial pressure?

A. Coma

B. Hypotension

C. Unreactive mid sized pupils

D. Cheyne Stokes style respiratory efforts

E. Bradycardia
Cushings triad

 Widening of the pulse


pressure
 Respiratory changes
 Bradycardia

Due to raised ICP systemic hypertension is usually seen. Compression of the


respiratory centre will typically result in Cheyne Stokes style respiration.

Coning

 The cranial vault is a confined cavity apart from infants with a non fused
fontanelle.
 Rises in ICP may be accommodated by shifts of CSF.
 Once the CSF shifting has reached its capacity ICP will start to rise briskly.
 The brain autoregulates its blood supply, as ICP rises the systemic circulation
will display changes to try and meet the perfusion needs of the brain. Usually
this will involve hypertension.
 As CSF rises further, the brain will be compressed, cranial nerve palsies may
be seen and compression of essential centres in the brain stem will occur.
When the cardiac centre is involved bradycardia will often develop.

Control of ventilation. Which statement is false?


A. Peripheral chemoreceptors are located in the bifurcation of the carotid
arteries and arch of the aorta

B. Central chemoreceptors respond to changes in O2

C. The respiratory centres control the rate and depth of respiration

D. Involuntary control of respiration is from the medulla and pons

E. Irritant receptors cause bronchospasm


- Central chemoreceptors: Respond to increased H+ in BRAIN INTERSTITIAL
FLUID to increase ventilation.

Control of ventilation

 Control of ventilation is coordinated by the respiratory centres,


chemoreceptors, lung receptors and muscles.
 Automatic, involuntary control of respiration occurs from the medulla.
 The respiratory centres control the respiratory rate and the depth of respiration.

Respiratory centres

 Medullary respiratory centre:

Inspiratory and expiratory neurones. Has ventral group which controls forced
voluntary expiration and the dorsal group controls inspiration. Depressed by opiates.

 Apneustic centre:

Lower pons
Stimulates inspiration - activates and prolongs inhalation
Overridden by pneumotaxic control to end inspiration

 Pneumotaxic centre:

Upper pons, inhibits inspiration at a certain point. Fine tunes the respiratory rate.

 Levels of PCO2 most important in ventilation control


 Levels of O2 are less important.
 Peripheral chemoreceptors: located in the bifurcation of carotid arteries and
arch of the aorta. They respond to changes in reduced pO2, increased H+ and
increased pCO2 in ARTERIAL BLOOD.
 Central chemoreceptors: located in the medulla. Respond to increased H+ in
BRAIN INTERSTITIAL FLUID to increase ventilation. NB the central
receptors are NOT influenced by O2 levels.

Lung receptors include:

 Stretch receptors: respond to lung stretching causing a reduced respiratory rate


 Irritant receptors: respond to smoke etc causing bronchospasm
 J (juxtacapillary) receptors

A 54 year old lady has her serum calcium measured. Assuming her renal function is
normal, what proportion of calcium filtered at the glomerulus will be reabsorbed by
the renal tubules?

A. 5%

B. 15%

C. 25%

D. 50%

E. 95%

Most filtered calcium is reabsorbed (95%) a rare disorder of familial hypercalcemic


calciurea may affect this proportion.

Calcium homeostasis

Calcium ions are linked to a wide range of physiological processes. The largest store
of bodily calcium is contained within the skeleton. Calcium levels are primarily
controlled by parathyroid hormone, vitamin D and calcitonin.

Hormonal regulation of calcium


Hormone Actions
Parathyroid hormone (PTH)  Increase calcium levels and decrease
phosphate levels
 Increases bone resorption
 Immediate action on osteoblasts to
increase ca2+ in extracellular fluid
 Osteoblasts produce a protein signaling
molecule that activate osteoclasts which
cause bone resorption
 Increases renal tubular reabsorption of
calcium
 Increases synthesis of 1,25(OH)2D
(active form of vitamin D) in the kidney
which increases bowel absorption of Ca2+
 Decreases renal phosphate reabsorption

1,25-dihydroxycholecalciferol  Increases plasma calcium and plasma


(the active form of vitamin D) phosphate
 Increases renal tubular reabsorption and
gut absorption of calcium
 Increases osteoclastic activity
 Increases renal phosphate reabsorption

Calcitonin  Secreted by C cells of thyroid


 Inhibits intestinal calcium absorption
 Inhibits osteoclast activity
 Inhibits renal tubular absorption of
calcium

Both growth hormone and thyroxine also play a small role in calcium metabolism.
Which of the following does not cause hyperkalaemia?

A. Haemolysis

B. Burns

C. Familial periodic paralysis

D. Type 4 renal tubular acidosis

E. Severe malnutrition
'Machine' - Causes of Increased
Serum K+

M - Medications - ACE inhibitors,


NSAIDS
A - Acidosis - Metabolic and
respiratory
C - Cellular destruction - Burns,
traumatic injury
H - Hypoaldosteronism,
haemolysis
I - Intake - Excessive
N - Nephrons, renal failure
E - Excretion - Impaired

Familial periodic paralysis has subtypes associated with hyper and hypokalaemia.

Hyperkalaemia

 Plasma potassium levels are regulated by a number of factors including


aldosterone, acid-base balance and insulin levels.
 Metabolic acidosis is associated with hyperkalaemia as hydrogen and
potassium ions compete with each other for exchange with sodium ions across
cell membranes and in the distal tubule.
 ECG changes seen in hyperkalaemia include tall-tented T waves, small P
waves, widened QRS leading to a sinusoidal pattern and asystole

Causes of hyperkalaemia

 Acute renal failure


 Drugs*: potassium sparing diuretics, ACE inhibitors, angiotensin 2 receptor
blockers, spironolactone, ciclosporin, heparin**
 Metabolic acidosis
 Addison's
 Tissue necrosis/rhabdomylosis: burns, trauma
 Massive blood transfusion

Foods that are high in potassium

 Salt substitutes (i.e. Contain potassium rather than sodium)


 Bananas, oranges, kiwi fruit, avocado, spinach, tomatoes

*beta-blockers interfere with potassium transport into cells and can potentially cause
hyperkalaemia in renal failure patients - remember beta-agonists, e.g. Salbutamol, are
sometimes used as emergency treatment

**both unfractionated and low-molecular weight heparin can cause hyperkalaemia.


This is thought to be caused by inhibition of aldosterone secretion
Which of the following statements is true of glucagon?

A. Produced in response to hyperglycaemia

B. Released by beta cells

C. Inhibits gluconeogenesis
D. Produced in response to an increase of amino acids

E. Composed of 2 alpha polypeptide chains linked by hydrogen bonds

Glucagon is a protein comprised of a single polypeptide chain.


Produced by alpha cells of pancreatic islets of Langerhans in response to
hypoglycaemia and amino acids.
It increases plasma glucose and ketones.

Glucagon

Glucagon, the hormonal antagonist to insulin, is released from the alpha cells of the
Islets of Langerhans in the pancreas. It will result in an increased plasma glucose
level.

Stimulation Inhibition
Decreased plasma glucose Somatostatin
Increased catecholamines Insulin
Increased free fatty acids and keto acids
Increased plasma amino acids
Sympathetic nervous system Increased urea

Acetylcholine

Cholecystokinin

A 28 year old man undergoes a completion right hemicolectomy for treatment of a


5cm appendiceal carcinoid. As part of his follow up he is due to undergo 24 hour
urine collection for 5-HIAA. Which of the following causes an elevated 5-HIAA in a
24-hour urine collection?

A. Isoniazid

B. Oranges

C. Flucloxacillin

D. Amiodarone

E. Beef

It is important to be aware of what can falsely elevate 5-HIAA to avoid diagnosing


carcinoid syndrome incorrectly. These include:
Food: spinach, cheese, wine, caffeine, tomatoes
Drugs: Isoniazid, Monoamine oxidase inhibitors

Carcinoid syndrome

 Carcinoid tumours secrete serotonin


 Originate in neuroendocrine cells mainly in the intestine (midgut-distal
ileum/appendix)
 Can occur in the rectum, bronchi
 Hormonal symptoms mainly occur when disease spreads outside the bowel

Clinical features
- Onset: years
- Flushing face
- Palpitations
- Tricuspid stenosis causing dyspnoea
- Asthma
- Severe diarrhoea (secretory, persists despite fasting)

Investigation
- 5-HIAA in a 24-hour urine collection
- Scintigraphy
- CT scan

Treatment

 Octreotide
 Surgical removal

A 52 year old man develops septic shock following a Hartmans procedure for
perforated diverticular disease. He is started on an adrenaline infusion. Which of the
following is least likely to occur?

A. Peripheral vasoconstriction

B. Coronary artery vasospasm

C. Gluconeogenesis

D. Lipolysis

E. Tachycardia

Its cardiac effects are mediated via β 1 receptors. The coronary arteries which have β
2 receptors are unaffected.
Adrenaline

 Fight or Flight response

- Catecholamine (phenylalanine and tyrosine)


- Neurotransmitter and hormone
- Released by the adrenal glands
- Effects on α 1 and 2, β 1 and 2 receptors
- Main effect on alpha 1 receptors in skeletal muscle-causing vasodilation
- Increase cardiac output and total peripheral resistance
- This leads to vasoconstriction in the skin and kidneys causing a narrow pulse
pressure

Actions
α adrenergic receptors:

 Inhibits insulin secretion by the pancreas


 Stimulates glycogenolysis in the liver and muscle
 Stimulates glycolysis in muscle

β adrenergic receptors:

 Stimulates glucagon secretion in the pancreas


 Stimulates ACTH
 Stimulates lipolysis by adipose tissue

Intra cranial pressure is governed by the principles of the Monroe-Kellie doctrine. To


which of the following does this concept not apply?

A. A 2 month old child

B. A 2 year old child

C. A 5 year old child

D. A 10 year old child

E. An adult

The Monroe-Kelly Doctrine assumes that the cranial cavity is a rigid box. In children
with non fused fontanells this is not the case.

Applied neurophysiology
 Pressure within the cranium is governed by the Monroe-Kelly doctrine. This
considers the skull as a closed box. Increases in mass can be accommodated
by loss of CSF. Once a critical point is reached (usually 100- 120ml of CSF
lost) there can be no further compensation and ICP rises sharply. The next step
is that pressure will begin to equate with MAP and neuronal death will occur.
Herniation will also accompany this process.
 The CNS can autoregulate its own blood supply. Vaso constriction and
dilatation of the cerebral blood vessels is the primary method by which this
occurs. Extremes of blood pressure can exceed this capacity resulting in risk
of stroke. Other metabolic factors such as hypercapnia will also cause
vasodilation, which is of importance in ventilating head injured patients.
 The brain can only metabolise glucose, when glucose levels fall,
consciousness will be impaired.

Which of the following is not caused by cortisol in the stress response?

A. Anti-inflammatory effects

B. Hypoglycaemia

C. Skeletal muscle protein breakdown

D. Stimulation of lipolysis

E. Mineralocorticoid effects

An 'anti insulin' effect occurs leading to hyperglycaemia.

Stress response: Endocrine and metabolic changes

 Surgery precipitates hormonal and metabolic changes causing the stress


response
 Stress response is associated with: substrate mobilization, muscle protein loss,
sodium and water retention, suppression of anabolic hormone secretion,
activation of the sympathetic nervous system, immunological and
haematological changes.
 The hypothalamic-pituitary axis and the sympathetic nervous systems are
activated and there is a failure of the normal feedback mechanisms of control
of hormone secretion.

A summary of the hormonal changes associated with the stress response:

Increased Decreased No Change


Growth hormone Insulin Thyroid stimulating hormone
Cortisol Testosterone Luteinizing hormone
Renin Oestrogen Follicle stimulating hormone
Adrenocorticotrophic hormone (ACTH)
Aldosterone

Prolactin

Antidiuretic hormone
Glucagon

Sympathetic nervous system

 Stimulates catecholamine release


 Causes tachycardia and hypertension

Pituitary gland

 ACTH and growth hormone (GH) is stimulated by hypothalamic releasing


factors, corticotrophin releasing factor (CRF) and somatotrophin (or growth
hormone releasing factor)
 Perioperative increased prolactin secretion occurs by release of inhibitory
control
 Secretion of thyroid stimulating hormone (TSH), luteinizing hormone (LH)
and follicle stimulating hormone (FSH) does not change significantly
 ACTH stimulates cortisol production within a few minutes of the start of
surgery. More ACTH is produced than needed to produce a maximum
adrenocortical response.

Cortisol

 Significant increases within 4-6h of surgery (>1000 nmol litre-1)


 The usual negative feedback mechanism fails and concentrations of ACTH
and cortisol remain persistently increased
 The magnitude and duration of the increase correlate with the severity of stress
and the response is not abolished by the administration of corticosteroids.
 The metabolic effects of cortisol are enhanced:

Skeletal muscle protein breakdown to provide gluconeogenic precursors and amino


acids for protein synthesis in the liver
Stimulation of lipolysis
'Anti-insulin effect'
Mineralocorticoid effects
Anti-inflammatory effects
Growth hormone

 Increased secretion after surgery has a minor role


 Most important for preventing muscle protein breakdown and promote tissue
repair by insulin growth factors

Alpha Endorphin

 Increased

Antidiuretic hormone

 An important vasopressor and enhances haemostasis


 Renin is released causing the conversion of angiotensin I to angiotensin II,
which causes the secretion of aldosterone from the adrenal cortex. This
increases sodium reabsorption at the distal convoluted tubule

Insulin

 Release inhibited by stress


 Occurs via the inhibition of the alpha cells in the pancreas by the α2-
adrenergic inhibitory effects of catecholamines
 Insulin resistance by target cells occurs later
 The perioperative period is characterized by a state of functional insulin
deficiency

Thyroxine (T4) and tri-iodothyronine (T3)

 Circulating concentrations are inversely correlated with sympathetic activity


and after surgery there is a reduction in thyroid hormone production, which
normalises over a few days.

Metabolic effect of endocrine response

Carbohydrate metabolism

 Hyperglycaemia is a main feature of the metabolic response to surgery


 Due to increased increase in glucose production and a reduction in glucose
utilization
 Catecholamines and cortisol promote glycogenolysis and gluconeogenesis
 Initial failure of insulin secretion followed by insulin resistance affects the
normal responses
 The proportion of the hyperglycaemic response reflects the severity of surgery
 Hyperglycaemia impairs wound healing and increase infection rates

Protein metabolism

 Initially there is inhibition of protein anabolism, followed later, if the stress


response is severe, by enhanced catabolism
 The amount of protein degradation is influenced by the type of surgery and
also by the nutritional status of the patient
 Mainly skeletal muscle protein is affected
 The amino acids released form acute phase proteins (fibrinogen, C reactive
protein, complement proteins, a2-macroglobulin, amyloid A and
ceruloplasmin) and are used for gluconeogenesis
 Nutritional support has little effect on preventing catabolism

Lipid metabolism
Increased catecholamine, cortisol and glucagon secretion, and insulin deficiency,
promotes lipolysis and ketone body production.

Salt and water metabolism

 ADH causes water retention, concentrated urine, and potassium loss and may
continue for 3 to 5 days after surgery
 Renin causes sodium and water retention

Cytokines

 Glycoproteins
 Interleukins (IL) 1 to 17, interferons, and tumour necrosis factor
 Synthesized by activated macrophages, fibroblasts, endothelial and glial cells
in response to tissue injury from surgery or trauma
 IL-6 main cytokine associated with surgery. Peak 12 to 24 h after surgery and
increase by the degree of tissue damage Other effects of cytokines include
fever, granulocytosis, haemostasis, tissue damage limitation and promotion of
healing.

Modifying the response

 Opioids suppress hypothalamic and pituitary hormone secretion


 At high doses the hormonal response to pelvic and abdominal surgery is
abolished. However, such doses prolong recovery and increase the need for
postoperative ventilatory support
 Spinal anaesthesia can reduce the glucose, ACTH, cortisol, GH and
epinephrine changes, although cytokine responses are unaltered
 Cytokine release is reduced in less invasive surgery
 Nutrition prevents the adverse effects of the stress response. Enteral feeding
improves recovery
 Growth hormone and anabolic steroids may improve outcome
 Normothermia decreases the metabolic response

References
Deborah Burton, Grainne Nicholson, and George Hall
Endocrine and metabolic response to surgery .

Contin Educ Anaesth Crit Care Pain (2004) 4(5): 144-147


doi:10.1093/bjaceaccp/mkh040
Which of the following features does not put a patient at risk of refeeding syndrome?

A. BMI < 16 kg/m2

B. Alcohol abuse

C. Thyrotoxicosis

D. Chemotherapy

E. Diuretics

Diuretics increase the risk of re-feeding syndrome through a process of increasing the
risk of depletion of key electrolytes.

Nutrition - Refeeding syndrome

Refeeding syndrome describes the metabolic abnormalities which occur on feeding a


person a starved state. The metabolic consequences include:

 Hypophosphataemia
 Hypokalaemia
 Hypomagnesaemia
 Abnormal fluid balance

These abnormalities can lead to organ failure.

Re-feeding problems
If patient not eaten for > 5 days, aim to re-feed at < 50% energy and protein levels

High risk for re-feeding problems


If one or more of the following:

 BMI < 16 kg/m2


 Unintentional weight loss >15% over 3-6 months
 Little nutritional intake > 10 days
 Hypokalaemia, Hypophosphataemia or hypomagnesaemia prior to feeding
(unless high)

If two or more of the following:

 BMI < 18.5 kg/m2


 Unintentional weight loss > 10% over 3-6 months
 Little nutritional intake > 5 days
 PMH alcohol abuse or drug therapy including insulin, chemotherapy,
diuretics, antacids

Prescription

 Start at up to 10 kcal/kg/day increasing to full needs over 4-7 days


 Start immediately before and during feeding: oral thiamine 200-300mg/day,
vitamin B co strong 1 tds and supplements
 Give K+ (2-4 mmol/kg/day), phosphate (0.3-0.6 mmol/kg/day), magnesium
(0.2-0.4 mmol/kg/day)

Which of the following statements relating to the regulation of renal blood flow is
untrue?

A. In a healthy 70Kg male, the glomerular filtration rate will be the


same at a systolic blood pressure of 120mmHg as a systolic blood
pressure of 95 mmHg

B. Over 90% of the blood supply to the kidney is distributed to the


cortex

C. The kidney receives approximately 25% of the total cardiac output at


rest

D. A decrease in renal perfusion pressure will cause the juxtaglomerular


cells to secrete renin

E. Systolic blood pressures of less than 65mmHg will cause the


mesangial cells to secrete aldosterone

The kidney autoregulates its blood supply over a range of systolic blood pressures.
Drop in arterial pressure is sensed by the juxtaglomerular cells and renin is released
leading to the activation of the renin-angiontensin system. Mesangial cells are
contractile cells that are located in the tubule and have no direct endocrine function.

Renal Physiology
Overview

 Each nephron is supplied with blood from an afferent arteriole that opens onto
the glomerular capillary bed.
 Blood then flows to an efferent arteriole, supplying the peritubular capillaries
and medullary vasa recta.
 The kidney receives up to 25% of resting cardiac output.

Control of blood flow

 The kidney is able to autoregulate its blood flow between systolic pressures of
80- 180mmHg so there is little variation in renal blood flow.
 This is achieved by myogenic control of arteriolar tone, both sympathetic
input and hormonal signals (e.g. renin) are responsible.

Glomerular structure and function

 Blood inside the glomerulus has considerable hydrostatic pressure.


 The basement membrane has pores that will allow free diffusion of smaller
solutes, larger negatively charged molecules such as albumin are unable to
cross.
 The glomerular filtration rate (GFR) is equal to the concentration of a solute in
the urine, times the volume of urine produced per minute, divided by the
plasma concentration (assuming that the solute is freely diffused e.g. inulin).
 In clinical practice creatinine is used because it is subjected to very little
proximal tubular secretion.
 Although subject to variability, the typical GFR is 125ml per minute.

 Glomerular filtration rate = Total volume of plasma per unit time leaving the
capillaries and entering the bowman's capsule

 Renal clearance = volume plasma from which a substance is removed per


minute by the kidneys

 Substances used to measure GFR have the following features:

1. Inert
2. Free filtration from the plasma at the glomerulus (not protein bound)
3. Not absorbed or secreted at the tubules
4. Plasma concentration constant during urine collection
Examples: inulin, creatinine

GFR = urine concentration (mmol/l) x urine volume (ml/min)


--------------------------------------------------------------------------
plasma concentration (mmol/l)

 The clearance of a substance is dependent not only on its diffusivity across the
basement membrane but also subsequent tubular secretion and / or
reabsorption.
 So glucose which is freely filtered across the basement membrane is usually
reabsorbed from tubules giving a clearance of zero.

Tubular function

 Reabsorption and secretion of substances occurs in the tubules.


 In the proximal tubule substrates such as glucose, amino acids and phosphate
are co-transported with sodium across the semi permeable membrane.
 Up to two thirds of filtered water is reabsorbed in the proximal tubules.
 This will lead to increase in urea concentration in the distal tubule allowing for
its increased diffusion.
 Substances to be secreted into the tubules are taken up from the peritubular
blood by tubular cells.
 Solutes such as paraaminohippuric acid are cleared with a single passage
through the kidneys and this is why it is used to measure renal plasma flow.
Ions such as calcium and phosphate will have a tubular reabsorption that is
influenced by plasma PTH levels.
 Potassium may be both secreted and re-absorbed and is co-exchanged with
sodium.

Loop of Henle

 Approximately 60 litres of water containing 9000mmol sodium enters the


descending limb of the loop of Henle in 24 hours.
 Loops from the juxtamedullary nephrons run deep into the medulla.
 The osmolarity of fluid changes and is greatest at the tip of the papilla.
 The thin ascending limb is impermeable to water, but highly permeable to
sodium and chloride ions.
 This loss means that at the beginning of the thick ascending limb the fluid is
hypo osmotic compared with adjacent interstitial fluid.
 In the thick ascending limb the reabsorption of sodium and chloride ions
occurs by both facilitated and passive diffusion pathways.
 The loops of Henle are co-located with vasa recta, these will have similar
solute compositions to the surrounding extracellular fluid so preventing the
diffusion and subsequent removal of this hypertonic fluid.
 The energy dependent reabsorption of sodium and chloride in the thick
ascending limb helps to maintain this osmotic gradient.
39 year old lady undergoes a laparoscopic cholecystectomy as a daycase. The
operation is more difficult than anticipated and the surgeon places a drain to the liver
bed. In recovery 1.5 litres of blood is seen to enter the drain. Which of the following
substances is the first to be released in this situation?

A. Angiotensinogen

B. Renin

C. Angiotensin I

D. Angiotensin II

E. Aldosterone

The decrease in blood pressure will be sensed by the juxtaglomerular cells in the
kidney. This will cause renin secretion.

Shock

 Shock occurs when there is insufficient tissue perfusion.


 The pathophysiology of shock is an important surgical topic and may be
divided into the following aetiological groups:
 Septic
 Haemorrhagic
 Neurogenic
 Cardiogenic
 Anaphylactic

Septic shock
Septic shock is a major problem and those patients with severe sepsis have a mortality
rate in excess of 40%. In those who are admitted to intensive care mortality ranges
from 6% with no organ failure to 65% in those with 4 organ failure.

Sepsis is defined as an infection that triggers a particular Systemic Inflammatory


Response Syndrome (SIRS). This is characterised by body temperature outside 36
o
C - 38 o C, HR >90 beats/min, respiratory rate >20/min, WBC count >12,000/mm3 or
< 4,000/mm3.

Patients with infections and two or more elements of SIRS meet the diagnostic criteria
for sepsis. Those with organ failure have severe sepsis and those with refractory
hypotension -septic shock.

During the septic process there is marked activation of the immune system with
extensive cytokine release. This may be coupled with or triggered by systemic
circulation of bacterial toxins. These all cause endothelial cell damage and neutrophil
adhesion. The overall hallmarks are thus those of excessive inflammation,
coagulation and fibrinolytic suppression.

The surviving sepsis campaign highlights the following key areas for attention:

 Prompt administration of antibiotics to cover all likely pathogens coupled with


a rigorous search for the source of infection.
 Haemodynamic stabilisation. Many patients are hypovolaemic and require
aggressive fluid administration. Aim for CVP 8-12 cm H2O, MAP >65mmHg.
 Modulation of the septic response. This includes manoeuvres to counteract the
changes and includes measures such as tight glycaemic control, use of
activated protein C and sometimes intravenous steroids.

In surgical patients, the main groups with septic shock include those with anastomotic
leaks, abscesses and extensive superficial infections such as necrotising fasciitis.
When performing surgery the aim should be to undertake the minimum necessary to
restore physiology. These patients do not fare well with prolonged surgery. Definitive
surgery can be more safely undertaken when physiology is restored and clotting in
particular has been normalised.

Haemorrhagic shock
The average adult blood volume comprises 7% of body weight. Thus in the 70 Kg
adult this will equate to 5 litres. This changes in children (8-9% body weight) and is
slightly lower in the elderly.

The table below outlines the 4 major classes of haemorrhagic shock and their
associated physiological sequelae:

Parameter Class I Class II Class III Class IV


Blood loss ml <750ml 750-1500ml 1500-2000ml >2000ml
Blood loss % <15% 15-30% 30-40% >40%
Pulse rate <100 >100 >120 >140ml
Blood pressure Normal Decreased Decreased Decreased
Respiratory rate 14-20 20-30 30-40 >35
Urine output >30ml 20-30ml 5-15ml <5ml
Symptoms Normal Anxious Confused Lethargic

Decreasing blood pressure during haemorrhagic shock causes organ hypoperfusion


and relative myocardial ishaemia. The cardiac index gives a numerical value for tissue
oxygen delivery and is given by the equation: Cardiac index= 13.4 - [Hb] - SaO2 +
0.03 PaO2. Where Hb is haemoglobin concentration in blood and SaO2 the saturation
and PaO2 the partial pressure of oxygen. Detailed knowledge of this equation is
required for the MRCS Viva but not for part A, although you should understand the
principle.
In patients suffering from trauma the most likely cause of shock is haemorrhage.
However, the following may also be the cause or occur concomitantly:

 Tension pneumothorax
 Spinal cord injury
 Myocardial contusion
 Cardiac tamponade

When assessing trauma patients it is worth remembering that in order to generate a


palpable femoral pulse an arterial pressure of >65mmHg is required.

Once bleeding is controlled and circulating volume normalised the levels of


transfusion should be to maintain a Hb of 7-8 in those with no risk factors for tissue
hypoxia and Hb 10 for those who have such risk factors.

Neurogenic shock
This occurs most often following a spinal cord transection, usually at a high level.
There is resultant interruption of the autonomic nervous system. The result is either
decreased sympathetic tone or increased parasympathetic tone, the effect of
which is a decrease in peripheral vascular resistance mediated by marked
vasodilation.

This results in decreased preload and thus decreased cardiac output (Starlings law).
There is decreased peripheral tissue perfusion and shock is thus produced. In contrast
with many other types of shock peripheral vasoconstrictors are used to return vascular
tone to normal.

Cardiogenic shock
In medical patients the main cause is ischaemic heart disease. In the traumatic
setting direct myocardial trauma or contusion is more likely. Evidence of ECG
changes and overlying sternal fractures or contusions should raise the suspicion of
injury. Treatment is largely supportive and transthoracic echocardiography should be
used to determine evidence of pericardial fluid or direct myocardial injury. The
measurement of troponin levels in trauma patients may be undertaken but they are
less useful in delineating the extent of myocardial trauma than following MI.

When cardiac injury is of a blunt nature and is associated with cardiogenic shock the
right side of the heart is the most likely site of injury with chamber and or valve
rupture. These patients require surgery to repair these defects and will require
cardiopulmonary bypass to achieve this. Some may require intra aortic balloon pump
as a bridge to surgery.

Anaphylactic shock
Anaphylaxis may be defined as a severe, life-threatening, generalised or systemic
hypersensitivity reaction.

Anaphylaxis is one of the few times when you would not have time to look up the
dose of a medication. The Resuscitation Council guidelines on anaphylaxis have
recently been updated. Adrenaline is by far the most important drug in anaphylaxis
and should be given as soon as possible. The recommended doses for adrenaline,
hydrocortisone and chlorphenamine are as follows:

Adrenaline Hydrocortisone Chlorphenamine


< 6 months 150 mcg (0.15ml 1 in 25 mg 250 mcg/kg
1,000)
6 months - 6 years 150 mcg (0.15ml 1 in 50 mg 2.5 mg
1,000)
6-12 years 300 mcg (0.3ml 1 in 100 mg 5 mg
1,000)
Adult and child 12 500 mcg (0.5ml 1 in 200 mg 10 mg
years 1,000)

Adrenaline can be repeated every 5 minutes if necessary. The best site for IM
injection is the anterolateral aspect of the middle third of the thigh.

Common identified causes of anaphylaxis

 food (e.g. Nuts) - the most common cause in children


 drugs
 venom (e.g. Wasp sting)

Which of the following drugs causes hyperkalaemia?

A. Heparin

B. Ciprofloxacin

C. Salbutamol

D. Levothyroxine

E. Codeine phosphate

Both unfractionated and low-molecular weight heparin can cause hyperkalaemia. This
is thought to be caused by inhibition of aldosterone secretion. Salbutamol is a
recognised treatment for hyperkalaemia.

Hyperkalaemia

 Plasma potassium levels are regulated by a number of factors including


aldosterone, acid-base balance and insulin levels.
 Metabolic acidosis is associated with hyperkalaemia as hydrogen and
potassium ions compete with each other for exchange with sodium ions across
cell membranes and in the distal tubule.
 ECG changes seen in hyperkalaemia include tall-tented T waves, small P
waves, widened QRS leading to a sinusoidal pattern and asystole

Causes of hyperkalaemia

 Acute renal failure


 Drugs*: potassium sparing diuretics, ACE inhibitors, angiotensin 2 receptor
blockers, spironolactone, ciclosporin, heparin**
 Metabolic acidosis
 Addison's
 Tissue necrosis/rhabdomylosis: burns, trauma
 Massive blood transfusion

Foods that are high in potassium

 Salt substitutes (i.e. Contain potassium rather than sodium)


 Bananas, oranges, kiwi fruit, avocado, spinach, tomatoes

*beta-blockers interfere with potassium transport into cells and can potentially cause
hyperkalaemia in renal failure patients - remember beta-agonists, e.g. Salbutamol, are
sometimes used as emergency treatment

**both unfractionated and low-molecular weight heparin can cause hyperkalaemia.


This is thought to be caused by inhibition of aldosterone secretion
A 25-year-old man who has been morbidly obese for the past five years is reviewed in
the surgical bariatric clinic. In this patient, release of which of the following
hormones would increase appetite?

A. Leptin

B. Thyroxine

C. Adiponectin

D. Ghrelin

E. Serotonin
Obesity hormones

 leptin decreases
appetite
 ghrelin increases
appetite
Whilst thyroxine can increase appetite it does not fit with the clinical picture being
described.

Obesity: physiology

Leptin
Leptin is thought to play a key role in the regulation of body weight. It is produced by
adipose tissue and acts on satiety centres in the hypothalamus and decreases appetite.
More adipose tissue (e.g. in obesity) results in high leptin levels.

Leptin stimulates the release of melanocyte-stimulating hormone (MSH) and


corticotrophin-releasing hormone (CRH). Low levels of leptin stimulates the release
of neuropeptide Y (NPY)

Ghrelin
Where as leptin induces satiety, ghrelin stimulates hunger. It is produced mainly by
the fundus of the stomach and the pancreas. Ghrelin levels increase before meals and
decrease after meals
Which of the following bony complications is not linked to excess glucocorticoids?

A. Avascular necrosis

B. Vertebral body collapse

C. Increased susceptibility to osteomyelitis from strep viridans

D. Decreased absorption of calcium from the gut

E. Growth retardation in children

This infection is not typical of steroid excess, although general increased susceptibilty
to infections is.

Cortisol

 Glucocorticoid
 Released by zona fasiculata of the adrenal gland
 90% protein bound; 10% active
 Circadian rhythm: High in the mornings
 Negative feedback via ACTH

Actions

 Glycogenolysis
 Glucaneogenesis
 Protein catabolism
 Lipolysis
 Stress response
 Anti-inflammatory
 Decrease protein in bones
 Increase gastric acid
 Increases neutrophils/platelets/red blood cells
 Inhibits fibroblastic activity

A 54-year-old woman is admitted to the Surgical Admissions Unit with abdominal


pain. Blood tests taken on admission show the following:

Magnesium 0.40 mmol/l

Which one of the following factors is most likely to be responsible for this result?

A. Excessive resuscitation with intravenous saline

B. Digoxin therapy

C. Diarrhoea

D. Hypothermia

E. Rhabdomyolysis

Hypomagnasaemia

Cause of low magnesium

 Diuretics
 Total parenteral nutrition
 Diarrhoea
 Alcohol
 Hypokalaemia, hypocalcaemia

Features

 Paraesthesia
 Tetany
 Seizures
 Arrhythmias
 Decreased PTH secretion --> hypocalcaemia
 ECG features similar to those of hypokalaemia
 Exacerbates digoxin toxicity

A 43 year old man has a nasogastric tube inserted. The nurse takes a small aspirate of the
fluid from the stomach and tests the pH of the aspirate. What is the normal intragastric pH?

A. 0.5

B. 2

C. 4

D. 5

E. 6

Theme from January 2012 Exam


The intragastric pH is usually 2. Administration of proton pump inhibitors can result in
almost complete abolition of acidity

Gastric secretions

A working knowledge of gastric secretions is important for surgery because peptic ulcers are
common, surgeons frequently prescribe anti secretory drugs and because there are still
patients around who will have undergone acid lowering procedures (Vagotomy) in the past.

Gastric acid

 Is produced by the parietal cells in the stomach


 pH of gastric acid is around 2 with acidity being maintained by the H+/K+ ATP ase
pump. As part of the process bicarbonate ions will be secreted into the surrounding
vessels.
 Sodium and chloride ions are actively secreted from the parietal cell into the
canaliculus. This sets up a negative potential across the membrane and as a result
sodium and potassium ions diffuse across into the canaliculus.
 Carbonic anhydrase forms carbonic acid which dissociates and the hydrogen ions
formed by dissociation leave the cell via the H+/K+ antiporter pump. At the same
time sodium ions are actively absorbed. This leaves hydrogen and chloride ions in
the canaliculus these mix and are secreted into the lumen of the oxyntic gland.

This is illustrated diagrammatically below:


Image sourced from Wikipedia

Phases of gastric acid secretion


There are 3 phases of gastric secretion:

1. Cephalic phase (smell / taste of food)

 30% acid produced


 Vagal cholinergic stimulation causing secretion of HCL and gastrin release from G
cells

2. Gastric phase (distension of stomach )

 60% acid produced


 Stomach distension/low H+/peptides causes Gastrin release

3. Intestinal phase (food in duodenum)


 10% acid produced
 High acidity/distension/hypertonic solutions in the duodenum inhibits gastric acid
secretion via enterogastrones (CCK, secretin) and neural reflexes.

Regulation of gastric acid production


Factors increasing production include:

 Vagal nerve stimulation


 Gastrin release
 Histamine release (indirectly following gastrin release) from enterchromaffin like
cells

Factors decreasing production include:

 Somatostatin (inhibits histamine release)


 Cholecystokinin
 Secretin

The diagram below illustrates some of the factors involved in regulating gastric acid
secretion and the relevant associated pharmacology

Image sourced from Wikipedia

Below is a brief summary of the major hormones involved in food digestion:

Source Stimulus Actions


Gastrin G cells in Distension of Increase HCL, pepsinogen and IF secretion,
antrum of stomach, extrinsic increases gastric motility, trophic effect on
the stomach nerves gastric mucosa
Inhibited by: low
antral pH,
somatostatin

CCK I cells in Partially digested Increases secretion of enzyme-rich fluid


upper small proteins and from pancreas, contraction of gallbladder
intestine triglycerides and relaxation of sphincter of Oddi,
decreases gastric emptying, trophic effect
on pancreatic acinar cells, induces satiety

Secretin S cells in Acidic chyme, fatty Increases secretion of bicarbonate-rich fluid


upper small acids from pancreas and hepatic duct cells,
intestine decreases gastric acid secretion, trophic
effect on pancreatic acinar cells

VIP Small Neural Stimulates secretion by pancreas and


intestine, intestines, inhibits acid and pepsinogen
pancreas secretion

Somatostatin D cells in the Fat, bile salts and Decreases acid and pepsin secretion,
pancreas and glucose in the decreases gastrin secretion, decreases
stomach intestinal lumen pancreatic enzyme secretion, decreases
insulin and glucagon secretion
inhibits trophic effects of gastrin, stimulates
gastric mucous production

Which of the following is the equivalent of cardiac preload?

A. End diastolic volume

B. Stroke volume

C. Systemic vascular resistance

D. Mean arterial pressure

E. Peak systolic arterial pressure


Preload is the same as end diastolic volume. When it is increased slightly there is an
associated increase in cardiac output (Frank Starling principle). When it is markedly
increased e.g. over 250ml then cardiac output falls.

Cardiac physiology

 The heart has four chambers ejecting blood into both low pressure and high
pressure systems.
 The pumps generate pressures of between 0-25mmHg on the right side and 0-120
mmHg on the left.
 At rest diastole comprises 2/3 of the cardiac cycle.
 The product of the frequency of heart rate and stroke volume combine to give the
cardiac output which is typically 5-6L per minute.

Detailed descriptions of the various waveforms are often not a feature of MRCS A (although
they are on the syllabus). However, they are a very popular topic for surgical physiology
vivas in the oral examination.

Electrical properties

 Intrinsic myogenic rhythm within cardiac myocytes means that even the denervated
heart is capable of contraction.
 In the normal situation the cardiac impulse is generated in the sino atrial node in the
right atrium and conveyed to the ventricles via the atrioventricular node.
 The sino atrial node is also capable of spontaneous discharge and in the absence of
background vagal tone will typically discharge around 100x per minute. Hence the
higher resting heart rate found in cardiac transplant cases. In the SA and AV nodes
the resting membrane potential is lower than in surrounding cardiac cells and will
slowly depolarise from -70mV to around -50mV at which point an action potential is
generated.
 Differences in the depolarisation slopes between SA and AV nodes help to explain
why the SA node will depolarise first. The cells have a refractory period during which
they cannot be re-stimulated and this period allows for adequate ventricular filling.
In pathological tachycardic states this time period is overridden and inadequate
ventricular filling may then occur, cardiac output falls and syncope may ensue.

Parasympathetic fibres project to the heart via the vagus and will release acetylcholine.
Sympathetic fibres release nor adrenaline and circulating adrenaline comes from the adrenal
medulla. Noradrenaline binds to β 1 receptors in the SA node and increases the rate of
pacemaker potential depolarisation.

Cardiac cycle
Image sourced from Wikipedia

 Mid diastole: AV valves open. Ventricles hold 80% of final volume. Outflow valves
shut. Aortic pressure is high.

 Late diastole: Atria contract. Ventricles receive 20% to complete filling. Typical end
diastolic volume 130-160ml.

 Early systole: AV valves shut. Ventricular pressure rises. Isovolumetric ventricular


contraction. AV Valves bulge into atria (c-wave). Aortic and pulmonary pressure
exceeded- blood is ejected. Shortening of ventricles pulls atria downwards and
drops intra atrial pressure (x-descent).

 Late systole: Ventricular muscles relax and ventricular pressures drop. Although
ventricular pressure drops the aortic pressure remains constant owing to peripheral
vascular resistance and elastic property of the aorta. Brief period of retrograde flow
that occurs in aortic recoil shuts the aortic valve. Ventricles will contain 60ml end
systolic volume. The average stroke volume is 70ml (i.e. Volume ejected).

 Early diastole: All valves are closed. Isovolumetric ventricular relaxation occurs.
Pressure wave associated with closure of the aortic valve increases aortic pressure.
The pressure dip before this rise can be seen on arterial waveforms and is called the
incisura. During systole the atrial pressure increases such that it is now above zero
(v- wave). Eventually atrial pressure exceed ventricular pressure and AV valves open
- atria empty passively into ventricles and atrial pressure falls (y -descent )

The negative atrial pressures are of clinical importance as they can allow air embolization to
occur if the neck veins are exposed to air. This patient positioning is important in head and
neck surgery to avoid this occurrence if veins are inadvertently cut, or during CVP line
insertion.

Mechanical properties

 Preload = end diastolic volume


 Afterload = aortic pressure

It is important to understand the principles of Laplace's law in surgery.

 It states that for hollow organs with a circular cross section, the total circumferential
wall tension depends upon the circumference of the wall, multiplied by the
thickness of the wall and on the wall tension.
 The total luminal pressure depends upon the cross sectional area of the lumen and
the transmural pressure. Transmural pressure is the internal pressure minus
external pressure and at equilibrium the total pressure must counterbalance each
other.
 In terms of cardiac physiology the law explains that the rise in ventricular pressure
that occurs during the ejection phase is due to physical change in heart size. It also
explains why a dilated diseased heart will have impaired systolic function.

Starlings law

 Increase in end diastolic volume will produce larger stroke volume.


 This occurs up to a point beyond which cardiac fibres are excessively stretched and
stroke volume will fall once more. It is important for the regulation of cardiac output
in cardiac transplant patients who need to increase their cardiac output.

Baroreceptor reflexes

 Baroreceptors located in aortic arch and carotid sinus.


 Aortic baroreceptor impulses travel via the vagus and from the carotid via the
glossopharyngeal nerve.
 They are stimulated by arterial stretch.
 Even at normal blood pressures they are tonically active.
 Increase in baroreceptor discharge causes:

*Increased parasympathetic discharge to the SA node.


*Decreased sympathetic discharge to ventricular muscle causing decreased contractility and
fall in stroke volume.
*Decreased sympathetic discharge to venous system causing increased compliance.
*Decreased peripheral arterial vascular resistance

Atrial stretch receptors

 Located in atria at junction between pulmonary veins and vena cava.


 Stimulated by atrial stretch and are thus low pressure sensors.
 Increased blood volume will cause increased parasympathetic activity.
 Very rapid infusion of blood will result in increase in heart rate mediated via atrial
receptors: the Bainbridge reflex.
 Decreases in receptor stimulation results in increased sympathetic activity this will
decrease renal blood flow-decreases GFR-decreases urinary sodium excretion-renin
secretion by juxtaglomerular apparatus-Increase in angiotensin II.
 Increased atrial stretch will also result in increased release of atrial natriuretic
peptide.
 A 73 year old female is referred to the surgical clinic with an iron deficiency
anaemia. As part of the diagnostic work up the doctor requests a serum ferritin
level. Which of the conditions listed is most likely to lead to a falsely elevated
result?

A. Locally perforated sigmoid colonic adenocarcinoma

B. Colonic angiodysplasia

C. Dieulafoy lesion of the stomach

D. Transitional cell carcinoma of the bladder

E. Endometrial adenocarcinoma

A locally perforated colonic tumour will typically cause an intense
inflammatory response and if peritonitis is not present clinically then at the
very least a localised abscess. This inflammatory process is the most likely
(from the list) to falsely raise the serum ferritin level. Angiodysplasia and
dieulafoy lesions are mucosal arteriovenous malformations and unlikely to
result in considerable inflammatory activity.
 Ferritin

Ferritin is an intracellular protein that binds iron and stores it to be released in
a controlled fashion at sites where iron is required. Because iron and ferritin
are bound the total body ferritin levels may be decreased in cases of iron
deficiency anaemia. Measurement of serum ferritin levels can be useful in
determining whether an apparently low haemoglobin and microcytosis is truly
caused by an iron deficiency state.
Ferritin is an acute phase protein and may be synthesised in increased
quantities in situations where inflammatory activity is ongoing. Falsely
elevated results may therefore be encountered clinically and need to be taken
in context of the clinical picture and full blood count results.
 Which of the following is least likely to cause a prolonged prothrombin time?

A. Cholestatic jaundice

B. Disseminated intravascular coagulation

C. Prolonged antibiotic treatment

D. Liver disease

E. Acquired factor 12 deficiency



Vitamin K deficiency results from cholestatic jaundice and prolonged
antibiotic therapy. Acquired factor 12 deficiency causes prolonged APTT.
 Abnormal coagulation

Cause Factors affected
Heparin Prevents activation factors 2,9,10,11
Warfarin Affects synthesis of factors 2,7,9,10
DIC Factors 1,2,5,8,11
Liver disease Factors 1,2,5,7,9,10

Interpretation blood clotting test results
Disorder APTT PT Bleeding time
Haemophilia Increased Normal Normal
von Willebrand's disease Increased Normal Increased
Vitamin K deficiency Increased Increased Normal

Which statement about peristalsis is true?

A. Longitudinal smooth muscle propels the food bolus through the


oesophagus

B. Secondary peristalsis occurs when there is no food bolus in the


oesophagus

C. Food transfer from the oesophagus to the stomach is 4 seconds

D. Circular smooth muscle is not involved in peristalsis

E. Peristalsis only occurs in the oesophagus


Peristalsis

 Circular smooth muscle contracts behind the food bolus and longitudinal
smooth muscle propels the food through the oesophagus
 Primary peristalsis spontaneously moves the food from the oesophagus into
the stomach (9 seconds)
 Secondary peristalsis occurs when food, which doesn't enter the stomach,
stimulates stretch receptors to cause peristalsis
 In the small intestine each peristalsis waves slows to a few seconds and causes
mixture of chyme
 In the colon three main types of peristaltic activity are recognised (see below)

Colonic peristalsis
Segmentation Localised contractions in which the bolus is subjected to
contractions local forces to maximise mucosal absorption
Antiperistaltic Localised reverse peristaltic waves to slow entry into
contractions towards colon and maximise absorption
ileum
Mass movements Waves migratory peristaltic waves along the entire colon
to empty the organ prior to the next ingestion of food
bolus
24 year old man is injured in a road traffic accident. He becomes oliguric and his
renal function deteriorates. Which of the options below would favor acute tubular
necrosis over pre renal uraemia?

A. No response to intravenous fluids

B. Urinary sodium < 20mmol/L

C. Bland coloured urinary sediment

D. Increased urine specific gravity

E. None of the above

In acute tubular necrosis there is no response to intravenous fluids because the


damage occurs from within the renal system rather than as a result of volume
depletion.

Acute renal failure: Pre renal failure vs. acute tubular necrosis

Prerenal uraemia - kidneys retain sodium to preserve volume


Pre-renal uraemia Acute tubular necrosis
Urine sodium < 20 mmol/L > 30 mmol/L
Fractional sodium excretion* < 1% > 1%
Fractional urea excretion** < 35% >35%
Urine:plasma osmolality > 1.5 < 1.1
Urine:plasma urea > 10:1 < 8:1
Specific gravity > 1020 < 1010
Urine 'bland' sediment brown granular casts
Response to fluid challenge Yes No

*fractional sodium excretion = (urine sodium/plasma sodium) / (urine


creatinine/plasma creatinine) x 100

**fractional urea excretion = (urine urea /blood urea ) / (urine creatinine/plasma


creatinine) x 100
hich of the following is not an effect of cholecystokinin?

A. It causes gallbladder contraction

B. It increases the rate of gastric emptying

C. It relaxes the sphincter of oddi

D. It stimulates pancreatic acinar cells

E. It has a trophic effect on pancreatic acinar cells

It decreases the rate of gastric emptying.

Gastric secretions

A working knowledge of gastric secretions is important for surgery because peptic ulcers are
common, surgeons frequently prescribe anti secretory drugs and because there are still
patients around who will have undergone acid lowering procedures (Vagotomy) in the past.

Gastric acid

 Is produced by the parietal cells in the stomach


 pH of gastric acid is around 2 with acidity being maintained by the H+/K+ ATP ase
pump. As part of the process bicarbonate ions will be secreted into the surrounding
vessels.
 Sodium and chloride ions are actively secreted from the parietal cell into the
canaliculus. This sets up a negative potential across the membrane and as a result
sodium and potassium ions diffuse across into the canaliculus.
 Carbonic anhydrase forms carbonic acid which dissociates and the hydrogen ions
formed by dissociation leave the cell via the H+/K+ antiporter pump. At the same
time sodium ions are actively absorbed. This leaves hydrogen and chloride ions in
the canaliculus these mix and are secreted into the lumen of the oxyntic gland.

This is illustrated diagrammatically below:

Image sourced from Wikipedia

Phases of gastric acid secretion


There are 3 phases of gastric secretion:

1. Cephalic phase (smell / taste of food)


 30% acid produced
 Vagal cholinergic stimulation causing secretion of HCL and gastrin release from G
cells

2. Gastric phase (distension of stomach )

 60% acid produced


 Stomach distension/low H+/peptides causes Gastrin release

3. Intestinal phase (food in duodenum)

 10% acid produced


 High acidity/distension/hypertonic solutions in the duodenum inhibits gastric acid
secretion via enterogastrones (CCK, secretin) and neural reflexes.

Regulation of gastric acid production


Factors increasing production include:

 Vagal nerve stimulation


 Gastrin release
 Histamine release (indirectly following gastrin release) from enterchromaffin like
cells

Factors decreasing production include:

 Somatostatin (inhibits histamine release)


 Cholecystokinin
 Secretin

The diagram below illustrates some of the factors involved in regulating gastric acid
secretion and the relevant associated pharmacology
Image sourced from Wikipedia

Below is a brief summary of the major hormones involved in food digestion:

Source Stimulus Actions

Gastrin G cells in Distension of Increase HCL, pepsinogen and IF secretion,


antrum of stomach, extrinsic increases gastric motility, trophic effect on
the stomach nerves gastric mucosa
Inhibited by: low
antral pH,
somatostatin

CCK I cells in Partially digested Increases secretion of enzyme-rich fluid


upper small proteins and from pancreas, contraction of gallbladder
intestine triglycerides and relaxation of sphincter of Oddi,
decreases gastric emptying, trophic effect
on pancreatic acinar cells, induces satiety

Secretin S cells in Acidic chyme, fatty Increases secretion of bicarbonate-rich fluid


upper small acids from pancreas and hepatic duct cells,
intestine decreases gastric acid secretion, trophic
effect on pancreatic acinar cells

VIP Small Neural Stimulates secretion by pancreas and


intestine, intestines, inhibits acid and pepsinogen
pancreas secretion
Somatostatin D cells in the Fat, bile salts and Decreases acid and pepsin secretion,
pancreas and glucose in the decreases gastrin secretion, decreases
stomach intestinal lumen pancreatic enzyme secretion, decreases
insulin and glucagon secretion
inhibits trophic effects of gastrin, stimulates
gastric mucous production

Which of the following is not an effect of cholecystokinin?

A. It causes gallbladder contraction

B. It increases the rate of gastric emptying

C. It relaxes the sphincter of oddi

D. It stimulates pancreatic acinar cells

E. It has a trophic effect on pancreatic acinar cells

It decreases the rate of gastric emptying.

Gastric secretions

A working knowledge of gastric secretions is important for surgery because peptic ulcers are
common, surgeons frequently prescribe anti secretory drugs and because there are still
patients around who will have undergone acid lowering procedures (Vagotomy) in the past.

Gastric acid

 Is produced by the parietal cells in the stomach


 pH of gastric acid is around 2 with acidity being maintained by the H+/K+ ATP ase
pump. As part of the process bicarbonate ions will be secreted into the surrounding
vessels.
 Sodium and chloride ions are actively secreted from the parietal cell into the
canaliculus. This sets up a negative potential across the membrane and as a result
sodium and potassium ions diffuse across into the canaliculus.
 Carbonic anhydrase forms carbonic acid which dissociates and the hydrogen ions
formed by dissociation leave the cell via the H+/K+ antiporter pump. At the same
time sodium ions are actively absorbed. This leaves hydrogen and chloride ions in
the canaliculus these mix and are secreted into the lumen of the oxyntic gland.
This is illustrated diagrammatically below:

Image sourced from Wikipedia

Phases of gastric acid secretion


There are 3 phases of gastric secretion:

1. Cephalic phase (smell / taste of food)

 30% acid produced


 Vagal cholinergic stimulation causing secretion of HCL and gastrin release from G
cells

2. Gastric phase (distension of stomach )

 60% acid produced


 Stomach distension/low H+/peptides causes Gastrin release
3. Intestinal phase (food in duodenum)

 10% acid produced


 High acidity/distension/hypertonic solutions in the duodenum inhibits gastric acid
secretion via enterogastrones (CCK, secretin) and neural reflexes.

Regulation of gastric acid production


Factors increasing production include:

 Vagal nerve stimulation


 Gastrin release
 Histamine release (indirectly following gastrin release) from enterchromaffin like
cells

Factors decreasing production include:

 Somatostatin (inhibits histamine release)


 Cholecystokinin
 Secretin

The diagram below illustrates some of the factors involved in regulating gastric acid
secretion and the relevant associated pharmacology

Image sourced from Wikipedia

Below is a brief summary of the major hormones involved in food digestion:


Source Stimulus Actions

Gastrin G cells in Distension of Increase HCL, pepsinogen and IF secretion,


antrum of stomach, extrinsic increases gastric motility, trophic effect on
the stomach nerves gastric mucosa
Inhibited by: low
antral pH,
somatostatin

CCK I cells in Partially digested Increases secretion of enzyme-rich fluid


upper small proteins and from pancreas, contraction of gallbladder
intestine triglycerides and relaxation of sphincter of Oddi,
decreases gastric emptying, trophic effect
on pancreatic acinar cells, induces satiety

Secretin S cells in Acidic chyme, fatty Increases secretion of bicarbonate-rich fluid


upper small acids from pancreas and hepatic duct cells,
intestine decreases gastric acid secretion, trophic
effect on pancreatic acinar cells

VIP Small Neural Stimulates secretion by pancreas and


intestine, intestines, inhibits acid and pepsinogen
pancreas secretion

Somatostatin D cells in the Fat, bile salts and Decreases acid and pepsin secretion,
pancreas and glucose in the decreases gastrin secretion, decreases
stomach intestinal lumen pancreatic enzyme secretion, decreases
insulin and glucagon secretion
inhibits trophic effects of gastrin, stimulates
gastric mucous production

Which part of the jugular venous waveform is associated with the closure of the
tricuspid valve?

A. a wave

B. c wave

C. x descent

D. y descent

E. v wave
JVP: {C} wave - {c}losure of
the tricuspid valve

The c wave of the jugular venous waveform is associated with the closure of the
tricuspid valve.

Jugular venous pressure

As well as providing information on right atrial pressure, the jugular vein waveform
may provide clues to underlying valvular disease. A non-pulsatile JVP is seen in
superior vena caval obstruction. Kussmaul's sign describes a paradoxical rise in JVP
during inspiration seen in constrictive pericarditis

'a' wave = atrial contraction

 large if atrial pressure e.g. tricuspid stenosis, pulmonary stenosis, pulmonary


hypertension
 absent if in atrial fibrillation

Cannon 'a' waves

 caused by atrial contractions against a closed tricuspid valve


 are seen in complete heart block, ventricular tachycardia/ectopics, nodal
rhythm, single chamber ventricular pacing

'c' wave

 closure of tricuspid valve


 not normally visible

'v' wave

 due to passive filling of blood into the atrium against a closed tricuspid valve
 giant v waves in tricuspid regurgitation

'x' descent = fall in atrial pressure during ventricular systole

'y' descent = opening of tricuspid valve


Which part of the jugular venous waveform is associated with the closure of the
tricuspid valve?

A. a wave

B. c wave
C. x descent

D. y descent

E. v wave
JVP: {C} wave - {c}losure of
the tricuspid valve

The c wave of the jugular venous waveform is associated with the closure of the
tricuspid valve.

Jugular venous pressure

As well as providing information on right atrial pressure, the jugular vein waveform
may provide clues to underlying valvular disease. A non-pulsatile JVP is seen in
superior vena caval obstruction. Kussmaul's sign describes a paradoxical rise in JVP
during inspiration seen in constrictive pericarditis

'a' wave = atrial contraction

 large if atrial pressure e.g. tricuspid stenosis, pulmonary stenosis, pulmonary


hypertension
 absent if in atrial fibrillation

Cannon 'a' waves

 caused by atrial contractions against a closed tricuspid valve


 are seen in complete heart block, ventricular tachycardia/ectopics, nodal
rhythm, single chamber ventricular pacing

'c' wave

 closure of tricuspid valve


 not normally visible

'v' wave

 due to passive filling of blood into the atrium against a closed tricuspid valve
 giant v waves in tricuspid regurgitation

'x' descent = fall in atrial pressure during ventricular systole

'y' descent = opening of tricuspid valve


Which one of the following serum proteins is most likely to increase in a patient with
severe sepsis?

A. Transferrin

B. Transthyretin

C. Ferritin

D. Albumin

E. Cortisol binding protein

Ferritin can be markedly increased during acute illness. The other parameters tend to
decrease during an acute phase response.

Acute phase proteins

Acute phase proteins

 CRP
 procalcitonin
 ferritin
 fibrinogen
 alpha-1 antitrypsin
 caeruloplasmin
 serum amyloid A
 haptoglobin
 complement

During the acute phase response the liver decreases the production of other proteins
(sometimes referred to as negative acute phase proteins). Examples include:

 albumin
 transthyretin (formerly known as prealbumin)
 transferrin
 retinol binding protein
 cortisol binding protein
 Theme: Critical care

A. Hypovolaemia
B. Normal
C. Cardiogenic shock
D. Septic shock

For each of the scenarios outlined in the tables below, please select the most
likely diagnosis from the list. Each option may be used once, more than once
or not at all.

17. A 45 year old man is admitted to the intensive care unit following a
laparotomy. He has a central line, pulmonary artery catheter and arterial lines
inserted. The following results are obtained:
Pulmonary artery occlusion Cardiac Systemic vascular
pressure output resistance
Low Low High

Hypovolaemia

Theme from April 2012 Exam


Cardiac output is lowered in hypovolaemia due to decreased preload.

18. A 75 year old man is admitted to the intensive care unit following a
laparotomy. He has a central line, pulmonary artery catheter and arterial lines
inserted. The following results are obtained:
Pulmonary artery occlusion Cardiac Systemic vascular
pressure output resistance
High Low High

Cardiogenic shock

In cardiogenic shock pulmonary pressures are often high. This is the basis for
the use of venodilators in the treatment of pulmonary oedema.

19. A 22 year old lady is admitted to the intensive care unit following a
laparotomy. She has a central line, pulmonary artery catheter and arterial lines
inserted. The following results are obtained:
Pulmonary artery occlusion Cardiac Systemic vascular
pressure output resistance
Low High Low
Septic shock

Decreased SVR is a major feature of sepsis. A hyperdynamic circulation is


often present. This is the reason for the use of vasoconstrictors.


 Pulmonary artery occlusion pressure monitoring

The pulmonary artery occlusion pressure is an indirect measure of left atrial


pressure, and thus filling pressure of the left heart. The low resistance within
the pulmonary venous system allows this useful measurement to be made. The
most accurate trace is made by inflating the balloon at the catheter tip and
"floating" it so that it occludes the vessel. If it is not possible to occlude the
vessel in this way then the measurement gained will be the pulmonary artery
end diastolic pressure.

Interpretation of PAOP
PAOP mmHg Scenario
Normal 8-12
Low <5 Hypovolaemia
Low with pulmonary oedema <5 ARDS
High >18 Overload

When combined with measurements of systemic vascular resistance and
cardiac output it is possible to accurately classify patients.

Systemic vascular resistance


Derived from aortic pressure, right atrial pressure and cardiac output.

SVR=80(mean aortic pressure-mean right atrial pressure)/cardiac output


A 73 year old man has an arterial line in situ. On studying the trace the incisura can be seen.
What is the physiological event which accounts for this process?

A. Atrial repolarisation

B. Mitral valve closure

C. Ventricular repolarisation

D. Elastic recoil of the aorta


E. Tricuspid valve closure

Theme from 2010 Exam

It is the temporary rise in aortic pressure occurring as a result of elastic recoil.

Cardiac physiology

 The heart has four chambers ejecting blood into both low pressure and high
pressure systems.
 The pumps generate pressures of between 0-25mmHg on the right side and 0-120
mmHg on the left.
 At rest diastole comprises 2/3 of the cardiac cycle.
 The product of the frequency of heart rate and stroke volume combine to give the
cardiac output which is typically 5-6L per minute.

Detailed descriptions of the various waveforms are often not a feature of MRCS A (although
they are on the syllabus). However, they are a very popular topic for surgical physiology
vivas in the oral examination.

Electrical properties

 Intrinsic myogenic rhythm within cardiac myocytes means that even the denervated
heart is capable of contraction.
 In the normal situation the cardiac impulse is generated in the sino atrial node in the
right atrium and conveyed to the ventricles via the atrioventricular node.
 The sino atrial node is also capable of spontaneous discharge and in the absence of
background vagal tone will typically discharge around 100x per minute. Hence the
higher resting heart rate found in cardiac transplant cases. In the SA and AV nodes
the resting membrane potential is lower than in surrounding cardiac cells and will
slowly depolarise from -70mV to around -50mV at which point an action potential is
generated.
 Differences in the depolarisation slopes between SA and AV nodes help to explain
why the SA node will depolarise first. The cells have a refractory period during which
they cannot be re-stimulated and this period allows for adequate ventricular filling.
In pathological tachycardic states this time period is overridden and inadequate
ventricular filling may then occur, cardiac output falls and syncope may ensue.

Parasympathetic fibres project to the heart via the vagus and will release acetylcholine.
Sympathetic fibres release nor adrenaline and circulating adrenaline comes from the adrenal
medulla. Noradrenaline binds to β 1 receptors in the SA node and increases the rate of
pacemaker potential depolarisation.
Cardiac cycle

Image sourced from Wikipedia

 Mid diastole: AV valves open. Ventricles hold 80% of final volume. Outflow valves
shut. Aortic pressure is high.

 Late diastole: Atria contract. Ventricles receive 20% to complete filling. Typical end
diastolic volume 130-160ml.

 Early systole: AV valves shut. Ventricular pressure rises. Isovolumetric ventricular


contraction. AV Valves bulge into atria (c-wave). Aortic and pulmonary pressure
exceeded- blood is ejected. Shortening of ventricles pulls atria downwards and
drops intra atrial pressure (x-descent).

 Late systole: Ventricular muscles relax and ventricular pressures drop. Although
ventricular pressure drops the aortic pressure remains constant owing to peripheral
vascular resistance and elastic property of the aorta. Brief period of retrograde flow
that occurs in aortic recoil shuts the aortic valve. Ventricles will contain 60ml end
systolic volume. The average stroke volume is 70ml (i.e. Volume ejected).
 Early diastole: All valves are closed. Isovolumetric ventricular relaxation occurs.
Pressure wave associated with closure of the aortic valve increases aortic pressure.
The pressure dip before this rise can be seen on arterial waveforms and is called the
incisura. During systole the atrial pressure increases such that it is now above zero
(v- wave). Eventually atrial pressure exceed ventricular pressure and AV valves open
- atria empty passively into ventricles and atrial pressure falls (y -descent )

The negative atrial pressures are of clinical importance as they can allow air embolization to
occur if the neck veins are exposed to air. This patient positioning is important in head and
neck surgery to avoid this occurrence if veins are inadvertently cut, or during CVP line
insertion.

Mechanical properties

 Preload = end diastolic volume


 Afterload = aortic pressure

It is important to understand the principles of Laplace's law in surgery.

 It states that for hollow organs with a circular cross section, the total circumferential
wall tension depends upon the circumference of the wall, multiplied by the
thickness of the wall and on the wall tension.
 The total luminal pressure depends upon the cross sectional area of the lumen and
the transmural pressure. Transmural pressure is the internal pressure minus
external pressure and at equilibrium the total pressure must counterbalance each
other.
 In terms of cardiac physiology the law explains that the rise in ventricular pressure
that occurs during the ejection phase is due to physical change in heart size. It also
explains why a dilated diseased heart will have impaired systolic function.

Starlings law

 Increase in end diastolic volume will produce larger stroke volume.


 This occurs up to a point beyond which cardiac fibres are excessively stretched and
stroke volume will fall once more. It is important for the regulation of cardiac output
in cardiac transplant patients who need to increase their cardiac output.

Baroreceptor reflexes

 Baroreceptors located in aortic arch and carotid sinus.


 Aortic baroreceptor impulses travel via the vagus and from the carotid via the
glossopharyngeal nerve.
 They are stimulated by arterial stretch.
 Even at normal blood pressures they are tonically active.
 Increase in baroreceptor discharge causes:
*Increased parasympathetic discharge to the SA node.
*Decreased sympathetic discharge to ventricular muscle causing decreased contractility and
fall in stroke volume.
*Decreased sympathetic discharge to venous system causing increased compliance.
*Decreased peripheral arterial vascular resistance

Atrial stretch receptors

 Located in atria at junction between pulmonary veins and vena cava.


 Stimulated by atrial stretch and are thus low pressure sensors.
 Increased blood volume will cause increased parasympathetic activity.
 Very rapid infusion of blood will result in increase in heart rate mediated via atrial
receptors: the Bainbridge reflex.
 Decreases in receptor stimulation results in increased sympathetic activity this will
decrease renal blood flow-decreases GFR-decreases urinary sodium excretion-renin
secretion by juxtaglomerular apparatus-Increase in angiotensin II.
 Increased atrial stretch will also result in increased release of atrial natriuretic
peptide.

A 72-year-old woman is admitted to the acute surgical unit with profuse vomiting.
Admission bloods show the following:

Na+ 131 mmol/l


K+ 2.2 mmol/l
Urea 3.1 mmol/l
Creatinine 56 µmol/l
Glucose 4.3 mmol/l

Which one of the following ECG features is most likely to be seen?

A. Short PR interval

B. Short QT interval

C. Flattened P waves

D. J waves

E. U waves
Hypokalaemia - U waves
on ECG

J waves are seen in hypothermia whilst delta waves are associated with Wolff
Parkinson White syndrome.
ECG features in hypokalemia

 U waves
 Small or absent T waves (occasionally inversion)
 Prolonged PR interval
 ST depression
 Long QT interval

One registered user suggests the following rhyme!

 In Hypokalaemia, U have no Pot and no T, but a long PR and a long QT!


 Which of the following secretions contains the highest composition of
potassium?

A. Bile

B. Small bowel

C. Pancreatic juice

D. Gastric juice

E. Saliva

Of the secretions shown above, saliva has the greatest composition of
potassium. The exact amount secreted will depend upon aldosterone levels.
 Potassium composition of secretions

Saliva 20-60 mmol/L
Gastric juice 5-10 mmol/L
Bile 5-8 mmol/L
Pancreatic juice 4-5 mmol/L
Small bowel 4-10 mmol/L
The oxygen-haemoglobin dissociation curve is shifted to the left in:

A. With decreased 2,3-DPG in transfused red cells

B. Respiratory acidosis
C. High altitude

D. Pyrexia

E. Haemolytic anaemia

S shaped
curve

The curve is shifted to the left when there is a decreased oxygen requirement by the tissue.
This includes:
1. Hypothermia
2. Alkalosis
3. Reduced levels of DPG:

 DPG is found in erythrocytes and is reduced in non exercising muscles, i.e. when
there is reduced glycolysis.

4. Polycythaemia

Oxygen Transport

Oxygen transport
Almost all oxygen is transported within erythrocytes. It has limited solubility and only 1% is
carried as solution. Therefore the amount of oxygen transported will depend upon
haemoglobin concentration and its degree of saturation.

Haemoglobin
Globular protein composed of 4 subunits. Haem consists of a protoporphyrin ring
surrounding an iron atom in its ferrous state. The iron can form two additional bonds; one
with oxygen and the other with a polypeptide chain. There are two alpha and two beta
subunits to this polypeptide chain in an adult and together these form globin. Globin cannot
bind oxygen but is able to bind to carbon dioxide and hydrogen ions, the beta chains are
able to bind to 2,3 diphosphoglycerate. The oxygenation of haemoglobin is a reversible
reaction. The molecular shape of haemoglobin is such that binding of one oxygen molecule
facilitates the binding of subsequent molecules.

Oxygen dissociation curve

 The oxygen dissociation curve describes the relationship between the percentage of
saturated haemoglobin and partial pressure of oxygen in the blood. It is not affected
by haemoglobin concentration.
 Chronic anaemia causes 2, 3 DPG levels to increase, hence shifting the curve to the
right

Bohr effect

 Shifts to left = for given oxygen tension there is increased saturation of Hb with
oxygen i.e. Decreased oxygen delivery to tissues
 Shifts to right = for given oxygen tension there is reduced saturation of Hb with
oxygen i.e. Enhanced oxygen delivery to tissues

Image sourced from Wikipedia

Shifts to Left = Lower oxygen delivery Shifts to Right = Raised oxygen


delivery
 HbF, methaemoglobin, carboxyhaemoglobin
 low [H+] (alkali)  raised [H+] (acidic)
 low pCO2  raised pCO2
 low 2,3-DPG  raised 2,3-DPG*
 low temperature  raised temperature

*2,3-diphosphoglycerate
A homeless 42 year old male had an emergency inguinal hernia repair 24 hours
previously. He has a BMI of 15. He has been put on a feeding regime of 35
kcal/kg/day with no additional medications. The nursing staff contact you as he has
become confused and unsteady. On examination the patient is disorientated to place,
has diplopia and nystagmus. What is the most likely diagnosis?

A. Cerebellar stroke

B. Acute dystonic reaction

C. Refeeding syndrome

D. Parkinsonism

E. Wernickes encephalopathy
Triad of Wernicke
encephalopathy:

 Acute confusion
 Ataxia
 Opthalmoplegia

This patient has received a carbohydrate rich diet without any thiamine or vitamin B
co strong replacement. This has led to Wernickes encephalopathy, which classically
presents with confusion, ataxia and opthalmoplegia. Characteristically it is associated
with chronic alcoholism, however it is also known to occur post bariatric surgery.

Nutrition - Refeeding syndrome

Refeeding syndrome describes the metabolic abnormalities which occur on feeding a


person a starved state. The metabolic consequences include:

 Hypophosphataemia
 Hypokalaemia
 Hypomagnesaemia
 Abnormal fluid balance

These abnormalities can lead to organ failure.

Re-feeding problems
If patient not eaten for > 5 days, aim to re-feed at < 50% energy and protein levels

High risk for re-feeding problems


If one or more of the following:

 BMI < 16 kg/m2


 Unintentional weight loss >15% over 3-6 months
 Little nutritional intake > 10 days
 Hypokalaemia, Hypophosphataemia or hypomagnesaemia prior to feeding
(unless high)

If two or more of the following:

 BMI < 18.5 kg/m2


 Unintentional weight loss > 10% over 3-6 months
 Little nutritional intake > 5 days
 PMH alcohol abuse or drug therapy including insulin, chemotherapy,
diuretics, antacids

Prescription

 Start at up to 10 kcal/kg/day increasing to full needs over 4-7 days


 Start immediately before and during feeding: oral thiamine 200-300mg/day,
vitamin B co strong 1 tds and supplements
 Give K+ (2-4 mmol/kg/day), phosphate (0.3-0.6 mmol/kg/day), magnesium
(0.2-0.4 mmol/kg/day)
 A 22 year old lady receives intravenous morphine for acute abdominal pain.
Which of the following best accounts for its analgesic properties?

A. Binding to δ opioid receptors in the brainstem

B. Binding to δ opioid receptors at peripheral nerve sites

C. Binding to β opioid receptors within the CNS

D. Binding to α opioid receptors within the CNS

E. Binding to µ opioid receptors within the CNS



Theme from April 2012 Exam
 Morphine

Strong opiate analgesic. It is a pro- type narcotic drug and its effects mediated
via the µ opioid receptor. Its clinical effects stem from binding to these
receptor sites within the CNS and gastrointestinal tract. Unwanted side effects
include nausea, constipation, respiratory depression and, if used long term,
addiction.
It may be administered orally or intravenously. It can be reversed with
naloxone.
Which one of the following reduces the secretion of renin?
A. Erect posture

B. Adrenaline

C. Hyponatraemia

D. Hypotension

E. Beta-blockers

Renin

Renin is secreted by juxtaglomerular cells and hydrolyses angiotensinogen to produce


angiotensin I

Factors stimulating renin secretion

 hypotension causing reduced renal perfusion


 hyponatraemia
 sympathetic nerve stimulation
 catecholamines
 erect posture

Factors reducing renin secretion

 drugs: beta-blockers, NSAIDs


 Which of the following stimulates prolactin release?

A. Leutinising hormone

B. Dopamine

C. Thyrotropin releasing hormone

D. Oestrogen

E. Follicle stimulating hormone



TRH stimulates prolactin release. Dopamine suppresses the release of
prolactin.
 Prolactin

Prolactin is a peptide hormone released from the anterior pituitary. It is under
tonic dopamine inhibition, thyrotropin releasing hormone has a stimulatory
effect on release. Prolactin release stimulates milk production but also reduces
gonadal activity. It decreases GnRH pulsatility at the hypothalamic level and
to a lesser extent, blocks the action of LH on the ovary or testis.
Which of the following statements are not typically true in hypokalaemia?

A. It may occur as a result of mechanical bowel preparation

B. Chronic vomiting may increase renal potassium losses

C. It may be associated with aciduria

D. It may cause hyponatraemia

E. It often accompanies acidosis

Potassium depletion occurs either through the gastrointestinal tract or the kidney.
Chronic vomiting in itself is less prone to induce potassium loss than diarrhoea as
gastric secretions contain less potassium than those in the lower GI tract. If vomiting
produces a metabolic alkalosis then renal potassium wasting may occur as potassium
is excreted in preference to hydrogen ions. The converse may occur in potassium
depletion resulting in acid urine.

Hypokalaemia

Potassium and hydrogen can be thought of as competitors. Hyperkalaemia tends to be


associated with acidosis because as potassium levels rise fewer hydrogen ions can
enter the cells

Hypokalaemia with alkalosis

 Vomiting
 Diuretics
 Cushing's syndrome
 Conn's syndrome (primary hyperaldosteronism)

Hypokalaemia with acidosis

 Diarrhoea
 Renal tubular acidosis
 Acetazolamide
 Partially treated diabetic ketoacidosis
 Theme: Vitamin deficiency
A. Vitamin A
B. Vitamin B1
C. Vitamin B12
D. Vitamin B3
E. Vitamin C
F. Vitamin K
G. Vitamin D

Please select the vitamin deficiency most closely associated with the situation
described. Each option may be used once, more than once or not at all.

9. A 3 year old child presents with Rickets

Vitamin D

Vitamin D is needed to help mineralise bone. When this is deficient,


mineralisation is inadequate and deformities mayt result.

10. A 44 year old lady presents with jaundice. Following a minor ward based
surgical procedure she develops troublesome and persistent bleeding.

Vitamin K

Patients who are jaundiced usually have impaired absorption of vitamin K.


This can result in loss of the vitamin K dependent clotting factors and
troublesome bleeding.

11. A 69 year old man who has been living in sheleted accomodation for many
months, with inadequate nutrition notices that his night vision is becoming
impaired.

Vitamin A

Loss of vitamin A will result in impaire rhodopsin synthesis and poor night
vision.


 Vitamin deficiency

Vitamin Effect of deficiency


A Night blindness
Epithelial atrophy
Infections
B1 Beriberi
B2 Dematitis and photosensitivity
B3 Pellagra
B12 Pernicious anaemia
C Poor wound healing
Impaired collagen synthesis
D Rickets (Children)
Osteomalacia (Adults)
K Clotting disorders

Which of the following statements is true with regards to gastric secretions?

A. During the cephalic phase 40% of the total gastric secretion occurs

B. Histamine release stimulates acid secretion in the stomach

C. Secretin stimulates gastric acid secretion

D. Intrinsic factor combines with B1 in the stomach

E. G cells are only present in the gastric mucosa

Histamine is released from enterochromaffin cells in the stomach mucosa which stimulates
acid secretion.
Intrinsic factor combines with B12 to prevent acid digestion in the stomach.
G cells can be found in the duodenum and jejunum

Gastric secretions

A working knowledge of gastric secretions is important for surgery because peptic ulcers are
common, surgeons frequently prescribe anti secretory drugs and because there are still
patients around who will have undergone acid lowering procedures (Vagotomy) in the past.

Gastric acid

 Is produced by the parietal cells in the stomach


 pH of gastric acid is around 2 with acidity being maintained by the H+/K+ ATP ase
pump. As part of the process bicarbonate ions will be secreted into the surrounding
vessels.
 Sodium and chloride ions are actively secreted from the parietal cell into the
canaliculus. This sets up a negative potential across the membrane and as a result
sodium and potassium ions diffuse across into the canaliculus.
 Carbonic anhydrase forms carbonic acid which dissociates and the hydrogen ions
formed by dissociation leave the cell via the H+/K+ antiporter pump. At the same
time sodium ions are actively absorbed. This leaves hydrogen and chloride ions in
the canaliculus these mix and are secreted into the lumen of the oxyntic gland.

This is illustrated diagrammatically below:

Image sourced from Wikipedia

Phases of gastric acid secretion


There are 3 phases of gastric secretion:

1. Cephalic phase (smell / taste of food)


 30% acid produced
 Vagal cholinergic stimulation causing secretion of HCL and gastrin release from G
cells

2. Gastric phase (distension of stomach )

 60% acid produced


 Stomach distension/low H+/peptides causes Gastrin release

3. Intestinal phase (food in duodenum)

 10% acid produced


 High acidity/distension/hypertonic solutions in the duodenum inhibits gastric acid
secretion via enterogastrones (CCK, secretin) and neural reflexes.

Regulation of gastric acid production


Factors increasing production include:

 Vagal nerve stimulation


 Gastrin release
 Histamine release (indirectly following gastrin release) from enterchromaffin like
cells

Factors decreasing production include:

 Somatostatin (inhibits histamine release)


 Cholecystokinin
 Secretin

The diagram below illustrates some of the factors involved in regulating gastric acid
secretion and the relevant associated pharmacology
Image sourced from Wikipedia

Below is a brief summary of the major hormones involved in food digestion:

Source Stimulus Actions

Gastrin G cells in Distension of Increase HCL, pepsinogen and IF secretion,


antrum of stomach, extrinsic increases gastric motility, trophic effect on
the stomach nerves gastric mucosa
Inhibited by: low
antral pH,
somatostatin

CCK I cells in Partially digested Increases secretion of enzyme-rich fluid


upper small proteins and from pancreas, contraction of gallbladder
intestine triglycerides and relaxation of sphincter of Oddi,
decreases gastric emptying, trophic effect
on pancreatic acinar cells, induces satiety

Secretin S cells in Acidic chyme, fatty Increases secretion of bicarbonate-rich fluid


upper small acids from pancreas and hepatic duct cells,
intestine decreases gastric acid secretion, trophic
effect on pancreatic acinar cells

VIP Small Neural Stimulates secretion by pancreas and


intestine, intestines, inhibits acid and pepsinogen
pancreas secretion
Somatostatin D cells in the Fat, bile salts and Decreases acid and pepsin secretion,
pancreas and glucose in the decreases gastrin secretion, decreases
stomach intestinal lumen pancreatic enzyme secretion, decreases
insulin and glucagon secretion
inhibits trophic effects of gastrin, stimulates
gastric mucous production

A 55 year old man undergoes a laparotomy and repair of incisional hernia. Which of
the following hormones is least likely to be released in increased quantities following
the procedure?

A. Insulin

B. ACTH

C. Glucocorticoids

D. Aldosterone

E. Growth hormone

Insulin and thyroxine are often have reduced levels of secretion in the post operative
period. This, coupled with increased glucocorticoid release may cause difficulty in
management of diabetes in individuals with insulin resistance.

Stress response: Endocrine and metabolic changes

 Surgery precipitates hormonal and metabolic changes causing the stress


response
 Stress response is associated with: substrate mobilization, muscle protein loss,
sodium and water retention, suppression of anabolic hormone secretion,
activation of the sympathetic nervous system, immunological and
haematological changes.
 The hypothalamic-pituitary axis and the sympathetic nervous systems are
activated and there is a failure of the normal feedback mechanisms of control
of hormone secretion.

A summary of the hormonal changes associated with the stress response:

Increased Decreased No Change


Growth hormone Insulin Thyroid stimulating hormone
Cortisol Testosterone Luteinizing hormone
Renin Oestrogen Follicle stimulating hormone
Adrenocorticotrophic hormone (ACTH)
Aldosterone

Prolactin

Antidiuretic hormone
Glucagon

Sympathetic nervous system

 Stimulates catecholamine release


 Causes tachycardia and hypertension

Pituitary gland

 ACTH and growth hormone (GH) is stimulated by hypothalamic releasing


factors, corticotrophin releasing factor (CRF) and somatotrophin (or growth
hormone releasing factor)
 Perioperative increased prolactin secretion occurs by release of inhibitory
control
 Secretion of thyroid stimulating hormone (TSH), luteinizing hormone (LH)
and follicle stimulating hormone (FSH) does not change significantly
 ACTH stimulates cortisol production within a few minutes of the start of
surgery. More ACTH is produced than needed to produce a maximum
adrenocortical response.

Cortisol

 Significant increases within 4-6h of surgery (>1000 nmol litre-1)


 The usual negative feedback mechanism fails and concentrations of ACTH
and cortisol remain persistently increased
 The magnitude and duration of the increase correlate with the severity of stress
and the response is not abolished by the administration of corticosteroids.
 The metabolic effects of cortisol are enhanced:

Skeletal muscle protein breakdown to provide gluconeogenic precursors and amino


acids for protein synthesis in the liver
Stimulation of lipolysis
'Anti-insulin effect'
Mineralocorticoid effects
Anti-inflammatory effects

Growth hormone
 Increased secretion after surgery has a minor role
 Most important for preventing muscle protein breakdown and promote tissue
repair by insulin growth factors

Alpha Endorphin

 Increased

Antidiuretic hormone

 An important vasopressor and enhances haemostasis


 Renin is released causing the conversion of angiotensin I to angiotensin II,
which causes the secretion of aldosterone from the adrenal cortex. This
increases sodium reabsorption at the distal convoluted tubule

Insulin

 Release inhibited by stress


 Occurs via the inhibition of the alpha cells in the pancreas by the α2-
adrenergic inhibitory effects of catecholamines
 Insulin resistance by target cells occurs later
 The perioperative period is characterized by a state of functional insulin
deficiency

Thyroxine (T4) and tri-iodothyronine (T3)

 Circulating concentrations are inversely correlated with sympathetic activity


and after surgery there is a reduction in thyroid hormone production, which
normalises over a few days.

Metabolic effect of endocrine response

Carbohydrate metabolism

 Hyperglycaemia is a main feature of the metabolic response to surgery


 Due to increased increase in glucose production and a reduction in glucose
utilization
 Catecholamines and cortisol promote glycogenolysis and gluconeogenesis
 Initial failure of insulin secretion followed by insulin resistance affects the
normal responses
 The proportion of the hyperglycaemic response reflects the severity of surgery
 Hyperglycaemia impairs wound healing and increase infection rates

Protein metabolism
 Initially there is inhibition of protein anabolism, followed later, if the stress
response is severe, by enhanced catabolism
 The amount of protein degradation is influenced by the type of surgery and
also by the nutritional status of the patient
 Mainly skeletal muscle protein is affected
 The amino acids released form acute phase proteins (fibrinogen, C reactive
protein, complement proteins, a2-macroglobulin, amyloid A and
ceruloplasmin) and are used for gluconeogenesis
 Nutritional support has little effect on preventing catabolism

Lipid metabolism
Increased catecholamine, cortisol and glucagon secretion, and insulin deficiency,
promotes lipolysis and ketone body production.

Salt and water metabolism

 ADH causes water retention, concentrated urine, and potassium loss and may
continue for 3 to 5 days after surgery
 Renin causes sodium and water retention

Cytokines

 Glycoproteins
 Interleukins (IL) 1 to 17, interferons, and tumour necrosis factor
 Synthesized by activated macrophages, fibroblasts, endothelial and glial cells
in response to tissue injury from surgery or trauma
 IL-6 main cytokine associated with surgery. Peak 12 to 24 h after surgery and
increase by the degree of tissue damage Other effects of cytokines include
fever, granulocytosis, haemostasis, tissue damage limitation and promotion of
healing.

Modifying the response

 Opioids suppress hypothalamic and pituitary hormone secretion


 At high doses the hormonal response to pelvic and abdominal surgery is
abolished. However, such doses prolong recovery and increase the need for
postoperative ventilatory support
 Spinal anaesthesia can reduce the glucose, ACTH, cortisol, GH and
epinephrine changes, although cytokine responses are unaltered
 Cytokine release is reduced in less invasive surgery
 Nutrition prevents the adverse effects of the stress response. Enteral feeding
improves recovery
 Growth hormone and anabolic steroids may improve outcome
 Normothermia decreases the metabolic response
References
Deborah Burton, Grainne Nicholson, and George Hall
Endocrine and metabolic response to surgery .

Contin Educ Anaesth Crit Care Pain (2004) 4(5): 144-147


doi:10.1093/bjaceaccp/mkh040
A 55 year old man undergoes a laparotomy and repair of incisional hernia. Which of
the following hormones is least likely to be released in increased quantities following
the procedure?

A. Insulin

B. ACTH

C. Glucocorticoids

D. Aldosterone

E. Growth hormone

Insulin and thyroxine are often have reduced levels of secretion in the post operative
period. This, coupled with increased glucocorticoid release may cause difficulty in
management of diabetes in individuals with insulin resistance.

Stress response: Endocrine and metabolic changes

 Surgery precipitates hormonal and metabolic changes causing the stress


response
 Stress response is associated with: substrate mobilization, muscle protein loss,
sodium and water retention, suppression of anabolic hormone secretion,
activation of the sympathetic nervous system, immunological and
haematological changes.
 The hypothalamic-pituitary axis and the sympathetic nervous systems are
activated and there is a failure of the normal feedback mechanisms of control
of hormone secretion.

A summary of the hormonal changes associated with the stress response:

Increased Decreased No Change


Growth hormone Insulin Thyroid stimulating hormone
Cortisol Testosterone Luteinizing hormone
Renin Oestrogen Follicle stimulating hormone
Adrenocorticotrophic hormone (ACTH)
Aldosterone

Prolactin

Antidiuretic hormone
Glucagon

Sympathetic nervous system

 Stimulates catecholamine release


 Causes tachycardia and hypertension

Pituitary gland

 ACTH and growth hormone (GH) is stimulated by hypothalamic releasing


factors, corticotrophin releasing factor (CRF) and somatotrophin (or growth
hormone releasing factor)
 Perioperative increased prolactin secretion occurs by release of inhibitory
control
 Secretion of thyroid stimulating hormone (TSH), luteinizing hormone (LH)
and follicle stimulating hormone (FSH) does not change significantly
 ACTH stimulates cortisol production within a few minutes of the start of
surgery. More ACTH is produced than needed to produce a maximum
adrenocortical response.

Cortisol

 Significant increases within 4-6h of surgery (>1000 nmol litre-1)


 The usual negative feedback mechanism fails and concentrations of ACTH
and cortisol remain persistently increased
 The magnitude and duration of the increase correlate with the severity of stress
and the response is not abolished by the administration of corticosteroids.
 The metabolic effects of cortisol are enhanced:

Skeletal muscle protein breakdown to provide gluconeogenic precursors and amino


acids for protein synthesis in the liver
Stimulation of lipolysis
'Anti-insulin effect'
Mineralocorticoid effects
Anti-inflammatory effects

Growth hormone
 Increased secretion after surgery has a minor role
 Most important for preventing muscle protein breakdown and promote tissue
repair by insulin growth factors

Alpha Endorphin

 Increased

Antidiuretic hormone

 An important vasopressor and enhances haemostasis


 Renin is released causing the conversion of angiotensin I to angiotensin II,
which causes the secretion of aldosterone from the adrenal cortex. This
increases sodium reabsorption at the distal convoluted tubule

Insulin

 Release inhibited by stress


 Occurs via the inhibition of the alpha cells in the pancreas by the α2-
adrenergic inhibitory effects of catecholamines
 Insulin resistance by target cells occurs later
 The perioperative period is characterized by a state of functional insulin
deficiency

Thyroxine (T4) and tri-iodothyronine (T3)

 Circulating concentrations are inversely correlated with sympathetic activity


and after surgery there is a reduction in thyroid hormone production, which
normalises over a few days.

Metabolic effect of endocrine response

Carbohydrate metabolism

 Hyperglycaemia is a main feature of the metabolic response to surgery


 Due to increased increase in glucose production and a reduction in glucose
utilization
 Catecholamines and cortisol promote glycogenolysis and gluconeogenesis
 Initial failure of insulin secretion followed by insulin resistance affects the
normal responses
 The proportion of the hyperglycaemic response reflects the severity of surgery
 Hyperglycaemia impairs wound healing and increase infection rates

Protein metabolism
 Initially there is inhibition of protein anabolism, followed later, if the stress
response is severe, by enhanced catabolism
 The amount of protein degradation is influenced by the type of surgery and
also by the nutritional status of the patient
 Mainly skeletal muscle protein is affected
 The amino acids released form acute phase proteins (fibrinogen, C reactive
protein, complement proteins, a2-macroglobulin, amyloid A and
ceruloplasmin) and are used for gluconeogenesis
 Nutritional support has little effect on preventing catabolism

Lipid metabolism
Increased catecholamine, cortisol and glucagon secretion, and insulin deficiency,
promotes lipolysis and ketone body production.

Salt and water metabolism

 ADH causes water retention, concentrated urine, and potassium loss and may
continue for 3 to 5 days after surgery
 Renin causes sodium and water retention

Cytokines

 Glycoproteins
 Interleukins (IL) 1 to 17, interferons, and tumour necrosis factor
 Synthesized by activated macrophages, fibroblasts, endothelial and glial cells
in response to tissue injury from surgery or trauma
 IL-6 main cytokine associated with surgery. Peak 12 to 24 h after surgery and
increase by the degree of tissue damage Other effects of cytokines include
fever, granulocytosis, haemostasis, tissue damage limitation and promotion of
healing.

Modifying the response

 Opioids suppress hypothalamic and pituitary hormone secretion


 At high doses the hormonal response to pelvic and abdominal surgery is
abolished. However, such doses prolong recovery and increase the need for
postoperative ventilatory support
 Spinal anaesthesia can reduce the glucose, ACTH, cortisol, GH and
epinephrine changes, although cytokine responses are unaltered
 Cytokine release is reduced in less invasive surgery
 Nutrition prevents the adverse effects of the stress response. Enteral feeding
improves recovery
 Growth hormone and anabolic steroids may improve outcome
 Normothermia decreases the metabolic response
References
Deborah Burton, Grainne Nicholson, and George Hall
Endocrine and metabolic response to surgery .

Contin Educ Anaesth Crit Care Pain (2004) 4(5): 144-147


doi:10.1093/bjaceaccp/mkh040
Which of the following is not a major function of the spleen in adults?

A. Iron reutilisation

B. Storage of platelets

C. Storage of monocytes

D. Haematopoeisis in haematological disorders

E. Storage red blood cells

The reservoir function of the spleen is less marked in humans than other animals (e.g.
pigs) and in normal individuals it can sequester between 5 and 10% of the red cell
mass. The other stated processes are major splenic functions and this accounts for the
answer provided.

Spleen

 Embryology: derived from mesenchymal tissue


 Shape: orange segment
 Position: below 9th-12th ribs
 Weight: 75-150g

Relations

 Superiorly- diaphragm
 Anteriorly- gastric impression
 Posteriorly- kidney
 Inferiorly- colon
 Hilum: tail of pancreas and splenic vessels (splenic artery divides here,
branches pass to the white pulp transporting plasma)
 Forms apex of lesser sac (containing short gastric vessels)

Contents
- White pulp: immune function. Contains central trabecular artery. The germinal
centres are supplied by arterioles called penicilliary radicles.
- Red pulp: filters abnormal red blood cells

Function

 Filtration of abnormal blood cells and foreign bodies such as bacteria.


 Immunity: IgM. Production of properdin, and tuftsin which help target fungi
and bacteria for phagocytosis.
 Haematopoiesis: up to 5th month gestation or in haematological disorders.
 Pooling: storage of 40% platelets.
 Iron reutilisation
 Storage red blood cells-animals, not humans.
 Storage monocytes

Disorders of the spleen


Massive splenomegaly

 Myelofibrosis
 Chronic myeloid leukaemia
 Visceral leishmaniasis (kala-azar)
 Malaria
 Gaucher's syndrome

Other causes (as above plus)

 Portal hypertension e.g. secondary to cirrhosis


 Lymphoproliferative disease e.g. CLL, Hodgkin's
 Haemolytic anaemia
 Infection: hepatitis, glandular fever
 Infective endocarditis
 Sickle-cell*, thalassaemia
 Rheumatoid arthritis (Felty's syndrome)

*the majority of adults patients with sickle-cell will have an atrophied spleen due to
repeated infarction
Which one of the following is associated with increased lung compliance?

A. Kyphosis

B. Pulmonary oedema

C. Emphysema

D. Pulmonary fibrosis
E. Pneumonectomy

Respiratory physiology: lung compliance

Lung compliance is defined as change in lung volume per unit change in airway
pressure

Causes of increased compliance

 age
 emphysema - this is due to loss alveolar walls and associated elastic tissue

Causes of decreased compliance

 pulmonary oedema
 pulmonary fibrosis
 pneumonectomy
 kyphosis

Which of the following areas is predominantly concerned with thermoregulation?

A. Hypothalamus

B. Anterior pituitary

C. Cerebellum

D. Brain stem

E. Temporal lobe

Theme from 2012 Exam


The hypothalamus is primarily concerned with thermoregulation. It may relay to the
cerebral cortex to induce behavioural adaptation to facilitate the thermoregulatory
process.

Thermoregulation

 The hypothalamus is the main centre for thermoregulation. Peripheral and


central thermoreceptors relay to this region.
 Central chemoreceptors play the main role in maintenance of core
temperature.
 Hypothalamus may initiate involuntary motor responses to raise body
temperature (e.g.shivering). It will also stimulate the sympathetic nervous
system to produce peripheral vasoconstriction and release of adrenaline from
the adrenal medulla.
 Heat loss is governed by behavioural responses and by autonomic responses
including peripheral vasodilation.
 Heat loss can be maintained within the thermoneutral zone (25 to 30 degrees)
although the absolute value depends upon atmospheric humidity.
 Sepsis results in the release of cytokines that reset the thermoregulatory centre
resulting in fever.

Which of the following drugs does not interfere with the measurement of cortisol
levels?

A. Dexamethasone

B. Prednisolone

C. Hydrocortisone IV

D. Hydrocortisone PO

E. Hydrocortisone IM

Dexamethasone can be given as glucorticoid replacement during testing for addisons


or adrenal insufficiency as it does not interfere with cortisol levels. For example if
you have a patient with polymyalgia rheumatica and they are on longterm
prednisolone, you can replace the prednisolone with dexamethasone to undertake a
short synacthen test.

Cortisol

 Glucocorticoid
 Released by zona fasiculata of the adrenal gland
 90% protein bound; 10% active
 Circadian rhythm: High in the mornings
 Negative feedback via ACTH

Actions

 Glycogenolysis
 Glucaneogenesis
 Protein catabolism
 Lipolysis
 Stress response
 Anti-inflammatory
 Decrease protein in bones
 Increase gastric acid
 Increases neutrophils/platelets/red blood cells
 Inhibits fibroblastic activity
 An elderly lady who presented with weight loss and malabsorption was found
to have amyloid of the small bowel. On presentation she was found to have
osteomalacia and was hypocalcaemic. Over the past seven days she has
received total parenteral nutrition with adequate calcium replacement. Despite
this she remained hypocalcaemic. Deficiency of which of the following
electrolytes is most likely to account for this process?

A. Magnesium

B. Potassium

C. Sodium

D. Phosphate

E. None of the above



Patients with malabsorption may develop magnesium deficiency, although her
TPN feeds may have contained magnesium it may not have been sufficient to
correct her losses. Sodium, phosphate and potassium would not have this
effect on serum calcium.
 Combined deficiency of magnesium and calcium

Magnesium is required for both PTH secretion and its action on target tissues.
Hypomagnesaemia may both cause hypocalcaemia and render patients
unresponsive to treatment with calcium and vitamin D supplementation.

Magnesium is the fourth most abundant cation in the body. The body contains
1000mmol, with half contained in bone and the remainder in muscle, soft
tissues and extracellular fluid. There is no one specific hormonal control of
magnesium and various hormones including PTH and aldosterone affect the
renal handling of magnesium.

Magnesium and calcium interact at a cellular level also and as a result


decreased magnesium will tend to affect the permeability of cellular
membranes to calcium, resulting in hyperexcitability.
A 19 year old man is attacked outside a club and beaten with a baseball bat. He
sustains a blow to the right side of his head. He is brought to the emergency
department and a policy of observation is adopted. His glasgow coma score
deteriorates and he becomes comatose. Which of the following haemodynamic
parameters is most likely to be present?
A. Hypertension and bradycardia

B. Hypotension and tachycardia

C. Hypotension and bradycardia

D. Hypertension and tachycardia

E. Normotension and bradycardia

Hypertension and bradycardia are seen prior to coning. The brain autoregulates its
blood supply by controlling systemic blood pressure.

Head injury

 Patients who suffer head injuries should be managed according to ATLS


principles and extra cranial injuries should be managed alongside cranial
trauma. Inadequate cardiac output will compromise CNS perfusion
irrespective of the nature of the cranial injury.

Types of traumatic brain injury


Extradural Bleeding into the space between the dura mater and the skull. Often
haematoma results from acceleration-deceleration trauma or a blow to the side
of the head. The majority of extradural haematomas occur in the
temporal region where skull fractures cause a rupture of the middle
meningeal artery.

Features

 Raised intracranial pressure


 Some patients may exhibit a lucid interval

Subdural Bleeding into the outermost meningeal layer. Most commonly


haematoma occur around the frontal and parietal lobes. May be either acute or
chronic.

Risk factors include old age and alcoholism.

Slower onset of symptoms than a extradural haematoma.


Subarachnoid Usually occurs spontaneously in the context of a ruptured cerebral
haemorrhage aneurysm but may be seen in association with other injuries when a
patient has sustained a traumatic brain injury

Pathophysiology
 Primary brain injury may be focal (contusion/ haematoma) or diffuse (diffuse
axonal injury)
 Diffuse axonal injury occurs as a result of mechanical shearing following
deceleration, causing disruption and tearing of axons
 Intra-cranial haematomas can be extradural, subdural or intracerebral, while
contusions may occur adjacent to (coup) or contralateral (contre-coup) to the
side of impact
 Secondary brain injury occurs when cerebral oedema, ischaemia, infection,
tonsillar or tentorial herniation exacerbates the original injury. The normal
cerebral auto regulatory processes are disrupted following trauma rendering
the brain more susceptible to blood flow changes and hypoxia
 The Cushings reflex (hypertension and bradycardia) often occurs late and is
usually a pre terminal event

Management

 Where there is life threatening rising ICP such as in extra dural haematoma
and whilst theatre is prepared or transfer arranged use of IV mannitol/
frusemide may be required.
 Diffuse cerebral oedema may require decompressive craniotomy
 Exploratory Burr Holes have little management in modern practice except
where scanning may be unavailable and to thus facilitate creation of formal
craniotomy flap
 Depressed skull fractures that are open require formal surgical reduction and
debridement, closed injuries may be managed non operatively if there is
minimal displacement.
 ICP monitoring is appropriate in those who have GCS 3-8 and normal CT
scan.
 ICP monitoring is mandatory in those who have GCS 3-8 and Abnormal CT
scan.
 Hyponatraemia is most likely to be due to syndrome of inappropriate ADH
secretion.
 Minimum of cerebral perfusion pressure of 70mmHg in adults.
 Minimum cerebral perfusion pressure of between 40 and 70 mmHg in
children.

Interpretation of pupillary findings in head injuries


Pupil size Light response Interpretation
Unilaterally dilated Sluggish or fixed 3rd nerve compression secondary to
tentorial herniation
Bilaterally dilated Sluggish or fixed  Poor CNS perfusion
 Bilateral 3rd nerve palsy

Unilaterally dilated Cross reactive Optic nerve injury


or equal (Marcus - Gunn)
Bilaterally May be difficult to  Opiates
constricted assess  Pontine lesions
 Metabolic encephalopathy

Unilaterally Preserved Sympathetic pathway disruption


constricted
Which of the following drugs does not cause pseudohaematuria?

A. Rifampicin

B. Quinine

C. Noradrenaline

D. Levodopa

E. Phenytoin

Rifampicin, phenytoin, levodopa, methyldopa, and quinine all cause


pseudohaematuria.

Haematuria

Causes of haematuria

Trauma  Injury to renal tract


 Renal trauma commonly due to blunt injury (others
penetrating injuries)
 Ureter trauma rare: iatrogenic
 Bladder trauma: due to RTA or pelvic fractures

Infection  Remember TB

Malignancy  Renal cell carcinoma (remember paraneoplastic


syndromes): painful or painless
 Urothelial malignancies: 90% are transitional cell
carcinoma, can occur anywhere along the urinary tract.
Painless haematuria.
 Squamous cell carcinoma and adenocarcinoma: rare
bladder tumours
 Prostate cancer
 Penile cancers: SCC

Renal disease  Glomerulonephritis

Stones  Microscopic haematuria common


Structural  Benign prostatic hyperplasia (BPH) causes haematuria
abnormalities due to hypervascularity of the prostate gland
 Cystic renal lesions e.g. polycystic kidney disease
 Vascular malformations
 Renal vein thrombosis due to renal cell carcinoma

Coagulopathy  Causes bleeding of underlying lesions

Drugs  Cause tubular necrosis or interstitial nephritis:


aminoglycosides, chemotherapy
 Interstitial nephritis: penicillin, sulphonamides, and
NSAIDs
 Anticoagulants

Benign  Exercise

Gynaecological  Endometriosis: flank pain, dysuria, and haematuria that


is cyclical

Iatrogenic  Catheterisation
 Radiotherapy; cystitis, severe haemorrhage, bladder
necrosis

Pseudohaematuria

References
Http://bestpractice.bmj.com/best-practice/monograph/316/overview/aetiology.html
A 74-year-old woman with thyroid cancer is admitted due to shortness of breath.
What is the best investigation to assess for possible compression of the upper
airways?

A. Arterial blood gases

B. Forced vital capacity

C. Transfer factor

D. Peak expiratory flow rate

E. Flow volume loop

Flow volume loop is the investigation of choice for upper airway compression.

Flow volume loop

A normal flow volume loop is often described as a 'triangle on top of a semi circle'
Flow volume loops are the most suitable way of assessing compression of the upper
airway
Which of the following statements relating to cerebrospinal fluid is untrue?

A. The choroid plexus is only present in the lateral ventricles

B. Total CSF volume is 100-150ml

C. CSF pressure is usually 10-15mmHg

D. The cerebral aqueduct connects the third and fourth ventricles

E. The foramen of Luschka are paired and lie laterally in the fourth
ventricle

The choroid plexus lies in all ventricles.

Cerebrospinal fluid

The CSF fills the space between the arachnoid mater and pia mater (covering surface
of the brain). The total volume of CSF in the brain is approximately 150ml.
Approximately 500 ml is produced by the ependymal cells in the choroid plexus
(70%), or blood vessels (30%). It is reabsorbed via the arachnoid granulations which
project into the venous sinuses.

Circulation
1. Lateral ventricles (via foramen Munro)
2. 3rd ventricle
3. Cerebral aqueduct (aqueduct Sylvius)
4. 4th ventricle (via foramina of Magendie and Luschka)
5. Subarachnoid space
6. Reabsorbed into venous system via arachnoid granulations in superior sagittal sinus

Composition

 Glucose: 50-80mg/dl
 Protein: 15-40 mg/dl
 Red blood cells: Nil
 White blood cells: 0-3 cells/ mm3

Which substance can be used to achieve the most accurate measurement of the
glomerular filtration rate?

A. Glucose
B. Protein

C. Inulin

D. Creatine

E. Para-amino hippuric acid

Creatinine declines with age due to decline in renal function and muscle mass.
Glucose, protein (amino acids) and PAH are reabsorbed by the kidney.

Renal Physiology

Overview

 Each nephron is supplied with blood from an afferent arteriole that opens onto
the glomerular capillary bed.
 Blood then flows to an efferent arteriole, supplying the peritubular capillaries
and medullary vasa recta.
 The kidney receives up to 25% of resting cardiac output.

Control of blood flow

 The kidney is able to autoregulate its blood flow between systolic pressures of
80- 180mmHg so there is little variation in renal blood flow.
 This is achieved by myogenic control of arteriolar tone, both sympathetic
input and hormonal signals (e.g. renin) are responsible.

Glomerular structure and function

 Blood inside the glomerulus has considerable hydrostatic pressure.


 The basement membrane has pores that will allow free diffusion of smaller
solutes, larger negatively charged molecules such as albumin are unable to
cross.
 The glomerular filtration rate (GFR) is equal to the concentration of a solute in
the urine, times the volume of urine produced per minute, divided by the
plasma concentration (assuming that the solute is freely diffused e.g. inulin).
 In clinical practice creatinine is used because it is subjected to very little
proximal tubular secretion.
 Although subject to variability, the typical GFR is 125ml per minute.

 Glomerular filtration rate = Total volume of plasma per unit time leaving the
capillaries and entering the bowman's capsule
 Renal clearance = volume plasma from which a substance is removed per
minute by the kidneys

 Substances used to measure GFR have the following features:

1. Inert
2. Free filtration from the plasma at the glomerulus (not protein bound)
3. Not absorbed or secreted at the tubules
4. Plasma concentration constant during urine collection

Examples: inulin, creatinine

GFR = urine concentration (mmol/l) x urine volume (ml/min)


--------------------------------------------------------------------------
plasma concentration (mmol/l)

 The clearance of a substance is dependent not only on its diffusivity across the
basement membrane but also subsequent tubular secretion and / or
reabsorption.
 So glucose which is freely filtered across the basement membrane is usually
reabsorbed from tubules giving a clearance of zero.

Tubular function

 Reabsorption and secretion of substances occurs in the tubules.


 In the proximal tubule substrates such as glucose, amino acids and phosphate
are co-transported with sodium across the semi permeable membrane.
 Up to two thirds of filtered water is reabsorbed in the proximal tubules.
 This will lead to increase in urea concentration in the distal tubule allowing for
its increased diffusion.
 Substances to be secreted into the tubules are taken up from the peritubular
blood by tubular cells.
 Solutes such as paraaminohippuric acid are cleared with a single passage
through the kidneys and this is why it is used to measure renal plasma flow.
Ions such as calcium and phosphate will have a tubular reabsorption that is
influenced by plasma PTH levels.
 Potassium may be both secreted and re-absorbed and is co-exchanged with
sodium.

Loop of Henle

 Approximately 60 litres of water containing 9000mmol sodium enters the


descending limb of the loop of Henle in 24 hours.
 Loops from the juxtamedullary nephrons run deep into the medulla.
 The osmolarity of fluid changes and is greatest at the tip of the papilla.
 The thin ascending limb is impermeable to water, but highly permeable to
sodium and chloride ions.
 This loss means that at the beginning of the thick ascending limb the fluid is
hypo osmotic compared with adjacent interstitial fluid.
 In the thick ascending limb the reabsorption of sodium and chloride ions
occurs by both facilitated and passive diffusion pathways.
 The loops of Henle are co-located with vasa recta, these will have similar
solute compositions to the surrounding extracellular fluid so preventing the
diffusion and subsequent removal of this hypertonic fluid.
 The energy dependent reabsorption of sodium and chloride in the thick
ascending limb helps to maintain this osmotic gradient.

A 45 year old man sustains a closed head injury. He is initially alert, however, his
level of consciousness deteriorates on arrival at hospital. An intra cranial pressure
monitor is inserted. What is the normal intracranial pressure?

A. 35 - 45mm Hg

B. 45 - 55mm Hg

C. <15mm Hg

D. 25 - 35mm Hg

E. 25 - 30 mm Hg

The normal intracranial pressure is between 7 and 15 mm Hg. The brain can
accommodate increases up to 24 mm Hg, thereafter clinical features will become
evident.

Applied neurophysiology

 Pressure within the cranium is governed by the Monroe-Kelly doctrine. This


considers the skull as a closed box. Increases in mass can be accommodated
by loss of CSF. Once a critical point is reached (usually 100- 120ml of CSF
lost) there can be no further compensation and ICP rises sharply. The next step
is that pressure will begin to equate with MAP and neuronal death will occur.
Herniation will also accompany this process.
 The CNS can autoregulate its own blood supply. Vaso constriction and
dilatation of the cerebral blood vessels is the primary method by which this
occurs. Extremes of blood pressure can exceed this capacity resulting in risk
of stroke. Other metabolic factors such as hypercapnia will also cause
vasodilation, which is of importance in ventilating head injured patients.
 The brain can only metabolise glucose, when glucose levels fall,
consciousness will be impaired.

A 55-year-old man with a history of type 2 diabetes mellitus, bipolar disorder and
chronic obstructive pulmonary disease has bloods taken during a pre operative
assessment of an inguinal hernia repair:

Na+ 125 mmol/l


K+ 3.8 mmol/l
Bicarbonate 24 mmol/l
Urea 3.7 mmol/l
Creatinine 92 µmol/l

Due to his smoking history a chest x-ray is ordered which is reported as normal. The
Consultant asks you what is the most likely cause for the hyponatraemia?

A. Metformin

B. Lithium

C. Carbamazepine

D. Sodium valproate

E. Pioglitazone
SIADH - drug causes: carbamazepine,
sulfonylureas, SSRIs, tricyclics

Lithium can cause diabetes insipidus but this is generally associated with a high
sodium. Lithium only tends to cause raised antidiuretic hormone levels following a
severe overdosage.

syndrome of inappropriate antidiuretic hormone (SIADH): causes

Malignancy

 especially small cell lung cancer


 also: pancreas, prostate

Neurological

 stroke
 subarachnoid haemorrhage
 subdural haemorrhage
 meningitis/encephalitis/abscess

Infections

 tuberculosis
 pneumonia

Drugs

 sulfonylureas
 SSRIs, tricyclics
 carbamazepine
 vincristine
 cyclophosphamide

Other causes

 positive end-expiratory pressure (PEEP)


 porphyrias
 A 39 year old lady has recurrent attacks of biliary colic. What is the
approximate volume of bile to enter the duodenum per 24 hours?

A. 500 mL

B. 50 mL

C. 100 mL

D. 2000 mL

E. 150 mL

Between 500 mL and 1.5 L of bile enters the small bowel daily. Most bile salts
are recycled by the enterohepatic circulation. When the gallbladder contracts
the lumenal pressure is approximately 25cm water, which is why biliary colic
may be so painful.
 Bile

Bile is produced at a rate of between 500ml and 1500mL per day. Bile is
composed of bile salts, bicarbonate, cholesterol, steroids and water. There are
three main factors regulating bile flow; hepatic secretion, gall bladder
contraction and sphincter of oddi resistance. Bile salts are absorbed in the
terminal ileum (and recycled to the liver). Over 90% of all bile salts are
recycled in this way, such that the total pool of bile salts is recycled up to six
times a day.

Primary bile salts


Cholate and chenodeoxycholate.

Secondary bile salts


Formed by bacterial action on primary bile salts. These are deoxycholate and
lithocholate. Of these deoxycholate is reabsorbed, whilst lithocholate is
insoluble and excreted.

Pathophysiology of gallstones
Bile salts have a detergent action. They aggregate to form micelles and these
have a lipid centre in which fats may be transported. Excessive quantities of
cholesterol cannot be transported in this way and will tend to precipitate,
resulting in the formation of cholesterol rich gallstones.
 At which of the following sites is the most water absorbed?

A. Right colon

B. Left colon

C. Stomach

D. Jejunum

E. Duodenum

Water absorption in the gastrointestinal tract predominantly occurs in the
small bowel (jejunum and ileum). The colon is an important site of water
absorption, however, its overall contribution is relatively small. The
importance of the colonic component to water absorption may increase
following extensive small bowel resections.
 Water absorption

During a 24 hours period the average person will ingest up to 2000ml of liquid
orally. In addition a further 8000ml of fluid will enter the small bowel as
gastrointestinal secretions. Intestinal water absorption is a passive process and
is related to solute load. In the jejunum the active absorption of glucose and
amino acids will create a concentration gradient that water will flow across. In
the ileum most water is absorbed by a process of facilitated diffusion (with
sodium).
Approximately 150ml of water enters the colon daily, most is absorbed, the
colon can adapt to, and increase this amount following resection.
Which of the following is not a characteristic of the proximal convoluted tubule in the
kidney?
A. Up to 95% of filtered amino acids will be reabsorbed at this site

B. It is a risk of damage in a patient with compartment syndrome due to


a tibial fracture

C. It is responsible for absorbing more than 50% of filtered water

D. Its secretory function is most effective at low systolic blood pressures


(typically less than 100mmHg)

E. Glucose is reabsorbed by a process of facilitated diffusion

The proximal convoluted tubule may undergo necrosis in situations such as


compartment syndrome. It is responsible for reabsorbing up to two thirds of filtered
water. Low systolic blood pressures (below the renal autoregulatory range) are a risk
factor for acute tubular necrosis. Within the autoregulatory range the absolute value of
systolic BP has little effect.

Renal Physiology

Overview

 Each nephron is supplied with blood from an afferent arteriole that opens onto
the glomerular capillary bed.
 Blood then flows to an efferent arteriole, supplying the peritubular capillaries
and medullary vasa recta.
 The kidney receives up to 25% of resting cardiac output.

Control of blood flow

 The kidney is able to autoregulate its blood flow between systolic pressures of
80- 180mmHg so there is little variation in renal blood flow.
 This is achieved by myogenic control of arteriolar tone, both sympathetic
input and hormonal signals (e.g. renin) are responsible.

Glomerular structure and function

 Blood inside the glomerulus has considerable hydrostatic pressure.


 The basement membrane has pores that will allow free diffusion of smaller
solutes, larger negatively charged molecules such as albumin are unable to
cross.
 The glomerular filtration rate (GFR) is equal to the concentration of a solute in
the urine, times the volume of urine produced per minute, divided by the
plasma concentration (assuming that the solute is freely diffused e.g. inulin).
 In clinical practice creatinine is used because it is subjected to very little
proximal tubular secretion.
 Although subject to variability, the typical GFR is 125ml per minute.

 Glomerular filtration rate = Total volume of plasma per unit time leaving the
capillaries and entering the bowman's capsule

 Renal clearance = volume plasma from which a substance is removed per


minute by the kidneys

 Substances used to measure GFR have the following features:

1. Inert
2. Free filtration from the plasma at the glomerulus (not protein bound)
3. Not absorbed or secreted at the tubules
4. Plasma concentration constant during urine collection

Examples: inulin, creatinine

GFR = urine concentration (mmol/l) x urine volume (ml/min)


--------------------------------------------------------------------------
plasma concentration (mmol/l)

 The clearance of a substance is dependent not only on its diffusivity across the
basement membrane but also subsequent tubular secretion and / or
reabsorption.
 So glucose which is freely filtered across the basement membrane is usually
reabsorbed from tubules giving a clearance of zero.

Tubular function

 Reabsorption and secretion of substances occurs in the tubules.


 In the proximal tubule substrates such as glucose, amino acids and phosphate
are co-transported with sodium across the semi permeable membrane.
 Up to two thirds of filtered water is reabsorbed in the proximal tubules.
 This will lead to increase in urea concentration in the distal tubule allowing for
its increased diffusion.
 Substances to be secreted into the tubules are taken up from the peritubular
blood by tubular cells.
 Solutes such as paraaminohippuric acid are cleared with a single passage
through the kidneys and this is why it is used to measure renal plasma flow.
Ions such as calcium and phosphate will have a tubular reabsorption that is
influenced by plasma PTH levels.
 Potassium may be both secreted and re-absorbed and is co-exchanged with
sodium.

Loop of Henle

 Approximately 60 litres of water containing 9000mmol sodium enters the


descending limb of the loop of Henle in 24 hours.
 Loops from the juxtamedullary nephrons run deep into the medulla.
 The osmolarity of fluid changes and is greatest at the tip of the papilla.
 The thin ascending limb is impermeable to water, but highly permeable to
sodium and chloride ions.
 This loss means that at the beginning of the thick ascending limb the fluid is
hypo osmotic compared with adjacent interstitial fluid.
 In the thick ascending limb the reabsorption of sodium and chloride ions
occurs by both facilitated and passive diffusion pathways.
 The loops of Henle are co-located with vasa recta, these will have similar
solute compositions to the surrounding extracellular fluid so preventing the
diffusion and subsequent removal of this hypertonic fluid.
 The energy dependent reabsorption of sodium and chloride in the thick
ascending limb helps to maintain this osmotic gradient.

Which of the following does not cause a normal anion gap acidosis?

A. Pancreatic fistula

B. Acetazolamide

C. Uraemia

D. Ureteric diversion

E. Renal tubular acidosis

Normal Gap Acidosis: HARDUP

H-
Hyperalimentation/hyperventilation
A - Acetazolamide
R - Renal tubular acidosis
D - Diarrhoea
U - Ureteral diversion
P - Pancreatic fistula/parenteral
saline

Uraemia will typically cause a high anion gap acidosis. It is one of the unmeasured anions.

Disorders of acid - base balance

Disorders of acid- base balance are often covered in the MRCS part A, both in the SBA and
EMQ sections.

The acid-base normogram below shows how the various disorders may be categorised

Image sourced from Wikipedia

Metabolic acidosis

 This is the most common surgical acid - base disorder.


 Reduction in plasma bicarbonate levels.
 Two mechanisms:

1. Gain of strong acid (e.g. diabetic ketoacidosis)


2. Loss of base (e.g. from bowel in diarrhoea)
- Classified according to the anion gap, this can be calculated by:
(Na+ + K+) - (Cl- + HCO3-).
- If a question supplies the chloride level then this is often a clue that the anion gap should
be calculated. The normal range = 10-18 mmol/L

Normal anion gap ( = hyperchloraemic metabolic acidosis)

 Gastrointestinal bicarbonate loss: diarrhoea, ureterosigmoidostomy, fistula


 Renal tubular acidosis
 Drugs: e.g. acetazolamide
 Ammonium chloride injection
 Addison's disease

Raised anion gap

 Lactate: shock, hypoxia


 Ketones: diabetic ketoacidosis, alcohol
 Urate: renal failure
 Acid poisoning: salicylates, methanol

Metabolic acidosis secondary to high lactate levels may be subdivided into two types:

 Lactic acidosis type A: (Perfusion disorders e.g.shock, hypoxia, burns)


 Lactic acidosis type B: (Metabolic e.g. metformin toxicity)

Metabolic alkalosis

 Usually caused by a rise in plasma bicarbonate levels.


 Rise of bicarbonate above 24 mmol/L will typically result in renal excretion of excess
bicarbonate.
 Caused by a loss of hydrogen ions or a gain of bicarbonate. It is due mainly to
problems of the kidney or gastrointestinal tract

Causes

 Vomiting / aspiration (e.g. Peptic ulcer leading to pyloric stenosis, nasogastric


suction)
 Diuretics
 Liquorice, carbenoxolone
 Hypokalaemia
 Primary hyperaldosteronism
 Cushing's syndrome
 Bartter's syndrome
 Congenital adrenal hyperplasia
Mechanism of metabolic alkalosis

 Activation of renin-angiotensin II-aldosterone (RAA) system is a key factor


 Aldosterone causes reabsorption of Na+ in exchange for H+ in the distal convoluted
tubule
 ECF depletion (vomiting, diuretics) --> Na+ and Cl- loss --> activation of RAA system --
> raised aldosterone levels
 In hypokalaemia, K+ shift from cells --> ECF, alkalosis is caused by shift of H+ into cells
to maintain neutrality

Respiratory acidosis

 Rise in carbon dioxide levels usually as a result of alveolar hypoventilation


 Renal compensation may occur leading to Compensated respiratory acidosis

Causes

 COPD
 Decompensation in other respiratory conditions e.g. Life-threatening asthma /
pulmonary oedema
 Sedative drugs: benzodiazepines, opiate overdose

Respiratory alkalosis

 Hyperventilation resulting in excess loss of carbon dioxide


 This will result in increasing pH

Causes

 Psychogenic: anxiety leading to hyperventilation


 Hypoxia causing a subsequent hyperventilation: pulmonary embolism, high altitude
 Early salicylate poisoning*
 CNS stimulation: stroke, subarachnoid haemorrhage, encephalitis
 Pregnancy

*Salicylate overdose leads to a mixed respiratory alkalosis and metabolic acidosis. Early
stimulation of the respiratory centre leads to a respiratory alkalosis whilst later the direct
acid effects of salicylates (combined with acute renal failure) may lead to an acidosis
Which one of the following would cause a rise in the carbon monoxide transfer factor
(TLCO)?
A. Emphysema

B. Pulmonary embolism

C. Pulmonary haemorrhage

D. Pneumonia

E. Pulmonary fibrosis
Transfer factor

 raised: asthma, haemorrhage, left-


to-right shunts, polycythaemia
 low: everything else

Where alveolar haemorrhage occurs the TLCO tends to increase due to the enhanced
uptake of carbon monoxide by intra-alveolar haemoglobin.

Transfer factor

The transfer factor describes the rate at which a gas will diffuse from alveoli into
blood. Carbon monoxide is used to test the rate of diffusion. Results may be given as
the total gas transfer (TLCO) or that corrected for lung volume (transfer coefficient,
KCO)

Causes of a raised TLCO Causes of a lower TLCO

 asthma  pulmonary fibrosis


 pulmonary haemorrhage (Wegener's,  pneumonia
Goodpasture's)  pulmonary emboli
 left-to-right cardiac shunts  pulmonary oedema
 polycythaemia  emphysema
 hyperkinetic states  anaemia
 male gender, exercise  low cardiac output

KCO also tends to increase with age. Some conditions may cause an increased KCO
with a normal or reduced TLCO

 pneumonectomy/lobectomy
 scoliosis/kyphosis
 neuromuscular weakness
 ankylosis of costovertebral joints e.g. ankylosing spondylitis
 Release of somatostatin from the pancreas will result in which of the
following?
A. Decrease in pancreatic exocrine secretions

B. Contraction of the gallbladder

C. Increase in the rate of gastric emptying

D. Increased synthesis of growth hormone

E. Increased insulin release



Octreotide reduces exocrine pancreatic secretions so is used to treat high
output pancreatic fistulae (though parenteral feeding is most effective). Other
uses include variceal bleeding and treatment of acromegaly.
Inhibits growth hormone and insulin release (when released from pancreas).
Somatostatin is also released by the hypothalamus causing a negative feedback
response on growth hormone.
 Somatostatin

Somatostatin is produced in the D cells of the pancreatic islets. It is also
produced in the gut (enterochromaffin cells) and is found in brain tissue.
Those substances that stimulate insulin release will also induce somatostatin
secretion. It is an inhibitor of growth hormone, it also delays gastric emptying
and reduces gastrin secretion.
It reduces pancreatic exocrine secretions and may be used therapeutically to
treat pancreatic fistulae.

Somatostatinomas are rare pancreatic endocrine tumours and will result in the
clinical manifestations of diabetes mellitus, gallstones and steatorrhoea.
A 34 year old lady develops septic shock and features of the systemic inflammatory
response syndrome as a complication of cholangitis. Which of the following is not a
typical feature of this condition?

A. Body temperature less than 36oC or greater than 38oC

B. Respiratory rate >20

C. Lactate <4 mmol/L

D. High levels of tumour necrosis factor α

E. WCC >12,000 mm3

Septic shock will typically result in end organ hypoperfusion and as a result lactate
levels will often be high. In the surviving sepsis campaign it is suggested that elevated
lactate levels are an independent indicator for vasopressor support in patient with
sepsis. The WCC may be paradoxically low in severe sepsis, although it is most often
elevated.

Shock

 Shock occurs when there is insufficient tissue perfusion.


 The pathophysiology of shock is an important surgical topic and may be
divided into the following aetiological groups:
 Septic
 Haemorrhagic
 Neurogenic
 Cardiogenic
 Anaphylactic

Septic shock
Septic shock is a major problem and those patients with severe sepsis have a mortality
rate in excess of 40%. In those who are admitted to intensive care mortality ranges
from 6% with no organ failure to 65% in those with 4 organ failure.

Sepsis is defined as an infection that triggers a particular Systemic Inflammatory


Response Syndrome (SIRS). This is characterised by body temperature outside 36
o
C - 38 o C, HR >90 beats/min, respiratory rate >20/min, WBC count >12,000/mm3 or
< 4,000/mm3.

Patients with infections and two or more elements of SIRS meet the diagnostic criteria
for sepsis. Those with organ failure have severe sepsis and those with refractory
hypotension -septic shock.

During the septic process there is marked activation of the immune system with
extensive cytokine release. This may be coupled with or triggered by systemic
circulation of bacterial toxins. These all cause endothelial cell damage and neutrophil
adhesion. The overall hallmarks are thus those of excessive inflammation,
coagulation and fibrinolytic suppression.

The surviving sepsis campaign highlights the following key areas for attention:

 Prompt administration of antibiotics to cover all likely pathogens coupled with


a rigorous search for the source of infection.
 Haemodynamic stabilisation. Many patients are hypovolaemic and require
aggressive fluid administration. Aim for CVP 8-12 cm H2O, MAP >65mmHg.
 Modulation of the septic response. This includes manoeuvres to counteract the
changes and includes measures such as tight glycaemic control, use of
activated protein C and sometimes intravenous steroids.

In surgical patients, the main groups with septic shock include those with anastomotic
leaks, abscesses and extensive superficial infections such as necrotising fasciitis.
When performing surgery the aim should be to undertake the minimum necessary to
restore physiology. These patients do not fare well with prolonged surgery. Definitive
surgery can be more safely undertaken when physiology is restored and clotting in
particular has been normalised.

Haemorrhagic shock
The average adult blood volume comprises 7% of body weight. Thus in the 70 Kg
adult this will equate to 5 litres. This changes in children (8-9% body weight) and is
slightly lower in the elderly.

The table below outlines the 4 major classes of haemorrhagic shock and their
associated physiological sequelae:

Parameter Class I Class II Class III Class IV


Blood loss ml <750ml 750-1500ml 1500-2000ml >2000ml
Blood loss % <15% 15-30% 30-40% >40%
Pulse rate <100 >100 >120 >140ml
Blood pressure Normal Decreased Decreased Decreased
Respiratory rate 14-20 20-30 30-40 >35
Urine output >30ml 20-30ml 5-15ml <5ml
Symptoms Normal Anxious Confused Lethargic

Decreasing blood pressure during haemorrhagic shock causes organ hypoperfusion


and relative myocardial ishaemia. The cardiac index gives a numerical value for tissue
oxygen delivery and is given by the equation: Cardiac index= 13.4 - [Hb] - SaO2 +
0.03 PaO2. Where Hb is haemoglobin concentration in blood and SaO2 the saturation
and PaO2 the partial pressure of oxygen. Detailed knowledge of this equation is
required for the MRCS Viva but not for part A, although you should understand the
principle.

In patients suffering from trauma the most likely cause of shock is haemorrhage.
However, the following may also be the cause or occur concomitantly:

 Tension pneumothorax
 Spinal cord injury
 Myocardial contusion
 Cardiac tamponade

When assessing trauma patients it is worth remembering that in order to generate a


palpable femoral pulse an arterial pressure of >65mmHg is required.

Once bleeding is controlled and circulating volume normalised the levels of


transfusion should be to maintain a Hb of 7-8 in those with no risk factors for tissue
hypoxia and Hb 10 for those who have such risk factors.

Neurogenic shock
This occurs most often following a spinal cord transection, usually at a high level.
There is resultant interruption of the autonomic nervous system. The result is either
decreased sympathetic tone or increased parasympathetic tone, the effect of
which is a decrease in peripheral vascular resistance mediated by marked
vasodilation.

This results in decreased preload and thus decreased cardiac output (Starlings law).
There is decreased peripheral tissue perfusion and shock is thus produced. In contrast
with many other types of shock peripheral vasoconstrictors are used to return vascular
tone to normal.

Cardiogenic shock
In medical patients the main cause is ischaemic heart disease. In the traumatic
setting direct myocardial trauma or contusion is more likely. Evidence of ECG
changes and overlying sternal fractures or contusions should raise the suspicion of
injury. Treatment is largely supportive and transthoracic echocardiography should be
used to determine evidence of pericardial fluid or direct myocardial injury. The
measurement of troponin levels in trauma patients may be undertaken but they are
less useful in delineating the extent of myocardial trauma than following MI.

When cardiac injury is of a blunt nature and is associated with cardiogenic shock the
right side of the heart is the most likely site of injury with chamber and or valve
rupture. These patients require surgery to repair these defects and will require
cardiopulmonary bypass to achieve this. Some may require intra aortic balloon pump
as a bridge to surgery.

Anaphylactic shock
Anaphylaxis may be defined as a severe, life-threatening, generalised or systemic
hypersensitivity reaction.

Anaphylaxis is one of the few times when you would not have time to look up the
dose of a medication. The Resuscitation Council guidelines on anaphylaxis have
recently been updated. Adrenaline is by far the most important drug in anaphylaxis
and should be given as soon as possible. The recommended doses for adrenaline,
hydrocortisone and chlorphenamine are as follows:

Adrenaline Hydrocortisone Chlorphenamine


< 6 months 150 mcg (0.15ml 1 in 25 mg 250 mcg/kg
1,000)
6 months - 6 years 150 mcg (0.15ml 1 in 50 mg 2.5 mg
1,000)
6-12 years 300 mcg (0.3ml 1 in 100 mg 5 mg
1,000)
Adult and child 12 500 mcg (0.5ml 1 in 200 mg 10 mg
years 1,000)

Adrenaline can be repeated every 5 minutes if necessary. The best site for IM
injection is the anterolateral aspect of the middle third of the thigh.
Common identified causes of anaphylaxis

 food (e.g. Nuts) - the most common cause in children


 drugs
 venom (e.g. Wasp sting)

Which of the following stimulates gastric acid secretion?

A. Cholecystokinin

B. Gastric inhibitory peptide

C. Secretin

D. Histamine

E. Somatostatin

Gastrin: From G cells: stimulates gastric acid production

Pepsin: Digestion of protein, secretion occurs simultaneously with gastrin

Secretin: From mucosal cells in the duodenum and jejunum: inhibits gastric acid,
stimulates bile and pancreatic juice production

Gastric inhibitory peptide: (produced in response to fatty acids) inhibits gastrin


release and acid secretion from parietal cells

Cholecystokinin: From mucosal cells in the duodenum and jejunum (produced in


response to fatty acids) inhibits acid secretion from parietal cells, causes gallbladder
contraction and relaxation of sphincter of Oddi

Somatostatin: From D cells

Hormonal control of gastric acid secretion

Source Stimulus Actions


Gastrin G cells in Distension of Increase HCL, pepsinogen and IF
antrum of stomach, secretion, increases gastric motility,
the stomach extrinsic nerves trophic effect on gastric mucosa
Inhibited by: low
antral pH,
somatostatin
CCK I cells in Partially digested Increases secretion of enzyme-rich
upper small proteins and fluid from pancreas, contraction of
intestine triglycerides gallbladder and relaxation of sphincter
of Oddi, decreases gastric emptying,
trophic effect on pancreatic acinar
cells, induces satiety
Secretin S cells in Acidic chyme, Increases secretion of bicarbonate-rich
upper small fatty acids fluid from pancreas and hepatic duct
intestine cells, decreases gastric acid secretion,
trophic effect on pancreatic acinar cells
VIP Small Neural Stimulates secretion by pancreas and
intestine, intestines, inhibits acid and pepsinogen
pancreas secretion
Somatostatin D cells in Fat, bile salts and Decreases acid and pepsin secretion,
the pancreas glucose in the decreases gastrin secretion, decreases
and stomach intestinal lumen pancreatic enzyme secretion, decreases
insulin and glucagon secretion
inhibits trophic effects of gastrin,
stimulates gastric mucous production

Which of the following statements relating to gastric acid secretions are untrue?

A. In parietal cells carbonic anhydrase generates hydrogen ions which are


then actively secreted

B. The cephalic phase is abolished following truncal vagotomy

C. The intestinal phase accounts for 60% of gastric acid produced

D. Histamine acts in a paracrine manner on H2 receptors

E. H2 receptor antagonists will not completely abolish gastric acid production

The intestinal phase of gastric acid secretion accounts for only 10% of gastric acid produced.

Gastric secretions

A working knowledge of gastric secretions is important for surgery because peptic ulcers are
common, surgeons frequently prescribe anti secretory drugs and because there are still
patients around who will have undergone acid lowering procedures (Vagotomy) in the past.

Gastric acid
 Is produced by the parietal cells in the stomach
 pH of gastric acid is around 2 with acidity being maintained by the H+/K+ ATP ase
pump. As part of the process bicarbonate ions will be secreted into the surrounding
vessels.
 Sodium and chloride ions are actively secreted from the parietal cell into the
canaliculus. This sets up a negative potential across the membrane and as a result
sodium and potassium ions diffuse across into the canaliculus.
 Carbonic anhydrase forms carbonic acid which dissociates and the hydrogen ions
formed by dissociation leave the cell via the H+/K+ antiporter pump. At the same
time sodium ions are actively absorbed. This leaves hydrogen and chloride ions in
the canaliculus these mix and are secreted into the lumen of the oxyntic gland.

This is illustrated diagrammatically below:

Image sourced from Wikipedia

Phases of gastric acid secretion


There are 3 phases of gastric secretion:

1. Cephalic phase (smell / taste of food)


 30% acid produced
 Vagal cholinergic stimulation causing secretion of HCL and gastrin release from G
cells

2. Gastric phase (distension of stomach )

 60% acid produced


 Stomach distension/low H+/peptides causes Gastrin release

3. Intestinal phase (food in duodenum)

 10% acid produced


 High acidity/distension/hypertonic solutions in the duodenum inhibits gastric acid
secretion via enterogastrones (CCK, secretin) and neural reflexes.

Regulation of gastric acid production


Factors increasing production include:

 Vagal nerve stimulation


 Gastrin release
 Histamine release (indirectly following gastrin release) from enterchromaffin like
cells

Factors decreasing production include:

 Somatostatin (inhibits histamine release)


 Cholecystokinin
 Secretin

The diagram below illustrates some of the factors involved in regulating gastric acid
secretion and the relevant associated pharmacology
Image sourced from Wikipedia

Below is a brief summary of the major hormones involved in food digestion:

Source Stimulus Actions

Gastrin G cells in Distension of Increase HCL, pepsinogen and IF secretion,


antrum of stomach, extrinsic increases gastric motility, trophic effect on
the stomach nerves gastric mucosa
Inhibited by: low
antral pH,
somatostatin

CCK I cells in Partially digested Increases secretion of enzyme-rich fluid


upper small proteins and from pancreas, contraction of gallbladder
intestine triglycerides and relaxation of sphincter of Oddi,
decreases gastric emptying, trophic effect
on pancreatic acinar cells, induces satiety

Secretin S cells in Acidic chyme, fatty Increases secretion of bicarbonate-rich fluid


upper small acids from pancreas and hepatic duct cells,
intestine decreases gastric acid secretion, trophic
effect on pancreatic acinar cells

VIP Small Neural Stimulates secretion by pancreas and


intestine, intestines, inhibits acid and pepsinogen
pancreas secretion
Somatostatin D cells in the Fat, bile salts and Decreases acid and pepsin secretion,
pancreas and glucose in the decreases gastrin secretion, decreases
stomach intestinal lumen pancreatic enzyme secretion, decreases
insulin and glucagon secretion
inhibits trophic effects of gastrin, stimulates
gastric mucous production

A 22 year old man is undergoing a daycase excision of a sebaceous cyst. He is needle


phobic and as the surgeon approaches with the needle the patient begins to
hyperventilate. He soon develops circumoral parasthesia and muscular twitching.
Which of the following is the most likely explanation for this event?

A. Temporal lobe epilepsy

B. Reduction in ionised calcium levels

C. Increase in ionised calcium levels

D. Fall in serum PTH levels

E. Rise in serum PTH levels

50% of plasma calcium is ionised. Hyperventilation will induce a state of alkalosis


which will lower ionised plasma calcium levels.

Calcium homeostasis

Calcium ions are linked to a wide range of physiological processes. The largest store
of bodily calcium is contained within the skeleton. Calcium levels are primarily
controlled by parathyroid hormone, vitamin D and calcitonin.

Hormonal regulation of calcium


Hormone Actions
Parathyroid hormone (PTH)  Increase calcium levels and decrease
phosphate levels
 Increases bone resorption
 Immediate action on osteoblasts to
increase ca2+ in extracellular fluid
 Osteoblasts produce a protein signaling
molecule that activate osteoclasts which
cause bone resorption
 Increases renal tubular reabsorption of
calcium
 Increases synthesis of 1,25(OH)2D
(active form of vitamin D) in the kidney
which increases bowel absorption of Ca2+
 Decreases renal phosphate reabsorption

1,25-dihydroxycholecalciferol  Increases plasma calcium and plasma


(the active form of vitamin D) phosphate
 Increases renal tubular reabsorption and
gut absorption of calcium
 Increases osteoclastic activity
 Increases renal phosphate reabsorption

Calcitonin  Secreted by C cells of thyroid


 Inhibits intestinal calcium absorption
 Inhibits osteoclast activity
 Inhibits renal tubular absorption of
calcium

Both growth hormone and thyroxine also play a small role in calcium metabolism.
Which of the following inhibits gastric acid secretion?

A. Histamine

B. Nausea

C. Calcium

D. Parasympathetic vagal stimulation

E. Gastrin

Nausea inhibits gastric secretion via higher cerebral activity and sympathetic innervation.

Gastric secretions

A working knowledge of gastric secretions is important for surgery because peptic ulcers are
common, surgeons frequently prescribe anti secretory drugs and because there are still
patients around who will have undergone acid lowering procedures (Vagotomy) in the past.

Gastric acid

 Is produced by the parietal cells in the stomach


 pH of gastric acid is around 2 with acidity being maintained by the H+/K+ ATP ase
pump. As part of the process bicarbonate ions will be secreted into the surrounding
vessels.
 Sodium and chloride ions are actively secreted from the parietal cell into the
canaliculus. This sets up a negative potential across the membrane and as a result
sodium and potassium ions diffuse across into the canaliculus.
 Carbonic anhydrase forms carbonic acid which dissociates and the hydrogen ions
formed by dissociation leave the cell via the H+/K+ antiporter pump. At the same
time sodium ions are actively absorbed. This leaves hydrogen and chloride ions in
the canaliculus these mix and are secreted into the lumen of the oxyntic gland.

This is illustrated diagrammatically below:

Image sourced from Wikipedia

Phases of gastric acid secretion


There are 3 phases of gastric secretion:

1. Cephalic phase (smell / taste of food)


 30% acid produced
 Vagal cholinergic stimulation causing secretion of HCL and gastrin release from G
cells

2. Gastric phase (distension of stomach )

 60% acid produced


 Stomach distension/low H+/peptides causes Gastrin release

3. Intestinal phase (food in duodenum)

 10% acid produced


 High acidity/distension/hypertonic solutions in the duodenum inhibits gastric acid
secretion via enterogastrones (CCK, secretin) and neural reflexes.

Regulation of gastric acid production


Factors increasing production include:

 Vagal nerve stimulation


 Gastrin release
 Histamine release (indirectly following gastrin release) from enterchromaffin like
cells

Factors decreasing production include:

 Somatostatin (inhibits histamine release)


 Cholecystokinin
 Secretin

The diagram below illustrates some of the factors involved in regulating gastric acid
secretion and the relevant associated pharmacology
Image sourced from Wikipedia

Below is a brief summary of the major hormones involved in food digestion:

Source Stimulus Actions

Gastrin G cells in Distension of Increase HCL, pepsinogen and IF secretion,


antrum of stomach, extrinsic increases gastric motility, trophic effect on
the stomach nerves gastric mucosa
Inhibited by: low
antral pH,
somatostatin

CCK I cells in Partially digested Increases secretion of enzyme-rich fluid


upper small proteins and from pancreas, contraction of gallbladder
intestine triglycerides and relaxation of sphincter of Oddi,
decreases gastric emptying, trophic effect
on pancreatic acinar cells, induces satiety

Secretin S cells in Acidic chyme, fatty Increases secretion of bicarbonate-rich fluid


upper small acids from pancreas and hepatic duct cells,
intestine decreases gastric acid secretion, trophic
effect on pancreatic acinar cells

VIP Small Neural Stimulates secretion by pancreas and


intestine, intestines, inhibits acid and pepsinogen
pancreas secretion
Somatostatin D cells in the Fat, bile salts and Decreases acid and pepsin secretion,
pancreas and glucose in the decreases gastrin secretion, decreases
stomach intestinal lumen pancreatic enzyme secretion, decreases
insulin and glucagon secretion
inhibits trophic effects of gastrin, stimulates
gastric mucous production

73 year old lady is diagnosed with hyperaldosteronism. From which of the following
structures is aldosterone released?

A. Zona fasciculata of the adrenal gland

B. Juxtaglomerular apparatus of the kidney

C. Zona reticularis of the adrenal gland

D. Adrenal medulla

E. Zona glomerulosa of the adrenal cortex

Aldosterone serves to conserve sodium and water. It is produced in the zona


glomerulosa of the adrenal cortex.

Aldosterone

Adosterone is secreted by the zona glomerulosa of the adrenal cortex. It is a


mineralocorticoid hormone. Secretion is regulated by the renin- angiotensin system,
and by plasma levels of sodium and potassium. Aldosterone conserves sodium by
stimulating the reabsorption of sodium in the distal nephron in exchange for
potassium. Lack of aldosterone release will result in hyperkalaemia and
hyponatraemia.
A 43 year old lady is admitted with cholestasis secondary to a stone impacted at the
level of the ampulla of vater. Which of the following tests is most likely to be
predictive of bleeding diathesis at the time of ERCP in this particular case?

A. Bleeding time

B. Prothrombin time

C. APTT

D. Platelet count
E. Factor I levels
PT: Vitamin K dependent
factors 2, 7, 9, 10
APTT: Factors 8, 9, 11, 12

Jaundice will impair the production of vitamin K dependent clotting factors. This is
most accurately tested by measuring the prothrombin time. APTT can be affected by
vitamin K deficiency (due to factor 9 deficiency), however this occurs to a lesser
extent and is normally associated with severe liver disease. The bleeding time is a
measure of platelet function.

Abnormal coagulation

Cause Factors affected


Heparin Prevents activation factors 2,9,10,11
Warfarin Affects synthesis of factors 2,7,9,10
DIC Factors 1,2,5,8,11
Liver disease Factors 1,2,5,7,9,10

Interpretation blood clotting test results


Disorder APTT PT Bleeding time
Haemophilia Increased Normal Normal
von Willebrand's disease Increased Normal Increased
Vitamin K deficiency Increased Increased Normal

Which of the following mechanisms best accounts for the release of adrenaline?

A. Release from the adrenal medulla in response to increased


angiotensin 1 levels

B. Release from the zona fasiculata from the adrenal gland in response
to increased sympathetic discharge

C. Release from the adrenal medulla in response to increased


noradrenaline levels

D. Release from the adrenal medulla in response to sympathetic


stimulation from the splanchnic nerves

E. None of the above

The adrenal gland releases adrenaline in response to increased sympathetic discharge


from preganglionic sympathetic fibres of the splanchnic nerves. These cause the
chromafin cells of the medulla to release adrenaline (which is preformed) by
exocytosis.
Adrenaline

 Fight or Flight response

- Catecholamine (phenylalanine and tyrosine)


- Neurotransmitter and hormone
- Released by the adrenal glands
- Effects on α 1 and 2, β 1 and 2 receptors
- Main effect on alpha 1 receptors in skeletal muscle-causing vasodilation
- Increase cardiac output and total peripheral resistance
- This leads to vasoconstriction in the skin and kidneys causing a narrow pulse
pressure

Actions
α adrenergic receptors:

 Inhibits insulin secretion by the pancreas


 Stimulates glycogenolysis in the liver and muscle
 Stimulates glycolysis in muscle

β adrenergic receptors:

 Stimulates glucagon secretion in the pancreas


 Stimulates ACTH
 Stimulates lipolysis by adipose tissue

The acute phase response to injury does not include:

A. Pyrexia

B. Decreased albumin

C. Hepatic sequestration of cations

D. Increased transferrin

E. Increased serum amyloid A

The acute phase response includes:

 Acute phase proteins


 Reduction of transport proteins (albumin, transferrin)
 Hepatic sequestration cations
 Pyrexia
 Neutrophil leucocytosis
 Increased muscle proteolysis
 Changes in vascular permeability

Stress response: Endocrine and metabolic changes

 Surgery precipitates hormonal and metabolic changes causing the stress


response
 Stress response is associated with: substrate mobilization, muscle protein loss,
sodium and water retention, suppression of anabolic hormone secretion,
activation of the sympathetic nervous system, immunological and
haematological changes.
 The hypothalamic-pituitary axis and the sympathetic nervous systems are
activated and there is a failure of the normal feedback mechanisms of control
of hormone secretion.

A summary of the hormonal changes associated with the stress response:

Increased Decreased No Change


Growth hormone Insulin Thyroid stimulating hormone
Cortisol Testosterone Luteinizing hormone
Renin Oestrogen Follicle stimulating hormone
Adrenocorticotrophic hormone (ACTH)
Aldosterone

Prolactin

Antidiuretic hormone
Glucagon

Sympathetic nervous system

 Stimulates catecholamine release


 Causes tachycardia and hypertension

Pituitary gland

 ACTH and growth hormone (GH) is stimulated by hypothalamic releasing


factors, corticotrophin releasing factor (CRF) and somatotrophin (or growth
hormone releasing factor)
 Perioperative increased prolactin secretion occurs by release of inhibitory
control
 Secretion of thyroid stimulating hormone (TSH), luteinizing hormone (LH)
and follicle stimulating hormone (FSH) does not change significantly
 ACTH stimulates cortisol production within a few minutes of the start of
surgery. More ACTH is produced than needed to produce a maximum
adrenocortical response.

Cortisol

 Significant increases within 4-6h of surgery (>1000 nmol litre-1)


 The usual negative feedback mechanism fails and concentrations of ACTH
and cortisol remain persistently increased
 The magnitude and duration of the increase correlate with the severity of stress
and the response is not abolished by the administration of corticosteroids.
 The metabolic effects of cortisol are enhanced:

Skeletal muscle protein breakdown to provide gluconeogenic precursors and amino


acids for protein synthesis in the liver
Stimulation of lipolysis
'Anti-insulin effect'
Mineralocorticoid effects
Anti-inflammatory effects

Growth hormone

 Increased secretion after surgery has a minor role


 Most important for preventing muscle protein breakdown and promote tissue
repair by insulin growth factors

Alpha Endorphin

 Increased

Antidiuretic hormone

 An important vasopressor and enhances haemostasis


 Renin is released causing the conversion of angiotensin I to angiotensin II,
which causes the secretion of aldosterone from the adrenal cortex. This
increases sodium reabsorption at the distal convoluted tubule

Insulin

 Release inhibited by stress


 Occurs via the inhibition of the alpha cells in the pancreas by the α2-
adrenergic inhibitory effects of catecholamines
 Insulin resistance by target cells occurs later
 The perioperative period is characterized by a state of functional insulin
deficiency

Thyroxine (T4) and tri-iodothyronine (T3)

 Circulating concentrations are inversely correlated with sympathetic activity


and after surgery there is a reduction in thyroid hormone production, which
normalises over a few days.

Metabolic effect of endocrine response

Carbohydrate metabolism

 Hyperglycaemia is a main feature of the metabolic response to surgery


 Due to increased increase in glucose production and a reduction in glucose
utilization
 Catecholamines and cortisol promote glycogenolysis and gluconeogenesis
 Initial failure of insulin secretion followed by insulin resistance affects the
normal responses
 The proportion of the hyperglycaemic response reflects the severity of surgery
 Hyperglycaemia impairs wound healing and increase infection rates

Protein metabolism

 Initially there is inhibition of protein anabolism, followed later, if the stress


response is severe, by enhanced catabolism
 The amount of protein degradation is influenced by the type of surgery and
also by the nutritional status of the patient
 Mainly skeletal muscle protein is affected
 The amino acids released form acute phase proteins (fibrinogen, C reactive
protein, complement proteins, a2-macroglobulin, amyloid A and
ceruloplasmin) and are used for gluconeogenesis
 Nutritional support has little effect on preventing catabolism

Lipid metabolism
Increased catecholamine, cortisol and glucagon secretion, and insulin deficiency,
promotes lipolysis and ketone body production.

Salt and water metabolism

 ADH causes water retention, concentrated urine, and potassium loss and may
continue for 3 to 5 days after surgery
 Renin causes sodium and water retention
Cytokines

 Glycoproteins
 Interleukins (IL) 1 to 17, interferons, and tumour necrosis factor
 Synthesized by activated macrophages, fibroblasts, endothelial and glial cells
in response to tissue injury from surgery or trauma
 IL-6 main cytokine associated with surgery. Peak 12 to 24 h after surgery and
increase by the degree of tissue damage Other effects of cytokines include
fever, granulocytosis, haemostasis, tissue damage limitation and promotion of
healing.

Modifying the response

 Opioids suppress hypothalamic and pituitary hormone secretion


 At high doses the hormonal response to pelvic and abdominal surgery is
abolished. However, such doses prolong recovery and increase the need for
postoperative ventilatory support
 Spinal anaesthesia can reduce the glucose, ACTH, cortisol, GH and
epinephrine changes, although cytokine responses are unaltered
 Cytokine release is reduced in less invasive surgery
 Nutrition prevents the adverse effects of the stress response. Enteral feeding
improves recovery
 Growth hormone and anabolic steroids may improve outcome
 Normothermia decreases the metabolic response

References
Deborah Burton, Grainne Nicholson, and George Hall
Endocrine and metabolic response to surgery .

Contin Educ Anaesth Crit Care Pain (2004) 4(5): 144-147


doi:10.1093/bjaceaccp/mkh040
Which of the following statements relating to blood transfusions in surgical patients is
false?

A. Packed red cells typically have a haematocrit of between 55 and 75%

B. Clotting factor activity in whole blood decreases in samples stored


for longer than 7 days

C. After 3 weeks of storage blood has a pH of 6.9

D. Gamma irradiated blood products are not required routinely


E. Patients should be transfused to achieve a target haemoglobin of 10
g/dl and a haematocrit of 30%

Patients can generally be managed without transfusion as long as the Hb is 7 or


greater. The exact level depends upon patient factors such as co-morbidities. Old
blood functions less effectively and should not be used during massive transfusions.

Blood products

Whole blood fractions

Fraction Key points


Packed red cells Used for transfusion in chronic anaemia and cases where infusion of
large volumes of fluid may result in cardiovascular compromise.
Product obtained by centrifugation of whole blood.
Platelet rich Usually administered to patients who are thrombocytopaenic and
plasma are bleeding or require surgery. It is obtained by low speed
centrifugation.
Platelet Prepared by high speed centrifugation and administered to patients
concentrate with thrombocytopaenia.
Fresh frozen  Prepared from single units of blood.
plasma  Contains clotting factors, albumin and immunoglobulin.
 Unit is usually 200 to 250ml.
 Usually used in correcting clotting deficiencies in patients
with hepatic synthetic failure who are due to undergo
surgery.
 Usual dose is 12-15ml/Kg-1.
 It should not be used as first line therapy for hypovolaemia.

Cryoprecipitate  Formed from supernatant of FFP.


 Rich source of Factor VIII and fibrinogen.
 Allows large concentration of factor VIII to be administered
in small volume.

SAG-Mannitol Removal of all plasma from a blood unit and substitution with:
Blood
 Sodium chloride
 Adenine
 Anhydrous glucose
 Mannitol

Up to 4 units of SAG M Blood may be administered. Thereafter


whole blood is preferred. After 8 units, clotting factors and platelets
should be considered.

Cell saver devices


These collect patients own blood lost during surgery and then reinfuse it. There are
two main types:
 Those which wash the blood cells prior to reinfusion. These are more
expensive to purchase and more complicated to operate. However, they reduce
the risk of reinfusing contaminated blood back into the patient.
 Those which do not wash the blood prior to reinfusion.

Their main advantage is that they avoid the use of infusion of blood from donors into
patients and this may reduce risk of blood borne infection. It may be acceptable to
Jehovah's witnesses. It is contraindicated in malignant disease for risk of facilitating
disease dissemination.

Blood products used in warfarin reversal


In some surgical patients the use of warfarin can pose specific problems and may
require the use of specialised blood products

Immediate or urgent surgery in patients taking warfarin(1) (2):

1. Stop warfarin

2. Vitamin K (reversal within 4-24 hours)


-IV takes 4-6h to work (at least 5mg)
-Oral can take 24 hours to be clinically effective

3. Fresh frozen plasma


Used less commonly now as 1st line warfarin reversal
-30ml/kg-1
-Need to give at least 1L fluid in 70kg person (therefore not appropriate in fluid
overload)
-Need blood group
-Only use if human prothrombin complex is not available

4. Human Prothrombin Complex (reversal within 1 hour)


-Bereplex 50 u/kg
-Rapid action but factor 6 short half life, therefore give with vitamin K

References
1. Dentali, F., C. Marchesi, et al. (2011). "Safety of prothrombin complex
concentrates for rapid anticoagulation reversal of vitamin K antagonists. A meta-
analysis." Thromb Haemost 106(3): 429-438.

2. http://www.transfusionguidelines.org/docs/pdfs/bbt-03warfarin-reversal-flowchart-
2006.pdf
Which of the following statements relating the fluid physiology of a physiologically
normal 70 Kg adult male is false?

A. He will have more water per unit of body weight than


a female of similar weight

B. Plasma will comprise 25% of his body weight


C. Interstitial fluid will account for up to 14% of body
weight

D. Approximately 65% of total body water is intracellular

E. 60% of his body weight is composed of water


The 60-40-20 rule:
60% total body weight is
water
40% of total body weight is
intracellular fluids
20% of body weight is
extracellular fluids

Plasma typically accounts for 4-6% of body weight in healthy individuals.


Males typically have more water per unit weight than females, as females have a
higher fat content.

Fluid compartment physiology

Body fluid compartments comprise intracellular and extracellular compartments. The


latter includes interstitial fluid, plasma and transcellular fluid.
Typical figures are based on the 70 Kg male.

Body fluid volumes


Compartment Volume in litres Percentage of total volume
Intracellular 28 L 60-65%
Extracellular 14 L 35-40%
Plasma 3L 8%
Interstitial 10 L 24%
Transcellular 1 L 3%
Figures are approximate
A 17 year old lady with long standing anorexia nervosa is due to undergo excision of
a lipoma. Which of the following nutritional deficiencies is most likely to be
implicated in poor collagen formation as the wound heals?

A. Deficiency of copper

B. Deficiency of iron

C. Deficiency of ascorbic acid

D. Deficiency of phosphate
E. None of the above

Theme from January 2011

Vitamin C is involved in the cross linkage of collagen and impaired wound healing is
well described in cases of vitamin C deficiency.

Collagen

One of the major connective tissue proteins

 Composed of 3 polypeptide strands that are woven into a helix


 Numerous hydrogen bonds exist within molecule to provide additional
strength
 Many sub types but commonest sub type is I (90% of bodily collagen)
 Vitamin c is important in establishing cross links

Collagen Diseases

 Osteogenesis imperfecta
 Ehlers Danlos

Osteogenesis imperfecta:
-8 Subtypes
-Defect of type I collagen
-In type I the collagen is normal quality but insufficient quantity
-Type II- poor quantity and quality
-Type III- Collagen poorly formed, normal quantity
-Type IV- Sufficient quantity but poor quality
Patients have bones which fracture easily, loose joint and multiple other defects
depending upon which sub type they suffer from

Ehlers Danlos:
-Multiple sub types
-Abnormality of types 1 and 3 collagen
-Patients have features of hypermobility.
-Individuals are prone to joint dislocations and pelvic organ prolapse. In addition to
many other diseases related to connective tissue defects
A 45 year old man is undergoing a small bowel resection. The anaesthetist decides to
administer an intravenous fluid which is electrolyte rich. Which of the following most
closely matches this requirement?

A. Dextrose / Saline

B. Pentastarch
C. Gelofusine

D. Hartmans

E. 5% Dextrose with added potassium 20mmol/ L

Theme from April 2012 Exam


Hartmans solution is the most electrolyte rich. However, both pentastarch and
gelofusine have more macromolecules.

Intra operative fluid management

Composition of commonly used intravenous fluids mmol-1

Na K Cl Bicarbonate Lactate
Plasma 137-147 4-5.5 95-105 22-25 -
0.9% Saline 153 - 153 - -
Dextrose / saline 30.6 - 30.6 - -
Hartmans 130 4 110 - 28

Recommendations for intra operative fluid management

 Intra operative fluids are recommended to optimise cardiac stroke volume.


 Patients undergoing non elective orthopaedic or abdominal surgery should
receive IV fluids for the 1st 8h post operatively. This may be supplemented by
a low dose dopexamine infusion in selected cases.

References
British Consensus Guidelines on Intravenous Fluid Therapy for Adult Surgical
Patients
GIFTASUP (2009) Revised May 2011.
A 45 year old man is undergoing a small bowel resection. The anaesthetist decides to
administer an intravenous fluid which is electrolyte rich. Which of the following most
closely matches this requirement?

A. Dextrose / Saline

B. Pentastarch

C. Gelofusine

D. Hartmans
E. 5% Dextrose with added potassium 20mmol/ L

Theme from April 2012 Exam


Hartmans solution is the most electrolyte rich. However, both pentastarch and
gelofusine have more macromolecules.

Intra operative fluid management

Composition of commonly used intravenous fluids mmol-1

Na K Cl Bicarbonate Lactate
Plasma 137-147 4-5.5 95-105 22-25 -
0.9% Saline 153 - 153 - -
Dextrose / saline 30.6 - 30.6 - -
Hartmans 130 4 110 - 28

Recommendations for intra operative fluid management

 Intra operative fluids are recommended to optimise cardiac stroke volume.


 Patients undergoing non elective orthopaedic or abdominal surgery should
receive IV fluids for the 1st 8h post operatively. This may be supplemented by
a low dose dopexamine infusion in selected cases.

References
British Consensus Guidelines on Intravenous Fluid Therapy for Adult Surgical
Patients
GIFTASUP (2009) Revised May 2011.
A 16 year old girl develops pyelonephritis and is admitted in a state of septic shock.
Which of the following is not typically seen in this condition?

A. Increased cardiac output

B. Increased systemic vascular resistance

C. Oliguria may occur

D. Systemic cytokine release

E. Tachycardia
Cardiogenic Shock:
e.g. MI, valve abnormality

increased SVR (vasoconstriction in response to low BP)


increased HR (sympathetic response)
decreased cardiac output
decreased blood pressure

Hypovolaemic shock:
blood volume depletion
e.g. haemorrhage, vomiting, diarrhoea, dehydration, third-space losses during major
operations

increased SVR
increased HR
decreased cardiac output
decreased blood pressure

Septic shock:
occurs when the peripheral vascular dilatation causes a fall in SVR
similar response may occur in anaphylactic shock, neurogenic shock

reduced SVR
increased HR
normal/increased cardiac output
decreased blood pressure

Theme from January 2012 Exam


The SVR is reduced in sepsis and for this reason a vasoconstricting inotrope such as
noradrenaline may be used if hypotension and oliguria remain a concern despite
administration of adequate amounts of intravenous fluids.

Shock

 Shock occurs when there is insufficient tissue perfusion.


 The pathophysiology of shock is an important surgical topic and may be
divided into the following aetiological groups:
 Septic
 Haemorrhagic
 Neurogenic
 Cardiogenic
 Anaphylactic

Septic shock
Septic shock is a major problem and those patients with severe sepsis have a mortality
rate in excess of 40%. In those who are admitted to intensive care mortality ranges
from 6% with no organ failure to 65% in those with 4 organ failure.

Sepsis is defined as an infection that triggers a particular Systemic Inflammatory


Response Syndrome (SIRS). This is characterised by body temperature outside 36
o
C - 38 o C, HR >90 beats/min, respiratory rate >20/min, WBC count >12,000/mm3 or
< 4,000/mm3.
Patients with infections and two or more elements of SIRS meet the diagnostic criteria
for sepsis. Those with organ failure have severe sepsis and those with refractory
hypotension -septic shock.

During the septic process there is marked activation of the immune system with
extensive cytokine release. This may be coupled with or triggered by systemic
circulation of bacterial toxins. These all cause endothelial cell damage and neutrophil
adhesion. The overall hallmarks are thus those of excessive inflammation,
coagulation and fibrinolytic suppression.

The surviving sepsis campaign highlights the following key areas for attention:

 Prompt administration of antibiotics to cover all likely pathogens coupled with


a rigorous search for the source of infection.
 Haemodynamic stabilisation. Many patients are hypovolaemic and require
aggressive fluid administration. Aim for CVP 8-12 cm H2O, MAP >65mmHg.
 Modulation of the septic response. This includes manoeuvres to counteract the
changes and includes measures such as tight glycaemic control, use of
activated protein C and sometimes intravenous steroids.

In surgical patients, the main groups with septic shock include those with anastomotic
leaks, abscesses and extensive superficial infections such as necrotising fasciitis.
When performing surgery the aim should be to undertake the minimum necessary to
restore physiology. These patients do not fare well with prolonged surgery. Definitive
surgery can be more safely undertaken when physiology is restored and clotting in
particular has been normalised.

Haemorrhagic shock
The average adult blood volume comprises 7% of body weight. Thus in the 70 Kg
adult this will equate to 5 litres. This changes in children (8-9% body weight) and is
slightly lower in the elderly.

The table below outlines the 4 major classes of haemorrhagic shock and their
associated physiological sequelae:

Parameter Class I Class II Class III Class IV


Blood loss ml <750ml 750-1500ml 1500-2000ml >2000ml
Blood loss % <15% 15-30% 30-40% >40%
Pulse rate <100 >100 >120 >140ml
Blood pressure Normal Decreased Decreased Decreased
Respiratory rate 14-20 20-30 30-40 >35
Urine output >30ml 20-30ml 5-15ml <5ml
Symptoms Normal Anxious Confused Lethargic

Decreasing blood pressure during haemorrhagic shock causes organ hypoperfusion


and relative myocardial ishaemia. The cardiac index gives a numerical value for tissue
oxygen delivery and is given by the equation: Cardiac index= 13.4 - [Hb] - SaO2 +
0.03 PaO2. Where Hb is haemoglobin concentration in blood and SaO2 the saturation
and PaO2 the partial pressure of oxygen. Detailed knowledge of this equation is
required for the MRCS Viva but not for part A, although you should understand the
principle.

In patients suffering from trauma the most likely cause of shock is haemorrhage.
However, the following may also be the cause or occur concomitantly:

 Tension pneumothorax
 Spinal cord injury
 Myocardial contusion
 Cardiac tamponade

When assessing trauma patients it is worth remembering that in order to generate a


palpable femoral pulse an arterial pressure of >65mmHg is required.

Once bleeding is controlled and circulating volume normalised the levels of


transfusion should be to maintain a Hb of 7-8 in those with no risk factors for tissue
hypoxia and Hb 10 for those who have such risk factors.

Neurogenic shock
This occurs most often following a spinal cord transection, usually at a high level.
There is resultant interruption of the autonomic nervous system. The result is either
decreased sympathetic tone or increased parasympathetic tone, the effect of
which is a decrease in peripheral vascular resistance mediated by marked
vasodilation.

This results in decreased preload and thus decreased cardiac output (Starlings law).
There is decreased peripheral tissue perfusion and shock is thus produced. In contrast
with many other types of shock peripheral vasoconstrictors are used to return vascular
tone to normal.

Cardiogenic shock
In medical patients the main cause is ischaemic heart disease. In the traumatic
setting direct myocardial trauma or contusion is more likely. Evidence of ECG
changes and overlying sternal fractures or contusions should raise the suspicion of
injury. Treatment is largely supportive and transthoracic echocardiography should be
used to determine evidence of pericardial fluid or direct myocardial injury. The
measurement of troponin levels in trauma patients may be undertaken but they are
less useful in delineating the extent of myocardial trauma than following MI.

When cardiac injury is of a blunt nature and is associated with cardiogenic shock the
right side of the heart is the most likely site of injury with chamber and or valve
rupture. These patients require surgery to repair these defects and will require
cardiopulmonary bypass to achieve this. Some may require intra aortic balloon pump
as a bridge to surgery.
Anaphylactic shock
Anaphylaxis may be defined as a severe, life-threatening, generalised or systemic
hypersensitivity reaction.

Anaphylaxis is one of the few times when you would not have time to look up the
dose of a medication. The Resuscitation Council guidelines on anaphylaxis have
recently been updated. Adrenaline is by far the most important drug in anaphylaxis
and should be given as soon as possible. The recommended doses for adrenaline,
hydrocortisone and chlorphenamine are as follows:

Adrenaline Hydrocortisone Chlorphenamine


< 6 months 150 mcg (0.15ml 1 in 25 mg 250 mcg/kg
1,000)
6 months - 6 years 150 mcg (0.15ml 1 in 50 mg 2.5 mg
1,000)
6-12 years 300 mcg (0.3ml 1 in 100 mg 5 mg
1,000)
Adult and child 12 500 mcg (0.5ml 1 in 200 mg 10 mg
years 1,000)

Adrenaline can be repeated every 5 minutes if necessary. The best site for IM
injection is the anterolateral aspect of the middle third of the thigh.

Common identified causes of anaphylaxis

 food (e.g. Nuts) - the most common cause in children


 drugs
 venom (e.g. Wasp sting)

A man is admitted after a period of prolonged self, induced starvation. Naso gastric
feeding is planned. Which of the following is least likely to occur?

A. Hypokalaemia

B. Increased risk of cardiac arrhythmias

C. His haemoglobin will have decreased affinity for oxygen

D. Hypophosphataemia

E. None of the above

The process of starvation may lower DPG levels, in practice this is unlikely to occur
early as it is generated during glycolysis. Altered metabolism in starvation may be
more acidotic and this would also tend to impair oxygen carriage.
Nutrition - Refeeding syndrome

Refeeding syndrome describes the metabolic abnormalities which occur on feeding a


person a starved state. The metabolic consequences include:

 Hypophosphataemia
 Hypokalaemia
 Hypomagnesaemia
 Abnormal fluid balance

These abnormalities can lead to organ failure.

Re-feeding problems
If patient not eaten for > 5 days, aim to re-feed at < 50% energy and protein levels

High risk for re-feeding problems


If one or more of the following:

 BMI < 16 kg/m2


 Unintentional weight loss >15% over 3-6 months
 Little nutritional intake > 10 days
 Hypokalaemia, Hypophosphataemia or hypomagnesaemia prior to feeding
(unless high)

If two or more of the following:

 BMI < 18.5 kg/m2


 Unintentional weight loss > 10% over 3-6 months
 Little nutritional intake > 5 days
 PMH alcohol abuse or drug therapy including insulin, chemotherapy,
diuretics, antacids

Prescription

 Start at up to 10 kcal/kg/day increasing to full needs over 4-7 days


 Start immediately before and during feeding: oral thiamine 200-300mg/day,
vitamin B co strong 1 tds and supplements
 Give K+ (2-4 mmol/kg/day), phosphate (0.3-0.6 mmol/kg/day), magnesium
(0.2-0.4 mmol/kg/day)

Where does spironolactone act in the kidney?

A. Glomerulus

B. Proximal convoluted tubule


C. Descending limb of the loop of Henle

D. Ascending limb of the loop of Henle

E. Distal convoluted tubule

Potassium sparing diuretics

Potassium-sparing diuretics may be divided into the epithelial sodium channel


blockers (amiloride and triamterene) and aldosterone antagonists (spironolactone and
eplerenone).

{Amiloride} is a weak diuretic which blocks the epithelial sodium channel in the
distal convoluted tubule.

Usually given with thiazides or loop diuretics as an alternative to potassium


supplementation.

{Spironolactone} is an aldosterone antagonist which acts act in the distal convoluted


tubule.

Indications

 ascites: patients with cirrhosis develop a secondary hyperaldosteronism.


Relatively large doses such as 100 or 200mg are often used
 heart failure
 nephrotic syndrome
 Conn's syndrome
 Which receptor does noradrenaline mainly bind to?

A. α 1 receptors

B. α 2 receptors

C. β 1 receptors

D. β 2 receptors

E. G receptors

Theme from 2009 Exam
Noradrenaline is the precursor of adrenaline. It is a powerful α 1 stimulant
(although it will increase myocardial contractility). Infusions will produce
vasoconstriction and an increase in total peripheral resistance. It is the inotrope
of choice in septic shock.
 Inotropes and cardiovascular receptors

Inotrope Cardiovascular receptor action
Adrenaline α-1, α-2, β-1, β-2
Noradrenaline α-1,( α-2), (β-1), (β-2)
Dobutamine β-1, (β 2)
Dopamine (α-1), (α-2), (β-1), D-1,D-2
 Minor receptor effects in brackets

Effects of receptor binding


α-1, α-2 vasoconstriction
β-1 increased cardiac contractility and HR
β-2 vasodilatation
D-1 renal and spleen vasodilatation
D-2 inhibits release of noradrenaline
A 47 year old lady is diagnosed as suffering from a phaeochromocytoma. From which
of the following amino acids are catecholamines primarily derived?

A. Aspartime

B. Glutamine

C. Arginine

D. Tyrosine

E. Alanine

Catecholamine hormones are derived from tyrosine, it is modified by a DOPA


decarboxylase enzyme to become dopamine and thereafter via two further enzymic
modifications to noradrenaline and finally adrenaline.

Adrenal physiology

Adrenal medulla
The chromaffin cells of the adrenal medulla secrete the catecholamines noradrenaline
and adrenaline. The medulla is innervated by the splanchnic nerves; the preganglionic
sympathetic fibres secrete acetylcholine causing the chromaffin cells to secrete their
contents by exocytosis.
Phaeochromocytomas are derived from these cells and will secrete both adrenaline
and nor adrenaline.
Adrenal cortex
Three histologically distinct zones are recognised:

Zone Location Hormone Secreted


Zona glomerulosa Outer zone Aldosterone
Zona fasiculata Middle zone Glucocorticoids
Zona reticularis Inner zone Androgens

The glucocorticoids and aldosterone are mostly bound to plasma proteins in the
circulation. Glucocorticoids are inactivated and excreted by the liver.
Where are the arterial baroreceptors located?

A. Carotid sinus and aortic arch

B. Carotid sinus only

C. Superior vena cava

D. External carotid artery

E. None of the above

They lie in the carotid sinus and aortic arch.

Cardiac physiology

 The heart has four chambers ejecting blood into both low pressure and high
pressure systems.
 The pumps generate pressures of between 0-25mmHg on the right side and 0-120
mmHg on the left.
 At rest diastole comprises 2/3 of the cardiac cycle.
 The product of the frequency of heart rate and stroke volume combine to give the
cardiac output which is typically 5-6L per minute.

Detailed descriptions of the various waveforms are often not a feature of MRCS A (although
they are on the syllabus). However, they are a very popular topic for surgical physiology
vivas in the oral examination.

Electrical properties
 Intrinsic myogenic rhythm within cardiac myocytes means that even the denervated
heart is capable of contraction.
 In the normal situation the cardiac impulse is generated in the sino atrial node in the
right atrium and conveyed to the ventricles via the atrioventricular node.
 The sino atrial node is also capable of spontaneous discharge and in the absence of
background vagal tone will typically discharge around 100x per minute. Hence the
higher resting heart rate found in cardiac transplant cases. In the SA and AV nodes
the resting membrane potential is lower than in surrounding cardiac cells and will
slowly depolarise from -70mV to around -50mV at which point an action potential is
generated.
 Differences in the depolarisation slopes between SA and AV nodes help to explain
why the SA node will depolarise first. The cells have a refractory period during which
they cannot be re-stimulated and this period allows for adequate ventricular filling.
In pathological tachycardic states this time period is overridden and inadequate
ventricular filling may then occur, cardiac output falls and syncope may ensue.

Parasympathetic fibres project to the heart via the vagus and will release acetylcholine.
Sympathetic fibres release nor adrenaline and circulating adrenaline comes from the adrenal
medulla. Noradrenaline binds to β 1 receptors in the SA node and increases the rate of
pacemaker potential depolarisation.

Cardiac cycle

Image sourced from Wikipedia

 Mid diastole: AV valves open. Ventricles hold 80% of final volume. Outflow valves
shut. Aortic pressure is high.
 Late diastole: Atria contract. Ventricles receive 20% to complete filling. Typical end
diastolic volume 130-160ml.

 Early systole: AV valves shut. Ventricular pressure rises. Isovolumetric ventricular


contraction. AV Valves bulge into atria (c-wave). Aortic and pulmonary pressure
exceeded- blood is ejected. Shortening of ventricles pulls atria downwards and
drops intra atrial pressure (x-descent).

 Late systole: Ventricular muscles relax and ventricular pressures drop. Although
ventricular pressure drops the aortic pressure remains constant owing to peripheral
vascular resistance and elastic property of the aorta. Brief period of retrograde flow
that occurs in aortic recoil shuts the aortic valve. Ventricles will contain 60ml end
systolic volume. The average stroke volume is 70ml (i.e. Volume ejected).

 Early diastole: All valves are closed. Isovolumetric ventricular relaxation occurs.
Pressure wave associated with closure of the aortic valve increases aortic pressure.
The pressure dip before this rise can be seen on arterial waveforms and is called the
incisura. During systole the atrial pressure increases such that it is now above zero
(v- wave). Eventually atrial pressure exceed ventricular pressure and AV valves open
- atria empty passively into ventricles and atrial pressure falls (y -descent )

The negative atrial pressures are of clinical importance as they can allow air embolization to
occur if the neck veins are exposed to air. This patient positioning is important in head and
neck surgery to avoid this occurrence if veins are inadvertently cut, or during CVP line
insertion.

Mechanical properties

 Preload = end diastolic volume


 Afterload = aortic pressure

It is important to understand the principles of Laplace's law in surgery.

 It states that for hollow organs with a circular cross section, the total circumferential
wall tension depends upon the circumference of the wall, multiplied by the
thickness of the wall and on the wall tension.
 The total luminal pressure depends upon the cross sectional area of the lumen and
the transmural pressure. Transmural pressure is the internal pressure minus
external pressure and at equilibrium the total pressure must counterbalance each
other.
 In terms of cardiac physiology the law explains that the rise in ventricular pressure
that occurs during the ejection phase is due to physical change in heart size. It also
explains why a dilated diseased heart will have impaired systolic function.

Starlings law

 Increase in end diastolic volume will produce larger stroke volume.


 This occurs up to a point beyond which cardiac fibres are excessively stretched and
stroke volume will fall once more. It is important for the regulation of cardiac output
in cardiac transplant patients who need to increase their cardiac output.

Baroreceptor reflexes

 Baroreceptors located in aortic arch and carotid sinus.


 Aortic baroreceptor impulses travel via the vagus and from the carotid via the
glossopharyngeal nerve.
 They are stimulated by arterial stretch.
 Even at normal blood pressures they are tonically active.
 Increase in baroreceptor discharge causes:

*Increased parasympathetic discharge to the SA node.


*Decreased sympathetic discharge to ventricular muscle causing decreased contractility and
fall in stroke volume.
*Decreased sympathetic discharge to venous system causing increased compliance.
*Decreased peripheral arterial vascular resistance

Atrial stretch receptors

 Located in atria at junction between pulmonary veins and vena cava.


 Stimulated by atrial stretch and are thus low pressure sensors.
 Increased blood volume will cause increased parasympathetic activity.
 Very rapid infusion of blood will result in increase in heart rate mediated via atrial
receptors: the Bainbridge reflex.
 Decreases in receptor stimulation results in increased sympathetic activity this will
decrease renal blood flow-decreases GFR-decreases urinary sodium excretion-renin
secretion by juxtaglomerular apparatus-Increase in angiotensin II.
 Increased atrial stretch will also result in increased release of atrial natriuretic
peptide.

Which one of the following cells secretes the majority of tumour necrosis factor in
humans?

A. Neutrophils
B. Macrophages

C. Natural killer cells

D. Killer-T cells

E. Helper-T cells

Tumour necrosis factor

Tumour necrosis factor (TNF) is a pro-inflammatory cytokine with multiple roles in


the immune system

TNF is secreted mainly by macrophages and has a number of effects on the immune
system, acting mainly in a paracrine fashion:

 activates macrophages and neutrophils


 acts as costimulator for T cell activation
 key mediator of bodies response to Gram negative septicaemia
 similar properties to IL-1
 anti-tumour effect (e.g. phospholipase activation)

TNF-alpha binds to both the p55 and p75 receptor. These receptors can induce
apoptosis. It also cause activation of NFkB

Endothelial effects include increase expression of selectins and increased production


of platelet activating factor, IL-1 and prostaglandins

TNF promotes the proliferation of fibroblasts and their production of protease and
collagenase. It is thought fragments of receptors act as binding points in serum

Systemic effects include pyrexia, increased acute phase proteins and disordered
metabolism leading to cachexia

TNF is important in the pathogenesis of rheumatoid arthritis - TNF blockers (e.g.


infliximab, etanercept) are now licensed for treatment of severe rheumatoid

Which of the following is responsible for the rapid depolarisation phase of the
myocardial action potential?

A. Rapid sodium influx

B. Rapid sodium efflux


C. Slow efflux of calcium

D. Efflux of potassium

E. Rapid calcium influx

Electrical activity of the heart

Myocardial action potential


Phase Description Mechanism
0 Rapid depolarisation Rapid sodium influx
These channels automatically deactivate after a few ms
1 Early repolarisation Efflux of potassium
2 Plateau Slow influx of calcium
3 Final repolarisation Efflux of potassium
4 Restoration of ionic Resting potential is restored by Na+/K+ ATPase
concentrations There is slow entry of Na+ into the cell decreasing the
potential difference until the threshold potential is
reached, triggering a new action potential

NB cardiac muscle remains contracted 10-15 times longer than skeletal muscle

Conduction velocity
Atrial Spreads along ordinary atrial myocardial fibres at 1 m/sec
conduction
AV node 0.05 m/sec
conduction
Ventricular Purkinje fibres are of large diameter and achieve velocities of 2-4
conduction m/sec (this allows a rapid and coordinated contraction of the
ventricles
Which of the following is not a feature of normal cerebrospinal fluid?

A. It has a pressure of between 10 and 15 mmHg.

B. It usually contains a small amount of glucose.

C. It may normally contain up to 5 red blood cells per mm3.

D. It may normally contain up to 3 white blood cells per mm3.

E. None of the above


It should not contain red blood cells.

Cerebrospinal fluid

The CSF fills the space between the arachnoid mater and pia mater (covering surface
of the brain). The total volume of CSF in the brain is approximately 150ml.
Approximately 500 ml is produced by the ependymal cells in the choroid plexus
(70%), or blood vessels (30%). It is reabsorbed via the arachnoid granulations which
project into the venous sinuses.

Circulation
1. Lateral ventricles (via foramen Munro)
2. 3rd ventricle
3. Cerebral aqueduct (aqueduct Sylvius)
4. 4th ventricle (via foramina of Magendie and Luschka)
5. Subarachnoid space
6. Reabsorbed into venous system via arachnoid granulations in superior sagittal sinus

Composition

 Glucose: 50-80mg/dl
 Protein: 15-40 mg/dl
 Red blood cells: Nil
 White blood cells: 0-3 cells/ mm3

Which of the following is not an effect of somatostatin?

A. It stimulates pancreatic acinar cells to release lipase

B. It decreases gastric acid secretion

C. It deceases gastrin release

D. It decreases pepsin secretion

E. It decreases glucagon release

It inhibits pancreatic enzyme secretion.

Gastric secretions

A working knowledge of gastric secretions is important for surgery because peptic ulcers are
common, surgeons frequently prescribe anti secretory drugs and because there are still
patients around who will have undergone acid lowering procedures (Vagotomy) in the past.

Gastric acid

 Is produced by the parietal cells in the stomach


 pH of gastric acid is around 2 with acidity being maintained by the H+/K+ ATP ase
pump. As part of the process bicarbonate ions will be secreted into the surrounding
vessels.
 Sodium and chloride ions are actively secreted from the parietal cell into the
canaliculus. This sets up a negative potential across the membrane and as a result
sodium and potassium ions diffuse across into the canaliculus.
 Carbonic anhydrase forms carbonic acid which dissociates and the hydrogen ions
formed by dissociation leave the cell via the H+/K+ antiporter pump. At the same
time sodium ions are actively absorbed. This leaves hydrogen and chloride ions in
the canaliculus these mix and are secreted into the lumen of the oxyntic gland.

This is illustrated diagrammatically below:

Image sourced from Wikipedia


Phases of gastric acid secretion
There are 3 phases of gastric secretion:

1. Cephalic phase (smell / taste of food)

 30% acid produced


 Vagal cholinergic stimulation causing secretion of HCL and gastrin release from G
cells

2. Gastric phase (distension of stomach )

 60% acid produced


 Stomach distension/low H+/peptides causes Gastrin release

3. Intestinal phase (food in duodenum)

 10% acid produced


 High acidity/distension/hypertonic solutions in the duodenum inhibits gastric acid
secretion via enterogastrones (CCK, secretin) and neural reflexes.

Regulation of gastric acid production


Factors increasing production include:

 Vagal nerve stimulation


 Gastrin release
 Histamine release (indirectly following gastrin release) from enterchromaffin like
cells

Factors decreasing production include:

 Somatostatin (inhibits histamine release)


 Cholecystokinin
 Secretin

The diagram below illustrates some of the factors involved in regulating gastric acid
secretion and the relevant associated pharmacology
Image sourced from Wikipedia

Below is a brief summary of the major hormones involved in food digestion:

Source Stimulus Actions

Gastrin G cells in Distension of Increase HCL, pepsinogen and IF secretion,


antrum of stomach, extrinsic increases gastric motility, trophic effect on
the stomach nerves gastric mucosa
Inhibited by: low
antral pH,
somatostatin

CCK I cells in Partially digested Increases secretion of enzyme-rich fluid


upper small proteins and from pancreas, contraction of gallbladder
intestine triglycerides and relaxation of sphincter of Oddi,
decreases gastric emptying, trophic effect
on pancreatic acinar cells, induces satiety

Secretin S cells in Acidic chyme, fatty Increases secretion of bicarbonate-rich fluid


upper small acids from pancreas and hepatic duct cells,
intestine decreases gastric acid secretion, trophic
effect on pancreatic acinar cells

VIP Small Neural Stimulates secretion by pancreas and


intestine, intestines, inhibits acid and pepsinogen
pancreas secretion
Somatostatin D cells in the Fat, bile salts and Decreases acid and pepsin secretion,
pancreas and glucose in the decreases gastrin secretion, decreases
stomach intestinal lumen pancreatic enzyme secretion, decreases
insulin and glucagon secretion
inhibits trophic effects of gastrin, stimulates
gastric mucous production

Which of the following is the least likely to increase acid secretion in the stomach?

A. Calcium

B. Alcohol

C. Caffeine

D. Pear

E. None of the above

Gastric emptying

 The stomach serves both a mechanical and immunological function. Solid and
liquid are retained in the stomach during which time repeated peristaltic
activity against a closed pyloric sphincter will cause fragmentation of food
bolus material. Contact with gastric acid will help to neutralise any pathogens
present.

 The amount of time material spends in the stomach is related to its


composition and volume. For example a glass of water will empty more
quickly than a large meal. The presence of amino acids and fat will all serve to
delay gastric emptying.

Controlling factors
Neuronal stimulation of the stomach is mediated via the vagus and the
parasympathetic nervous system will tend to favor an increase in gastric motility. It is
for this reason that individuals who have undergone truncal vagotomy will tend to
routinely require either a pyloroplasty or gastro-enterostomy as they would otherwise
have delayed gastric emptying.

The following hormonal factors are all involved:


Delay emptying Increase emptying
Gastric inhibitory peptide Gastrin
Cholecystokinin
Enteroglucagon

Diseases affecting gastric emptying


All diseases that affect gastric emptying may result in bacterial overgrowth, retained
food and eventually the formation of bezoars that may occlude the pylorus and make
gastric emptying even worse. Fermentation of food may cause dyspepsia, reflux and
foul smelling belches of gas.

Iatrogenic
Gastric surgery can have profound effects on gastric emptying. As stated above any
procedure that disrupts the vagus can cause delayed emptying. Whilst this is
particularly true of Vagotomy this operation is now rarely performed. Surgeons are
divided on the importance of vagal disruption that occurs during an oesophagectomy
and some will routinely perform a pyloroplasty and other will not.

When a distal gastrectomy is performed the type of anastomosis performed will


impact on emptying. When a gastro-enterostomy is constructed, a posterior, retrocolic
gastroenterostomy will empty better than an anterior one.

Diabetic gastroparesis
This is predominantly due to neuropathy affecting the vagus nerve. The stomach
empties poorly and patients may have episodes of repeated and protracted vomiting.
Diagnosis is made by upper GI endoscopy and contrast studies, in some cases a radio
nucleotide scan is needed to demonstrate the abnormality more clearly. In treating
these conditions drugs such as metoclopramide will be less effective as they exert
their effect via the vagus nerve. One of the few prokinetic drugs that do not work in
this way is the antibiotic erythromycin.

Malignancies
Obviously a distal gastric cancer may obstruct the pylorus and delay emptying. In
addition malignancies of the pancreas may cause extrinsic compression of the
duodenum and delay emptying. Treatment in these cases is by gastric decompression
using a wide bore nasogastric tube and insertion of a stent or if that is not possible by
a surgical gastroenterostomy. As a general rule gastroenterostomies constructed for
bypass of malignancy are usually placed on the anterior wall of the stomach (in spite
of the fact that they empty less well). A Roux en Y bypass may also be undertaken but
the increased number of anastomoses for this in malignant disease that is being
palliated is probably not justified.

Congenital Hypertrophic Pyloric Stenosis


This is typically a disease of infancy. Most babies will present around 6 weeks of age
with projectile non bile stained vomiting. It has an incidence of 2.4 per 1000 live
births and is more common in males. Diagnosis is usually made by careful history and
examination and a mass may be palpable in the epigastrium (often cited seldom felt!).
The most important diagnostic test is an ultrasound that usually demonstrates the
hypertrophied pylorus. Blood tests may reveal a hypochloraemic metabolic alkalosis
if the vomiting is long standing. Once the diagnosis is made the infant is resuscitated
and a pyloromyotomy is performed (usually laparoscopically). Once treated there are
no long term sequelae.

Anda mungkin juga menyukai